You are on page 1of 241

TRATAMIENTO DE LA SEAL I

ESCUELA POLITCNICA SUPERIOR


Universidad de Lleida
(Revisin 2015)
F. Clari

TABLA DE CONTENIDO

1 SEALES Y ANLISIS DE FOURIER ............................................................................. 1


1.1 INTRODUCCIN ............................................................................................................ 1
1.2 SEALES ......................................................................................................................... 1
1.2.1 COMPARACIN DE SEALES ............................................................................... 4
1.3 APROXIMACIN DE UNA FUNCIN MEDIANTE UN CONJUNTO DE
FUNCIONES ORTONORMALES. ....................................................................................... 7
1.4 DESARROLLO EN SERIE DE FOURIER .................................................................... 10
1.4.1 PECULIARIDADES DEL DESARROLLO EN SERIE DE FOURIER .................... 15
2 TRANSFORMADA DE FOURIER Y SU APLICACIN .............................................. 21
2.1 CONVOLUCIN DE DOS SEALES .......................................................................... 21
2.2 TRANSFORMADA DE FOURIER ............................................................................... 24
2.2.1 ALGUNAS CONVERSIONES Y VERSIONES ALTERNATIVAS............................. 25
2.3 TRANSFORMADAS DE FOURIER DE ALGUNAS FUNCIONES DE INTERS .... 27
2.4 PROPIEDADES DE LA TRANSFORMADA DE FOURIER ....................................... 30
3 DENSIDAD ESPECTRAL Y CORRELACIN .............................................................. 50
3.1 ENERGA DE UNA SEAL.......................................................................................... 50
3.2 DENSIDAD ESPECTRAL DE ENERGA .................................................................... 50
3.3 DENSIDAD ESPECTRAL DE POTENCIA .................................................................. 52
3.4 CORRELACIN DE DOS SEALES DE ENERGA FINITA .................................... 53
3.5 CORRELACIN DE DOS SEALES DE POTENCIA MEDIA FINITA .................... 55
3.5.1 ALGUNAS PROPIEDADES DE LA CORRELACIN Y DE LA DENSIDAD
ESPECTRAL ..................................................................................................................... 56
3.6 TRANSFORMADA DE HILBERT Y SEAL ANALTICA........................................ 58
3.6.1 SEAL ANALTICA................................................................................................. 59
3.6.2 ENVOLVENTE, FASE Y FRECUENCIA INSTANTNEA DE UNA SEAL REAL60
3.6.3 SEAL REAL PASO BANDA EN FUNCIN DE SEALES PASO BAJO............. 61
3.7 EL TEOREMA DEL MUESTREO ................................................................................ 61
3.8 TRANSFORMADA DISCRETA DE FOURIER ........................................................... 67
3.9 CONVOLUCIN Y CORRELACIN DISCRETAS.................................................... 75
4 MODULACIONES ANALGICAS ................................................................................. 84
4.1 MODULACIONES ........................................................................................................ 84
4.2 MODULACIONES ANALGICAS DE AMPLITUD .................................................. 84
4.2.1 MODULACIN EN DOBLE BANDA LATERAL CON PORTADORA SUPRIMIDA
.......................................................................................................................................... 85
4.2.1.1 DEMODULACIN ..................................................................................... 87
4.2.2 MODULACIN EN CUADRATURA DE DOBLE BANDA LATERAL SIN
PORTADORA ................................................................................................................... 94
4.2.3 MODULACIN EN DOBLE BANDA LATERAL CON PORTADORA .................. 95
4.2.4 MULTIPLEXADO POR DIVISIN EN FRECUENCIA ....................................... 102
4.2.5 MODULACIN EN BANDA LATERAL NICA .................................................. 104
4.3 MODULACIONES ANALGICAS ANGULARES................................................... 110
4.3.1 ANLISIS ESPECTRAL ........................................................................................ 113
i

4.3.2 MODULADORES DE FM Y DE PM .................................................................... 117


4.3.2.1 MODULACIN DE PM DE BANDA ESTRECHA ................................ 118
4.3.3 DEMODULACIN DE FM Y PM ........................................................................ 121
4.3.4 MTODOS DE EXTENSIN DE UMBRAL. ........................................................ 125
Comparador ................................................................................................................ 137
h(t)=1/t ...................................................................................................................... 138
d(t)=c(t)2 ..................................................................................................................... 138
e(t)=xAM(t)2 ................................................................................................................. 138
5 MODULACIONES DE PULSOS .................................................................................... 167
5.1 INTRODUCCIN ........................................................................................................ 167
5.2 MODULACIONES ANALGICAS DE PULSOS ...................................................... 168
5.2.1 MODULACIN EN AMPLITUD DE PULSOS (PAM) ........................................ 169
5.2.2 MODULACIN PULSOS EN DURACIN (PDM) .............................................. 172
5.2.3 MODULACIN DE PULSOS EN POSICIN (PPM) .......................................... 172
5.3 MULTIPLEXADO POR DIVISIN EN EL TIEMPO ................................................ 173
5.4 MODULACIN POR PULSOS CODIFICADOS ....................................................... 185
5.4.1 CUANTIFICACIN Y CODIFICACIN .............................................................. 185
5.4.2 RUIDO DE CUANTIFICACIN ........................................................................... 187
5.4.3 TRANSMISIN Y RECEPCIN DE LA SEAL PCM ......................................... 188
5.4.4 MULTIPLEXADO POR DIVISIN EN EL TIEMPO DE SEALES PCM .......... 190
5.4.5 DETECCIN DE PALABRAS DE CDIGO CON FILTRO ADAPTADO .......... 193
5.4.6 UMBRALES DE ERROR ....................................................................................... 195
5.4.7 COMPORTAMIENTO CUALITATIVO DE LA SEAL PCM............................... 196
5.5 CDIGOS DE DETECCIN Y CORRECCIN DE ERROR..................................... 197
6 RUIDO ALEATORIO ...................................................................................................... 205
6.1 PROCESOS ALEATORIOS ........................................................................................ 205
6.2 PROCESOS ALEATORIOS ESTACIONARIOS ........................................................ 208
6.2.1 ERGODICIDAD .................................................................................................... 209
6.3 RUIDO .......................................................................................................................... 211
6.3.1 RUIDO TRMICO ................................................................................................ 212
6.3.2 RUIDO BLANCO .................................................................................................. 214
6.3.3 COMENTARIOS .................................................................................................... 216
6.4 DENSIDAD ESPECTRAL DE POTENCIA DE RUIDO DISPONIBLE .................... 218
6.4.1 TEMPERATURA EFECTIVA DE RUIDO ............................................................ 221
6.4.2 FILTRADO DEL RUIDO BLANCO ...................................................................... 221
6.4.3 ANCHO DE BANDA EQUIVALENTE DE RUIDO .............................................. 224
6.5 RUIDO GENERADO EN UN CUADRIPOLO ............................................................ 226
6.5.1 TEMPERATURA EFECTIVA, FACTOR Y FIGURA DE RUIDO ........................ 228
6.5.2 RUIDO DE CUADRIPOLOS EN SERIE .............................................................. 232
7 REFERENCIAS BIBLIOGRFICAS ............................................................................ 237

ii

INTRODUCCIN
Este texto, Tratamiento de la seal I, creado como documentacin base para estudiantes de
ingeniera mecnica, electrnica o informtica, presenta una exposicin de tcnicas en tiempo
y en frecuencia del tratamiento y modulacin de seales, reflejando la creciente importancia
de estos principios que se aplican al diseo de sistemas electrnicos, tanto de control como de
transmisin de la informacin y hace hincapi, principalmente, en mtodos analgicos.
En los tres primero captulos de este texto se presentan los conceptos bsicos de procesado
de la seal en tiempo y en frecuencia, para que este texto tambin pueda ser usado por
aquellos estudiantes que proviniendo de otras disciplinas afines estn interesados en aprender,
revisar o actualizar sus conocimientos en la teora del tratamiento y modulacin de la seal.
Se utilizan frecuentemente ejemplos y ejercicios de aplicacin, que facilitan el proceso de
aprendizaje, siendo prerrequisito para aprovechar los conceptos presentados, haber cursado
las materias de clculo integral bsicas en ingeniera a la vez que resulta muy conveniente
tener conocimientos de electrnica bsica, anlisis de circuitos y sistemas lineales.
Las modulaciones digitales de amplitud, de frecuencia y de fase, entropa de la
informacin, capacidad de un canal y el estudio de sistemas de espectro expandido no han
sido incluidas en este documento y son, entre otras explicaciones, materia que se presentan en
el documento Tratamiento de la seal II, orientada a cursos de especialidad posteriores.

iii

Equation Section 1

CAPTULO 1
1 SEALES Y ANLISIS DE FOURIER
1.1 INTRODUCCIN
Definiendo seal como la representacin fsica de una informacin, tratamiento o
procesado de la seal consistira en manipular sta para dotarla o extraer de ella propiedades
tiles. Estas propiedades estn orientadas a la comunicacin, almacenamiento y presentacin
de la informacin. Cuantitativamente hablando, informacin es todo aquello que puede
incrementar nuestro nivel de conocimiento, y un trasvase de informacin implica
comunicacin que es el proceso por el que se transfiere informacin de una fuente a un
destinatario a travs de un sistema mas o menos complejo.
El procesado de la seal est orientado tambin a facilitar la interaccin directa o indirecta
con los sistemas fsicos; stos, en un sentido amplio, son una interconexin de componentes,
dispositivos o subsistemas. En contextos que van del procesamiento de seales y
comunicaciones a motores electromecnicos, vehculos y plantas de procesos qumicos, un
sistema puede considerarse como un proceso en el cual las seales de entrada son
transformadas por el sistema o provocan que ste responda de alguna forma, lo que da como
resultado otras seales como salida. El conocimiento de cmo tratar estas seales resulta
determinante cuando se pretende disear sistemas o simularlos.

1.2 SEALES
La clasificacin de las seales es algo bastante relativo, puede efectuarse segn sea el
punto de vista del autor de la clasificacin o con miras a algn objetivo concreto. En general
puede darse una clasificacin como la siguiente:
1.- Segn su evolucin a lo largo del tiempo
2.- Segn su energa
3.- Segn su morfologa
4.- Segn su espectro
5.- Otros aspectos

Maticemos cada uno de los aspectos anteriores.


1.- Segn su evolucin a lo largo del tiempo
Debe distinguirse entre dos clases:
A.- Deterministas, son aquellas que con determinado conocimiento acerca de ellas, quedan
absolutamente especificadas para cualquier instante de tiempo; dicho de otra forma, las que
admiten una expresin analtica. A su vez estas pueden clasificarse en,
- Peridicas: x(t)=x(t-kT)
Sinusoidales: x(t)= cos(2ft).

Peridico-compuestas: x0 An cos(2f 0 t n )
n 1

Pseudo aleatorias: Las secuencias aleatorias se repiten peridicamente.


- No peridicas
Cuasi-peridicas: ejemplo la seal ECG
Transitorias: Propias de cambios fsicos seguidos de una estabilizacin.
B.- Aleatorias. Estas seales pueden tomar en cada instante de tiempo uno de entre un
conjunto de valores con una probabilidad preferible; pero no es posible decidir con certeza
total que valor toman si no es mediante observacin directa. No tienen un modelo
matemtico. La comunicacin solo tiene sentido usando seales que, de alguna manera, sean
aleatorias; sin embargo el aprendizaje pasa por la consideracin de seales deterministas ya
que sirven para asentar las bases conceptuales para abordar el estudio de la otra clase de
seales; y adems muchas de las conclusiones obtenidas con seales deterministas son de
inmediata aplicacin con las aleatorias. A su vez estas pueden clasificarse en,
- Estacionarias: Existen criterios matemticos que especifican cuando una seal
aleatoria puede considerarse estacionaria o no, pero de una manera ms coloquial podemos
decir que en una seal estacionaria su espectro no cambia ostensiblemente con el tiempo.
- No estacionarias.
2.- Segn su energa. Pueden clasificarse en seales de

- Energa finita: W x

x(t )

dt y W x

1 2
2
T x (t ) dt y PT
T T
2

- Potencia media finita: PT lim

3.- Segn su morfologa


Para efectuar claramente la distincin se debe considerar el dominio que es el conjunto de
instantes de tiempo en los que la seal est definida y el rango que es el conjunto de valores
que toma la seal en su dominio. El grfico siguiente muestra esta clasificacin.

Discreto

Tiempo

Continuo

Amplitud
Continua

Discreta

Seal analgica

Seal mantenida

Seal muestreada

Seal digital
Convert. A/D

4.- Segn su espectro en seales,


-Paso bajo
-Paso alto
-Paso banda
-Banda eliminada
5.- Otros matices
- Seales de tiempo limitado
- Pares: x(t)=x(-t)
- Impares: x(t)=-x(-t)
- Causales: x(t)=0 para t<0

Cuando se habla de causalidad de seales parece tambin pertinente hacer comentarios


acerca de sistemas causales.
Un sistema es causal si su salida en cualquier instante del tiempo depende solo de valores
de la entrada en el momento presente y en el pasado.
Aunque lo sistemas causales son de gran importancia, de ninguna manera constituyen los
nicos sistemas que resultan de inters prctico. Por ejemplo la causalidad no es de
importancia fundamental en aplicaciones como el procesado de imgenes, en las que la
variable independiente no es el tiempo. En el procesado de datos que han sido grabados
previamente, es decir cuando no se trabaja en tiempo real tampoco estamos obligados a
procesar estos datos de forma causal.

1.2.1 COMPARACIN DE SEALES


La comparacin de funciones es una actividad muy comn y til en procesado de la seal.
El concepto de ortogonalidad se revela como algo fundamental y permite cuantificar esta
comparacin.
Cuando se habla de vectores el concepto de ortogonalidad se muestra muy claro y preciso.
Se dice que dos vectores son ortogonales cuando ninguno de ellos tiene componentes en la
direccin del otro, es decir que son perpendiculares entre s.
Una forma de comparar dos vectores es calculando su producto escalar. Por ejemplo se

dispone del vector A a1 x a 2 y a1 z y del vector B b1 x b2 y b1 z su producto escalar


es A B a1b1 a 2 b2 a3 b3 que es lo mismo que A B A B cos( ) . Por otra parte
B cos( ) es la proyeccin de B sobre A, es decir cuanto se parece B a A, cosa que se podra

escribir
k

tambin

A B
A

como

B cos( ) k A .

As

A B k A

A B a1b1 a 2 b2 a3 b3

A A
a12 a 22 a 32

Si estos vectores son idnticos este coeficiente vale uno. Si los dos vectores son
perpendiculares su producto escalar es cero y por tanto este coeficiente que mide el parecido
tambin.

Cuando se trata de seales, el procedimiento es semejante. Por ejemplo para dos seales
x(t) e y(t) desendose medir el parecido de y(t) con respecto a x(t) en un intervalo de tiempo
t1-t2 la expresin a utilizar sera,

t2

x(t ) y (t )dt

t1

t2

t1

(1.1)

x(t ) dt

que es semejante a la que se ha usado con vectores.


Por analoga se puede decir que la seal y(t) tiene una componente x(t) y que esta
componente tiene una magnitud k. Si esta componente fuese cero las dos seales seran
ortogonales entre si. Tambin se podra decir que su producto escalar es cero.
Con este resultado se podra aproximar una funcin con la otra, es decir y(t) kx(t). En
este caso faltara comprobar que k es el mejor coeficiente posible con algn determinado
criterio. La funcin error que se obtiene al tomar la aproximacin es e(t) = y(t) - kx(t) y la
mejor forma de cuantificarlo () es promediar esta funcin error en todo el intervalo t1-t2 en
que se desea que la aproximacin sea vlida. Sin embargo podra suceder que esta funcin
error tuviera valores positivos y negativos que se compensasen de tal forma que su promedio
en el intervalo fuera prcticamente nulo, lo que parecera falsamente una buena eleccin de k.
Si se realiza el promediado sobre el cuadrado de la funcin de error se evita esta situacin.
Esto es,

1
t 2 t1

y(t ) kx(t )
t2

t1

dt

(1.2)

Como que nuestra intencin es minimizar este error en funcin del valor de k debemos
hacer

d
0 . En esta operacin la integral se realiza con respecto al t y la derivada con
dk

respecto a k, por esto nada impide cambiar el orden en que se efectan estas operaciones.
t2
t2

d
1 t2 dy (t ) 2

dt 2 y (t ) x(t )dt 2k x(t ) 2 dt 0


t1
t
t
1
1
dk t 2 t1
dk

Como la primera integral es nula, al despejar k se obtiene la expresin 1.1. Por tanto el
coeficiente hallado es el mejor de los coeficientes posibles segn criterio del mnimo del error
cuadrtico.
Si k fuese cero tambin podramos decir que las seales x(t) e y(t) son ortogonales en el
intervalo considerado. Esto es,
5

t2

t1

x(t ) y (t )dt 0

Ejemplo.

Se desea aproximar la funcin y(t) dada en la grfica por la funcin x(t)=sen(t) en el


intervalo [0-2]. Que amplitud debe tener la funcin x(t)?
y(t)
1

2
t

-1

Figura 1.1
Solucin.
La aproximacin es y(t) kx(t) o lo que es lo mismo y(t) k sen(t). As pues debemos
encontrar el valor del coeficiente k. Utilizando la expresin (1.1),
k

y (t ) sen(t )dt

As, y (t )

sen(t ) 2 dt

2
1
4

sen(
)
sen(t )dt
t
dt

sen(t )
y(t)

4/ sen(t)

4/
1

2
t

-1
-4/
Figura 1.2
Esta aproximacin reduce al mnimo el error cuadrtico medio.

Conocido es, en el espacio de vectores, el hecho que cualquiera de estos puede expresarse
como una suma de sus componentes contenidas en un cierto nmero de vectores, que
llamamos base, caracterizados por ser mutuamente ortogonales. Tambin se podra expresar
cualquier funcin f(t) como una suma de componentes sobre un conjunto completo de
funciones ortogonales entre s.

1.3 APROXIMACIN DE UNA FUNCIN MEDIANTE UN CONJUNTO DE


FUNCIONES ORTONORMALES.

Supngase que se dispone de un conjunto de n funciones g1 g2 ...gn ortogonales entre s en


el intervalo t1-t2. Esto es,

t2

t2

t1

t1

g j (t ) g k (t )dt 0

para j k y

g j (t ) g k (t )dt K j

para k = j

y se desea que una funcin cualquiera f(t) sea aproximada en el intervalo por una
combinacin lineal de las n funciones ortogonales. Esto es,
n

f (t ) d1 g1 (t ) d 2 g 2 (t ) d 3 g 3 (t ) ......... d n g n (t ) d l g l (t )
l 1

El error se definir de la misma forma que en el apartado anterior, ecuacin (1.2), para un
solo coeficiente.
1

t 2 t1

f (t ) d l g l (t ) dt
l 1

t2

t1

Como que este error es funcin de los n coeficientes, cada una de las derivadas parciales
del error respecto a cada coeficiente debe ser cero. Esto es,

.......
0
d1 d 2
d n
Hagamos la operacin para un coeficiente genrico j y su resultado ser vlido para
cualquier coeficiente.

d j

d j

t2

t1

t2

t1

f (t ) d l g l (t ) dt 0
l 1

2
n

n

2
f (t ) 2 f (t ) d l g l (t ) d l g l (t ) dt 0
l 1
l 1

(1.3)
7

En la ecuacin (1.3) el primer sumando de la integral (f(t)2) que es la funcin original no


contiene coeficientes dn por tanto su derivada con respecto a cualquiera de ellos es cero. Del
n

segundo sumando, 2 f (t ) d l g l (t ) , solo contribuir a la derivada el trmino que contenga dj.


l 1

As este sumando puede quedar reducido a 2 f (t )d j g j (t ) . Por ltimo en el tercer sumando,

que por estar elevado al cuadrado contendr una suma de todos los posible trminos
d k g k (t )d m g m (t ) que al ser integrados darn Kj o cero dependiendo de que k sea igual m o
que sea distinto, puesto que las funciones g(t) son ortogonales. La expresin anterior con estas
consideraciones es,

d j

2 f (t )d
t2

t1

g j (t ) d 2j g 2j (t ) dt 0

t2

t2

t1

t1

2 f (t ) g j (t )dt 2d j g 2j (t )dt
dj

t2

t1

f (t ) g j (t )dt

t2

t1

g 2j (t )dt

(1.4)

Y por ltimo,
dj

1
Kj

t2

t1

f (t ) g j (t )dt

(1.5)

De modo que los coeficientes dn se obtienen calculando la expresin anterior.


Una vez calculados los coeficientes se puede obtener el valor del error mediante la
expresin,

1
t2 t1

f (t )
t2

t1

(d12 K1 d 22 K 2 d32 K3 ........ d n2 K n ) dt

Si n tiende a infinito el error tiende a cero.

Ejemplo.

Se desea aproximar la funcin y(t) dada en la grfica, figura 1.1, del ejemplo anterior por
un conjunto de funciones gj(t) de la forma sen(2nf0t) que son ortogonales en el intervalo [t0,
t0+1/f0] para todos los valores enteros de n. Que valor deben tener las amplitudes de las
funciones?

Solucin.
La funcin aproximada ser,
y (t ) d1 sen(2f 0 t ) d 2 sen(2 2 f 0 t ) d 3 sen(2 3 f 0 t ) ...........d n sen(2nf 0 t )
La expresin de los dk, ecuacin (1.4), es d k

t2

y (t ) g j (t )dt

t1

t2

t1

g k2 (t )dt

para evaluarla calculemos

primero su denominador.

T0

sen 2 (2 kf 0t )dt

T0

1 1
T
cos(2 2kf 0t )dt 0 . As que el producto escalar de dos de
2 2
2

estas funciones vale T0/2 cuando presentan el mismo ndice (misma frecuencia) y se puede
comprobar que valen cero cuando el ndice es distinto (diferente frecuencia), puesto que son
ortogonales.
As los coeficientes son,
dk

2
T0

T0

y (t ) sen(2 kf 0t )dt

T0

2 T0 2
sen(2 kf 0t )dt T0 sen(2 kf 0t )dt

T0 0
2

4
2

1 2 cos(k ) cos(2k ) k
2 kf 0T0
0

para k impar
para k par

Por lo tanto la funcin y(t) obtenida por este conjunto de funciones es exacta cuando n es
infinito,
y (t )

sen(2f 0 t )

4
4
sen(2 3 f 0 t )
sen(2 5 f 0 t ) ...........
3
5

Si se toman solo los tres primeros trminos la seal y(t) aproximada es la que se muestra
en la figura siguiente.
aproximacin

y(t)
1
T0/2

T0
t

-1

Figura 1.3

El conjunto de funciones sen(2nf0t) para n entero, no es un conjunto ortogonal completo.


Si se calcula el producto escalar de sen(2nf0t) y cos(2mf0t) comprobaremos que da cero,
luego se pueden aadir ms funciones al conjunto de senos. Para que el conjunto de funciones
sinusoidales sea completo debe estar formado por las funciones seno y sus correspondientes
funciones coseno. As cualquier funcin f(t) puede representarse en un intervalo como una
combinacin lineal de senos y cosenos de diferentes frecuencias.
Por otra parte, si se escoge un conjunto gn(t) de funciones complejas mutuamente
ortogonales en algn intervalo temporal [t1, t2], cuyo producto escalar es,

t2

t1

para m n

0
g m (t ) g n (t )dt
K m

para m n

siendo gn*(t) la funcin conjugada de gn(t)

se puede demostrar que los coeficientes o las amplitudes de las componentes para
aproximar una funcin f(t) estn expresados por,
dn

1
Kn

t2

t1

f (t ) g n (t )dt

(1.6)

Resultado muy similar a lo que se ha comentado para funciones reales.

1.4 DESARROLLO EN SERIE DE FOURIER

Con estos ltimos comentarios del apartado anterior no se pretende ms que introducir de
una

forma

natural

el

conjunto

de

funciones

exponenciales

complejas

g n (t ) e j 2nf 0t cos(2nf 0 t ) j sen(2nf 0 t ) . Este conjunto es ortogonal en un intervalo [t0,


t0+T0], con T0 el periodo de la seal sinusoidal para n=1, es decir es ortogonal en el periodo
de lo que se suele llamar la seal fundamental.
Puesto que este conjunto de funciones ortogonales ser ampliamente utilizado,
comprobemos cual es el valor de su producto escalar.

T0

T0

e j 2nf 0t e j 2mf 0t dt e j 2 ( n m ) f 0t dt
0

Si n=m entonces la integral vale T0


Si n es distinto de m entonces,

T0

e j 2 ( n m ) f 0t dt

1
e j 2 ( n m ) f 0t
j 2 (n m) f 0

T0
0

1
e j 2 ( n m ) 1 0
j 2 (n m) f 0

10

puesto que si n y m son enteros su diferencia tambin es un entero que al ser multiplicado
por 2 proporciona una exponencial compleja cuyo valor es siempre uno.
Por tanto su producto escalar es,

t 0 T0

t0

para m n

0
e j 2nf 0t e j 2mf 0t dt
T0

para m n

y sus coeficientes se calcularn mediante la

expresin,
dn

1
T0

t 0 T0

t0

f (t )e j 2mf 0t dt

(1.7)

De modo que se podra utilizar este conjunto de funciones para aproximar una seal f(t) en
un intervalo mediante combinacin lineal de las mismas. Si se toma un nmero infinito de
coeficientes el error en la aproximacin es cero.
f (t ) .....d 3e j 2 3 f 0t d 2 e j 2 2 f 0t d 1e j 2f 0t d 0 d1e j 2 1 f 0t d 2 e j 2 2 f 0t d 3e j 2 3 f 0t .......
As, f (t )

e j 2nf 0t representa la funcin f(t) desarrollada en serie de funciones

exponenciales. Este desarrollo se llama desarrollo en serie de Fourier.


El hecho que una seal f(t) pueda ser aproximada, incluso de forma exacta, en un intervalo
[t0, t0+T0] hace pensar que una seal que se repita indefinidamente con este intervalo, como
puede ser una seal peridica, puede ser aproximada tambin de forma exacta sin restriccin
de intervalo. Esto resulta intuitivo puesto que la aproximacin realizada con los desarrollos
expuestos anteriormente para un intervalo, que resulta ser el periodo de la seal, puede ser
aplicada idnticamente a cada uno los infinitos periodos de la seal.
Podemos enunciar que cualquier seal peridica f(t) puede ser presentada como una
combinacin lineal de infinitas funciones exponenciales complejas. Esto es,

f (t )

d e

Las
1
dn
T

amplitudes
T

f (t )e

j 2nf 0t

de

estos

j 2nf 0t

coeficientes

son

obtenidas

de

la

expresin

(1.7),

dt evaluada en un periodo.

11

Ejemplo.

Desarrollar en serie de Fourier la seal f (t )

t nT
cuya grfica se muestra a
T

continuacin.
f(t)
f(t)
1

- T/ 2 - T/ 4 T/ 4 T/ 2

Figura 1.4
Solucin.
La expresin general del desarrollo de una funcin en serie de Fourier es
f (t )

d ne

j 2nf 0t

1
con d n
T

f (t )e

j 2nf 0t

dt . Teniendo en cuenta que f0=1/T, dn se

desarrolla como sigue:


1
dn
T

j 2nf 0t

dn

jn
jn
e j 2nf0t 4
1 e 2 e 2

j
2

nf
T
j 2nf 0
0 T

1
dt
T

sen n

sen n
21
2 1 sinc n

n
n
2
2
2
2

Los valores dn son muestras de la grfica de la figura siguiente:


1/2

dn

1/

-5

-3 -1 1

1/

Figura 1.5

12

Debe hacerse notar la peculiar forma de esta grfica cuando se interpolan los valores de los
coeficientes dn.
As pues:
d n (...0,

1
1 1 1
1
,0, , , ,0, ,0,.....)
3
3 2

Sustituyendo dn ,que es una funcin par, en la expresin general tenemos que:


f (t ) ...
f (t )

1 j 2 3 f0t 1 j 2f0t 1 1 j 2f0t 1 j 2 3 f0t


e
e
e
e
....

3
2
3

1 1 j 2 f0t j 2 f0t
1 j 2 3 f0t j 2 3 f0t
)
(e
).....
(e
e
e
2
3

Y escribiendo la expresin anterior de una forma mas compacta:

f (t )

1 2 (1) n1

cos 2 (2n 1) f 0t
2 n1 2n 1

Ejercicio.

Desarrollar en serie de Fourier la seal f (t )

t T nT
n
4
2 cuya grfica se

(
1
)

T
n
2

muestra a continuacin.
f(t)
1

t
-1

Figura 1.6

13

La expresin general del desarrollo de una funcin en serie de Fourier es


f (t )

d ne

j 2nf 0t

1
con d n
T

f (t )e

j 2nf 0t

dt . Teniendo en cuenta que f0=1/T, dn se

desarrolla como sigue:


T
0

2
1
d n e j 2nf0t dt e j 2nf0t dt
T
0
T 2

e j 2nf0t 2
1 e j 2nf0t
dn

T j 2nf 0 T
j 2nf 0 0

dn

sen 2

dn

1 1
T j 2nf 0

dn

1
2
n
(2 2 cos n)
sen 2
2
j 2nf 0T
jn

1 e j 2nf0 T 2 1 e j 2nf0 T 2

2 n sinc 2 n ,
2 j n 2
2j
2

2

con n=..-3,-2,-1,0,1,2,3,4....

As que :
f (t )

n sinc
2j
n

n j 2nf 0t
e
2

Como que la funcin dn es impar, f(t) se puede escribir de la forma:

f (t ) n sinc 2
n 1

f (t )

sen 2f 0t

n
sen(2nf 0t )
2

4
4
sen 2 3 f 0t
sen 2 5 f 0t...
5
3

Tambin puede escribirse de una forma ms compacta,


f (t )

2n 1 sen 2 (2n 1) f t

n 1

14

1.4.1 PECULIARIDADES DEL DESARROLLO EN SERIE DE FOURIER


Cuando se calcula el desarrollo en serie de Fourier de una funcin peridica en el tiempo
en realidad estamos presentando esta funcin del tiempo como una combinacin lineal de sus
componentes de frecuencia, es decir estamos expresando esta funcin mediante su espectro.
Notemos tambin que las seales peridicas tienen componentes de frecuencia discretas,
concretamente la frecuencia fundamental y frecuencias mltiplo de la fundamental. As el
espectro no es una curva continua si no que existe solo en algunos valores discretos de la
frecuencia, por consiguiente se llama espectro discreto o de lneas.
La figura siguiente muestra el espectro de la seal peridica de la figura 1.4 entre n=-20 y
n=20 cuyos coeficientes estn dados por d n

1
n 1 sen( n 2)
. En esta figura el eje x
sinc
2
2 2 n 2

representan las frecuencias nf0 y el eje y las amplitudes de estas componentes.


0.5

0.4

amplitud

0.3

0.2

0.1

-0.1

-0.2
-20

-15

-10

-5

0
n

10

15

20

Figura 1.7
El espectro no necesariamente debe estar formado por valores reales, pueden ser
imaginarios o complejos. En este caso sera necesario presentarlo mediante dos grficas,
normalmente una para el mdulo y otra para su fase. Como ejemplo consideremos los
coeficientes de la seal peridica de onda cuadrada con valor medio nulo (figura 1.6 ),
coeficientes que estn expresados por,
n n sen( n 2)

dn
sinc

2j
2 2 j n 2)

abs(n)
cuyo mdulo seria d n
2

sen( n 2)

y su fase sera /2 para n negativo y -/2

n
2
)

para n positivo.
15

Las grficas siguientes muestran esta situacin.


2

0.7

1.5

0.6

0.5

amplitud

fase

0.5
0

0.4

0.3

-0.5

0.2
-1

0.1

-1.5
-2
-20

-15

-10

-5

0
n

10

15

20

0
-20

-15

-10

-5

a)

0
n

10

15

20

b)

Figura 1.8: a) Espectro de fase b) Espectro de amplitud


Destacar que el mdulo del espectro de cualquier seal real, como por ejemplo las de los
ejercicios anteriores, siempre tendr simetra con referencia a la frecuencia cero, es decir ser
una funcin par y su fase ser impar.
Otro hecho destacable que se observar a continuacin consiste en que si el periodo
aumenta con respecto a la anchura efectiva de la funcin generatriz de la seal peridica, la
densidad del espectro aumenta. Hagamos el siguiente ejercicio como ejemplo.

Ejemplo.

t nT
cuya grfica se muestra a
Desarrollar en serie de Fourier la seal f (t )

n T
10

continuacin.
Solucin.
La grfica de la seal peridica es,
f(t)
1

.............

.............
T

T/10

Figura 1.9
16

Es un tren de pulsos, semejante al ejemplo realizado anteriormente, sin embargo el ancho


de pulso es considerablemente ms estrecho.
La

expresin

es f (t )

d ne

general

j 2nf 0t

del

1
con d n
T

desarrollo
T

f (t )e

j 2nf 0t

de

una

funcin

en

serie

de

Fourier

dt

Recordando que f0=1/T, dn se desarrolla como sigue y teniendo en cuenta que en todo el
periodo de integracin solo hay valor no nulo entre -T/20 y T/20 entonces,
T

jn
jn
1 20 j 2nf 0t
1 e j 2nf 0t 20
1 e 10 e 10

dn
e
dt

T T
T j 2nf 0 T
T
j 2nf 0

20
20

dn

sen n

sen n
10 1
10 1 sinc n
n
10 n
10
10
10

La grfica de dn es:
0.1

0.08

amplitud

0.06

0.04

0.02

-0.02

-0.04
-40

-30

-20

-10

0
n

10

20

30

40

Figura 1.10

El espectro de amplitud de la figura 1.10, aunque con un nivel menor (valor mximo 0.1),
tiene lgicamente la misma forma que el correspondiente al tren de pulsos con la misma
duracin de nivel alto que bajo (figura 1.4 ), sin embargo la densidad de las lneas espectrales
ha aumentado. Si T aumenta hasta hacerse infinito, que equivaldra a una seal con solo un
pulso, entonces la densidad del espectro ira aumentando hasta hacerse continuo. Tambin ha
aumentado la anchura del lbulo principal con lo que podra afirmarse que si para reconstruir
17

el pulso ancho con una buena aproximacin basta un solo armnico, para reconstruir el pulso
estrecho se necesitaran 10 armnicos.

18

PROBLEMAS PROPUESTOS

1.1.- Si la seal peridica . y (t )

t nT0
con T0=10, se filtra mediante un filtro ideal

cuya funcin de transferencia solo deja pasar las frecuencias entre 4.5f0 y 5.5f0 y recordando
que toda seal peridica puede desarrollarse en serie de Fourier, Que se obtiene exactamente
a la salida del filtro?.

1.2.- Se dispone de una seal senoidal s(t)=cos(2103t) y se desea disponer adems de seales
senoidales a frecuencias 2 kHz, 3 kHz y 4 kHz tambin con amplitud unidad. Se pide:
a) Dibujar un diagrama de bloques que permita obtener estas seales.
b) Basndose en el diagrama diseado en el apartado anterior, demostrar analticamente su
funcionamiento especificando concretamente los parmetros que intervengan. Apoye su
anlisis con las expresiones, grficas y explicaciones que considere oportunas.

1.3.- La figura a) muestra la grfica de una seal peridica con periodo de repeticin T0
segundos. La figura b) muestra, en lnea gruesa, la seal generatriz basada en una funcin
senoidal.

Figura b)
p(t)

1
-T0/4

............
-2T0

Figura a)

g(t)

-T0/4
-T0

0 T0/4

T0

.............
2T0
t s

T0/4

T0

Se pide:
a) Escriba la expresin de la funcin generatriz g(t).
b) Desarrolle en serie de Fourier la seal p(t). Calcule el valor sus coeficientes desde n=-6
hasta n=6. Escriba el desarrollo como una suma de funciones sinusoidales usando los valores
calculados en los primeros trminos.

19

20

CAPTULO 2
2 TRANSFORMADA DE FOURIER Y SU APLICACIN
En este captulo se presentan dos herramientas bsicas para el procesado de la seal, la
integral de convolucin y la transformacin de Fourier. Se ha elegido ste orden de
presentacin por la facilidad pedaggica que representa la utilizacin de la integral de
convolucin para fijar las ideas y solucionar ejercicios de ejemplo que comporta la
presentacin de la transformada de Fourier.

2.1 CONVOLUCIN DE DOS SEALES

La convolucin es una operacin que se utiliza ampliamente en el procesado de seal y en


el anlisis de sistemas. El resultado de la convolucin de dos funciones es otra funcin en el
mismo dominio, sea del tiempo o de la frecuencia, que las funciones de las que se deriva.
La convolucin de dos seales x(t) e y(t) es la seal definida de la forma,

z (t ) x(t ) y (t )

x(t t ' ) y(t ' )dt '

Esta operacin tiene las propiedades siguientes:


Asociativa.

x ( y z ) x y z

Conmutativa.

x y yx

Distributiva con respecto a la suma

x ( y z) x y x z

Derivacin.

x y x y x y

Seguidamente, mediante un ejemplo grfico, se mostrar la peculiar forma de cmo puede


ser realizada la operacin de convolucin.

Ejemplo.

Realizar la convolucin de las dos seales x(t) e y(t) dadas en la figura.

21

x(t)

y(t)

2
1
-1

Solucin.

La expresin de la convolucin de las dos seales es,

z (t ) x(t ) y (t )

x(t t ') y (t ')dt '

x(t ') y(t t ')dt '

Por conveniencia didctica se utilizar la ltima de las expresiones de la igualdad. Tambin


se sistematizar el procedimiento que se llevar a cabo en pasos o apartados.
1- Realizar el cambio en x(t) de nombre de la variable, t= t. Este cambio deja invariable a
la funcin x(t).
2- Realizar el cambio en y(t) de nombre de la variable, t= -t. Este cambio produce el
efecto de giro de la funcin y(t) alrededor del eje y, a modo de bisagra. Este efecto se muestra
en la figura a) siguiente.
Despus sumar a esta nueva variable t un retardo arbitrario de valor t. Este efecto se
muestra en la figura b) siguiente.
y(t-t)

y(-t)
1

-2

1
t

a)

t-2

t
b)

Figura 2.1
Notar que la variable independiente de la funcin resultado z(t) es precisamente este
retardo. Dependiendo del valor de t, la funcin y(-t) (que es la invertida con respecto al eje y
de la funcin original) estar situada en algn punto del eje t. Por tanto dando valores a t se
mueve la funcin y(t) a lo largo del eje t.
3- Realizar el producto de las dos funciones para cada valor de t e integrar la funcin
producto resultante. As:

22

Para t 1

x(t)
2

-1

y(t-t)

1
1

t-2

Para este intervalo de t no hay solapamiento entre las dos funciones, su producto es cero y
por tanto z(t)=0.

x(t)
2

Para 1 t 1

-1

y(t-t)

t-2

t
z(t)
4

z (t ) 2 dt ' 2t 1

Para 1 t 3

-1

x(t)
2

-1
y(t-t)
1

t-2

z(t)
4

z (t ) 2 dt ' 23 t
t 2

-1

Para 3 t z(t)=0
As pues la convolucin z(t) de x(t) e y(t) es una funcin tringulo z (t ) 4 t 1
2

23

Este ejemplo esta orientado a dar una idea intuitiva de las acciones que se llevan a cabo
sobre las funciones x(t) e y(t) durante el clculo de esta operacin. Sin embargo este es un
procedimiento poco operativo y normalmente, en lo sucesivo, la obtencin de la funcin
convolucin de dos funciones se realizar por otros medios basados en propiedades de la
transformada de Fourier

2.2 TRANSFORMADA DE FOURIER

Hasta ahora se ha limitado el clculo del espectro a seales peridicas. Se puede extender
la validez de las expresiones utilizadas para obtener el espectro de seales no peridicas, cuya
caracterstica esencial es que su espectro no es discreto. En el apartado anterior se ha
comentado que cuando el periodo T se extiende hasta infinito el espectro se hace continuo y el
valor de sus componentes disminuye, puesto que se est haciendo un promedio en todo el
periodo. Para una seal x(t), si no se realizase este promediado, la ecuacin siguiente
T

T d n F (nf 0 )

f (t )e

j 2nf 0 t

dt

representara simplemente el rea del producto escalar de dos funciones, es decir, la


comparacin de una con otra dentro de un intervalo, siendo la componente dn proporcional al
rea, esto es d n f 0 F (nf 0 ) . Si se hace tender T a infinito lo que se consigue es un paso de
frecuencia cada vez ms pequeo hasta hacerse nulo. As esta expresin se podra escribir
como,
TF f (t ) F ( f )

f (t )e

j 2f t

dt

(2.1)

Esta expresin es la transformada de Fourier de una funcin sin restriccin de intervalo, y


representa el espectro o contenido frecuencial de la funcin temporal f(t). La funcin F(f) ser
en general compleja, pudindose escribir F ( f ) e j arg F ( f ) . As F ( f ) se llama espectro de
amplitud y argF ( f ) se llama espectro de fase.
Por otra parte la expresin f (t )

e j 2nf 0t que es el desarrollo en serie de Fourier de la

funcin f(t) se podra escribir como,


24

f (t )

1
j 2nf 0 t
j 2nf 0 t
F
(
nf
)
e

f
F
(
nf
)
e

F (nf 0 )e j 2nf 0t f 0

0
0
0
T n
n
n

Razonando del mismo modo que en el caso anterior, si T aumenta el paso de frecuencia f0
se hace pequeo y en el lmite el sumatorio se convierte en integral,
TF 1 F ( f ) f (t )

F ( f )e

j 2ft

df

(2.2)

Es la transformada inversa de Fourier.


La condicin suficiente para que una funcin f(t) tenga transformada de Fourier es que la

integral de su mdulo entre los lmites mencionados est acotada. Esto es

f (t ) dt . Pero

esta condicin no es necesaria puesto que funciones como cos(2ft) o u(t) no cumplen esta
condicin y sin embargo s tienen transformada de Fourier.
2

En general se dice que una seal x(t), cuya potencia instantnea es x(t ) , tiene
transformada de Fourier cuando esta seal es de energa finita. Es decir,

x(t )

dt

T 2

1
2
Para el caso de seales de potencia media finita, Pm lim
x(t ) dt , puede existir
T T
T 2
transformada de Fourier (como para seales peridicas) o no.

2.2.1 ALGUNAS CONVERSIONES Y VERSIONES ALTERNATIVAS


La transformada de Fourier (TF) de una seal x(t) tambin puede escribirse como,

X(f )

x(t )e

j 2ft

dt

X(f )

x(t )cos(2ft ) j sen(2ft )dt

x(t ) cos(2ft )dt j x(t ) sen(2ft )dt

Si la seal x(t) es par, se hace cero la integral de la parte imaginaria del segundo miembro
y si la seal es impar es la integral de la parte real la que se anula. Por tanto para una seal
x(t) par su TF es,

X c ( f ) 2 x(t ) cos(2ft )dt

(2.3)

25

Para una seal x(t) impar su TF, con una fase adicional de -/2 para frecuencias positivas y
+/2 para las negativas es,

X s ( f ) 2 j x (t )sen(2 ft )dt

(2.4)

La ecuacin (2.3) es vlida para funciones pares y la ecuacin (2.4) es vlida para
funciones impares. Sus correspondientes transformadas inversas son,

x(t ) 2 X c ( f ) cos(2ft )df


0

y x(t ) 2 X s ( f ) sen(2ft )df

vlidas para t>0

Lo expuesto anteriormente permite afirmar que si,


x(t) es

X(f) es

real y par

real y par

maginaria y par

imaginaria y par

real e impar

imaginaria e impar

imaginaria e impar

real e impar

Si las funciones son asimtricas ( ni pares, ni impares) puede decirse que,


x(t) es

X(f) es

real y asimtrica

compleja y asimtrica

imaginaria y asimtrica

compleja y asimtrica

compleja y asimtrica

real y asimtrica

compleja y asimtrica

imaginaria y asimtrica

Ejercicio propuesto.

Demostrar que TF[x(-t)]=X(-f). Tambin que TF[x*(t)]=X*(-f).

Ejercicio.

Determinar las partes par e impar de una seal x(t).


Solucin:
Si x(t ) x p (t ) xi (t ) entonces x(t ) x p (t ) xi (t ) y por tanto sumando y restando ambas
expresiones se obtiene: x p (t )

x(t ) x(t )
x(t ) x(t )
y xi (t )
siendo, respectivamente,
2
2

la parte par e impar de la seal x(t).


26

2.3 TRANSFORMADAS DE FOURIER DE ALGUNAS FUNCIONES DE INTERS

En procesado de la seal suele ser necesario conocer la expresin de las funciones tanto en
tiempo como en frecuencia que brinda posibilidades de clculo alternativo, as como diversos
procedimientos de operacin entre seales que involucran ambos dominios, tiempo y
frecuencia. Seguidamente se presentan algunas funciones que resultan de gran ayuda en el
aprendizaje de esta disciplina y que permitirn el clculo, a partir de ellas, de otras funciones.

Delta de Dirac.

Tal como se ha definido en cursos anteriores,


0
para t 0

indefinida para t 0

(t )

Cuyas propiedades,

x(t ) (t ) x(0) (t ) o bien x(t ) (t t 0 ) x(t 0 ) (t t 0 )

(t )dt 1 y

x(t ) (t )dt x(0) o bien

x(t ) (t t

)dt x(t 0 )

Con todo ello se puede comprobar que la TF de la Delta de Dirac es,

(t )e

j 2ft

dt 1

Obsrvese que una funcin de duracin nula en el tiempo, como es (t ) tiene un ancho de
banda infinito.
La convolucin con una funcin x(t) es,

x(t ) (t t 0 )

x(t t ' ) (t 't

)dt ' x(t t 0 )

Por tanto para encontrar la convolucin de una (t ) con una funcin x(t) basta sustituir la
variable independiente de la funcin x por el argumento de la delta.

- Funcin pulso rectangular.

1 para t 1 / 2
0 para t 1 / 2

(t )

La TF (t ) sinc f

sen(f )
f
27

Estando la funcin sinc definida de la forma, sinc ( f ) sen(f )


f

para f 0
para f 0

Es el espectro de un pulso unitario de anchura 1 s. Su grfica dibujada entre 8 Hz y +8 Hz


se muestra en la figura 2.1. Obsrvese que el lbulo central tiene anchura doble que los
laterales.
1

0.8

amplitud

0.6

0.4

0.2

-0.2

-0.4
-8

-6

-4

-2

0
frecuencia

Figura 2.1

Ejercicio propuesto.

Demostrar que TF (t ) sinc f

sen(f )
f

- Funcin pulso triangular.

1 t
(t )
0

para t 1
para t 1

La TF (t ) sinc 2 f

sen 2 (f )

f 2

Notar que la duracin del pulso triangular es doble que la del pulso rectangular.
La grfica de su espectro se muestra en la figura siguiente.

28

1
0.9
0.8
0.7

amplitud

0.6
0.5
0.4
0.3
0.2
0.1
0
-8

-6

-4

-2

0
frecuencia

Figura 2.2

- Funcin signo.

1 para t 0
sign(t )
Cuya grfica es,
1 para t 0
Su TF sign(t )

j
f

-1

Su espectro es imaginario a diferencia de las funciones anteriores

que es real. La figura siguiente muestra su espectro de amplitud y de fase.


Modulo

Fase

/2
-/2

Figura 2.3

- Funcin exponencial negativa unilateral.


e t
e u (t )
0
t

para t 0
para t 0

Su TF e t u (t )

1
1 j 2f

Su espectro es una funcin

compleja de la frecuencia.

Ejercicio propuesto.

29

Dibujar el mdulo y la fase de la funcin

1
1 j 2f

- Funcin exponencial negativa bilateral.


e

e t
t
e

para t 0

2
Su TF e t
2
para t 0
1 2 f

Su espectro es una funcin real de la

frecuencia.

- Funcin gaussiana.

e t

Su TF e t e f

Es una autofuncin de la transformada de Fourier.

- Funcin inversa de la raz cuadrada.

1
1
Su TF
f
t

Es una autofuncin de la transformada de Fourier.

El conjunto de funciones que se ha propuesto y el uso de las propiedades de la TF que se


expone a continuacin, permitir obtener un nmero suficiente de pares de transformadas (la
funcin en el tiempo y su correspondiente transformada de Fourier) para abordar los
conceptos e ideas objeto de esta disciplina.

2.4 PROPIEDADES DE LA TRANSFORMADA DE FOURIER

a) Dualidad.
TF X (t ) x( f )

Quiere decir que conocida la expresin de una funcin en el dominio de la frecuencia X(f)
cuya transformada inversa es x(t), si esta expresin X se encontrara en el dominio del tiempo
su transformada de Fourier sera la expresin que tena en el dominio del tiempo pero
cambiando su variable independiente a la frecuencia con el signo cambiado. Seguidamente se
muestra un ejemplo.

Ejemplo.

30

Encontrar la TF de sinc(t), sinc2(t) y 1/t.


Solucin.
TF

TF

Se sabe que (t ) sinc( f ) entonces por la propiedad de dualidad sinc (t ) ( f ) . Pero


adems la funcin pulso es par, as ( f ) ( f )
TF

TF

Se sabe que (t ) sinc 2 ( f ) entonces por la propiedad de dualidad sinc 2 (t ) ( f ) .


Pero adems la funcin pulso triangular tambin es par, as ( f ) ( f )
TF

Se sabe que sign(t )

j TF
j
entonces por la propiedad de dualidad
jsign( f ) . Pero
j t
f

como la funcin signo es impar jsign( f ) jsign( f )

b) Linealidad y superposicin.
TF ax(t ) by (t ) aX ( f ) bY ( f )

Ejemplo.

Encontrar la transformada de Fourier de la seal u(t).


Solucin.

1 para t 0
1 sign(t )
La funcin u (t )
se puede escribir tambin u (t )
2
2
0 para t 0
j
1 sign(t ) ( f )
As, TF

2
2
2f
2

c) Traslacin en el tiempo.
TF x(t t0 ) X ( f )e j 2 ft0

d) Convolucin.
TF x(t ) y (t ) X ( f )Y ( f ) y tambin TF 1 X ( f )Y ( f ) x(t ) y (t )

Lo que permite una alternativa para el clculo de la convolucin de dos funciones a partir
de la transformacin inversa del producto de sus transformadas.

31

Ejercicio.

Como ejercicio demostrar que la TF de la convolucin de x(t) e y(t) es igual al producto de


sus transformadas.
Solucin.

j 2 ft
TF x(t ) y (t ) x(t t ') y (t ')dt ' e
dt y (t ')dt ' x(t t ')e j 2 ft dt

X ( f ) y (t ')e j 2 ft ' dt ' X ( f )Y ( f )

Ejercicio.

Como ejercicio demostrar que la convolucin de (t) y la funcin x(t) es igual a la funcin
pero cambiando la variable de la funcin por el argumento de la delta.
Solucin.

(t ) x(t )

(t ') x(t t ')dt ' (t ') x(t 0)dt ' x(t ) (t ')dt ' x(t )

(t t0 ) x (t )

o bien

(t ' t ) x(t t ')dt ' (t ' t ) x(t t )dt ' x(t t ) (t ' t )dt ' x(t t )
0

Ejercicio.

Si x(t) es la seal de entrada a un sistema con respuesta impulsional h(t)= sinc(t) y si

X(f)=0 para |f|1/2, es decir x(t) es una seal cuyo contenido frecuencial es menor que medio
Hz, determinar cual es la seal de salida del sistema.
Solucin.
La seal de salida y(t) ser la convolucin de la seal de entrada con la respuesta
impulsional. Esto es, y (t ) x(t ) h(t ) . Tambin se sabe que Y ( f ) X ( f ) H ( f ) . As,

Y ( f ) X ( f ) ( f ) X ( f ) y la transformada inversa de Fourier de Y(f) es x(t).


De modo que la seal de entrada pasa por el sistema sin alteracin.

Ejercicio propuesto.

Demostrar que

1 1
(t )
t t

32

e) Derivacin.


TF x(t ) j 2fX ( f )

Demostracin:

Si la transformada de Fourier de x(t) es X(f) entonces x(t )

X ( f )e

j 2ft

df asi,

dx(t ) d
de j 2ft
j 2ft

X
(
f
)
e
df

X
(
f
)
df j 2fX ( f )e j 2ft df

dt
dt
dt

Por tanto ya est demostrado puesto que la ltima expresin muestra que la transformada
inversa de j2fX(f) es la derivada de x(t).
f) Producto.
TF x(t ) y (t ) X ( f ) Y ( f )

Demostrar esta propiedad como ejercicio propuesto. (se sugiere proceder como en la

propiedad de la TF de la convolucin).

Ejercicio.

Encontrar la TF de la funcin rampa r(t)=tu(t).


Solucin.
Primeramente se debe calcular la TF de la funcin t. Se sabe que
por dualidad

1 d ( f )
j 2t TF 1 d ( f )

j 2
j 2 d ( f )
j 2 df

d (t ) TF
j 2 f , entonces
dt

puesto que la delta es una funcin par.

As,

TF tu (t )

1 d ( f ) ( f )
1 d ( f ) 1
j
j d ( f )

2f 4 df
j 2 df
f
4 df
2

Puesto que,
d1 f
d ( f ) 1
1
( f )
2 entonces,
df
f
df
f

TF tu (t )

j d ( f )
1 d ( f ) 1
1

2
2
f
4 df
4 df
4 f

j d ( f )
4 df

33

Ejercicio propuesto.

Demostrar que TF t TF t sign(t )

1
2 2 f

g) Cambio de escala.

TF x(at )

1 f
X , siendo a una constante real.
a a

Si a es menor que 1 la funcin en el tiempo se expande y por tanto en frecuencia se


contrae. Si a es mayor que 1 sucede lo contrario. Una vez ms se pone de manifiesto el
compromiso duracin en el tiempo y ancho de banda.

Ejercicio.

t t0
t t0
Encontrar la transformada de Fourier de A
y de A



Solucin.
t t0
TF A

j 2ft 0
Asinc (f )e

t t0
TF A

j 2ft 0
Asinc 2 (f )e

h) Integracin.

t
X ( f ) X (0) ( f )
TF x(t ' )dt '

f
2
2

Demostracin.

La integral puede escribirse tambin como

x(t ' )dt '

t
( f )
j

TF x(t ' )dt ' TF x(t ) u (t ) X ( f )


2f
2

x(t ' )u(t t ' )dt ' x(t ) u (t ) . As,

X (0) ( f ) X ( f )

2
j 2f

i) Traslacin en frecuencia.

TF x(t )e j 2 f0t X ( f f 0 )
34

Ejemplos de aplicacin.

TF e j 2f 0t ( f f 0 )
e j 2 f0t e j 2 f0t 1
TF cos(2 f 0t TF
( f f 0 ) ( f f 0 )
2

2
e j 2 f0t e j 2 f0t 1
TF sen(2 f 0t TF
( f f 0 ) ( f f 0 )
2j

2j

Con ayuda de esta propiedad se obtiene el mtodo que permite calcular la transformada de
Fourier de una seal peridica. Puesto que toda seal peridica z(t) puede ser desarrollada en
serie de Fourier z (t )

de
n

j 2 f 0 t

, entonces la transformada de Fourier de una suma de

funciones exponenciales es tambin una suma, por la propiedad de linealidad. Por la


propiedad de traslacin en frecuencia esta suma estar formada de deltas de Dirac.

Z( f )

d ( f nf ) .

Llegados a este punto se puede demostrar que el desarrollo en serie de una seal peridica
z(t)

z (t )

cuya

funcin

x(t mT0 ) f 0

generatriz

X (nf )e
0

j 2 nf 0t

es

x(t)

puede

escribirse

como,

. Este resultado permite calcular fcilmente estos

desarrollos conociendo la transformada de Fourier de la funcin generatriz, porque expresa


los coeficientes dn como la TF de la funcin generatriz pero sustituyendo la variable f por nf0.

Ejercicio.

Demostrar que una seal peridica z(t) cuya funcin generatriz es x(t) que se repite cada
T0=1/f0 segundos, puede escribirse como,

z (t )

x(t mT0 ) f 0

X (nf )e

j 2 nf 0t

(2.5)

Solucin.
La funcin generatriz que se repite puede escribirse tambin como,

x(t mT ) x(t ) (t mT )

(2.6)

35

Por otra parte el sumatorio de deltas de Dirac es una funcin peridica y como tal puede
desarrollarse en serie de Fourier. As,

(t mT0 )

d n e j 2 nf0t

1
con d n
T0

T0

T
0

(t )e j 2 nf t dt f 0
0

La ecuacin (2.6) se desarrolla como sigue,

x(t mT ) x(t ) (t mT ) x(t ) f e


0

j 2 nf 0t

y su transformada de Fourier es,

TF

j 2 nf0t
x
(
t
mT
)
X
(
f
)
TF
f
e
f
X
(
f
)

( f nf0 )
0
0
0

m
n
n

con lo que

Z ( f ) TF

x(t mT0 ) f 0

X (nf ) ( f nf )
0

y finalmente haciendo su transformada inversa de Fourier,

z (t )

x(t mT0 ) f 0

X (nf )e

j 2 nf 0t

Esta expresin es conocida como formula sumatoria de Poisson.

La demostracin de la expresin dual Z ( f ) f 0

X ( f mf 0 )

x(nT )e

j 2 nfT0

es

decir una seal peridica en frecuencia se obtiene de forma similar a la realizada en el


ejercicio anterior.
Seguidamente se muestra un ejercicio de aplicacin en el que se podr apreciar la
dificultad de clculo que se presentara en caso de no utilizar el concepto de la formula
sumatoria de Poisson.

Ejercicio.

Desarrollar en serie de Fourier la seal de la grfica que se muestra a continuacin.

36

f(t)
1

-T
2

T
2

Cuya expresin es:

f (t ) abs(sen(2f 0t ))

con T=1/f0

Solucin.
La expresin de la seal peridica anterior, tambin puede escribirse como:
f (t )

cos( 2f 0 (t

t T nT
T
4
nT ))

T
4
2

T
cos( 2f (t T nT )) t 4 nT
0

T
4
2

La funcin generatriz de la seal peridica es:


t T
T
4
f p ( t ) cos( 2 f 0 ( t ))
T
4
2

cos( 2 f ( t T )) t 4
0

T
4
2

Una til forma de encontrar el valor de dn para el desarrollo en serie de Fourier, es


precisamente aplicar la transformada de Fourier, particularizada para f=nf0 ,a la funcin
generatriz de la seal peridica. As:
F( f ) 1 1
1
fT j 2 f T4
T
( f f0 ) ( f f0 ) sin c( ) e

T
T 2
2
2
2

1 1
1
fT j 2 f T4
T
(
)
(
)
sin
(
) e
f
f
f
f
c

0
0

T 2
2
2
2
1
T
T j 2 f T4
T
T j 2 f T4

sin c ( f f0 ) sin c ( f f0 ) e
sin c ( f f0 ) sin c ( f f0 ) e

4
2
2
2
2

T
j 2 f
j 2 f T4
4
2
T
T e
e

sin c ( f f 0 ) sin c ( f f 0 )
4
2
2
2

dn

F (nf 0 ) 1
n 1
n 1
n
) sin c(
) cos( )
sin c(
T
2
2
2
2

Con n entero desde - a .


37

Si se da valores a n desde -8 a 8 para visualizar la tendencia de los trminos tenemos:


dn [...

2
2
2
2
2
2
2
2
2
,0,
,0,
,0, ,0, ,0, ,0,
,0,
,0,
...]
63 35
15 3 3
15 35
63

Por lo tanto el desarrollo en serie se escribe como:


f (t) ....

4
4
4
cos(2 2 f 0 t )
cos(2 4 f 0 t )
cos(2 6 f 0 t )...
15
35
3
Y de una forma mas compacta:

f (t )

f (t )

2 j 2 4 f0t 2 j 2 2 f0t 2 2 j 2 2 f0t 2 j 2 4 f0t


e
e
e

e
....
15
3
15
3

(1) n
n 1

4
((2n) 1)
2

cos(2 2nf 0 t ) con n 1,2,3...

Ejercicio.

Se dispone de un tren de pulsos, como en la figura siguiente, de amplitud unitaria y de


periodo T =2 segundos, con una anchura del pulso bsico de T/2

f(t)
1

-T/2 -T/4

T/4 T/2

Este tren de pulsos ser filtrado paso-bajo ideal con frecuencia de corte 3Hz, es decir, la

f
funcin de transferencia H(f) del filtro es H ( f ) . Cul es la seal de salida del filtro?
6
Solucin.
Recordemos que la expresin general del desarrollo de una funcin f(t) en serie de Fourier
es:

f (t )

d ne

j 2nf 0t

con

1
dn
T

f (t )e

j 2nf 0t

dt

38

Teniendo en cuenta que f0=1/T, dn se desarrolla como sigue:


1
dn
T

1
j 2nf t
T e 0 dt T
4

dn

jn
jn
e j 2nf0t 4
1 e 2 e 2

j 2nf 0
j 2nf 0 T 4 T

sen n

sen n
21
2 1 sinc n
2 n
n
2
2
2

(Tambin se podra haber hecho aplicando la formula sumatoria de Poisson).


La grfica de dn , como se recordar de ejercicios anteriores, es:

0.5

0.4

amplitud

0.3

0.2

0.1

-0.1

-0.2
-20

-15

-10

-5

0
n

10

15

20

Figura 2.4
Si se ha de filtrar la seal peridica f (t )

d e

j 2nf 0t

, es preciso encontrar su

transformada de Fourier F(f), con objeto de obtener la salida del sistema Y(f)=F(f)H(f). As
pues:

TF f (t ) TF d n e j 2 nf0t
n

TF d n e j 2 nf0t d n ( f nf 0 )
n
n

F( f )

2 sin c 2 ( f nf )

As pues:
F( f ) ...

1
1
1
1
1
( f 3 f0 ) ( f f0 ) ( f ) ( f f0 ) ( f 3 f0 ).....

3
2
3

Por tanto al multiplicar F(f) por H(f), quedaran nicamente los trminos:
39

F( f )

1
1
1
1
( f 5 f0 ) ( f 3 f0 ) ( f f0 ) ( f )

5
3
2

( f f0 )

1
1
( f 3 f0 ) ( f 5 f0 )
3
5

Y haciendo la transformada inversa de Fourier de F(f)

f (t )

1 2
2
2
cos 2f 0t
cos 2 3 f 0t
cos 2 5 f 0t
2
3
5

Si se sustituye f0 =0.5 Hz., entonces:


1 2
2
2
cos 2 0.5t
cos 2 1.5t
cos 2 2.5t
2
3
5
En la grfica de la figura siguiente se puede observar la forma del tren de pulsos despus

f (t )

de filtrar, es decir la suma de las tres seales coseno anteriores.


1 .2
1

Amplitud

0 .8
0 .6
0 .4
0 .2
0
-0 .2

10

Ejercicio.

Desarrollar en serie y encontrar el espectro de la funcin peridica x(t) dada en la grfica


siguiente.

x(t)
a

..........
Solucin.

x(t )

d n e j 2nf0t con

..........
T0

dn

1
T0

x(t )e

j 2nf 0t

dt , sin embargo la integral de los

coeficientes entre -T0/2 y T0/2 resulta un tanto molesto de clculo.


Con objeto de simplificarlo se puede desplazar T0/2 toda la funcin x(t), con lo que la
nueva funcin y(t) queda como en la figura siguiente.
40

y(t)
a

..........
-T0/2

..........

T0/2

T
As, la funcin original x(t) se puede escribir como x(t ) y t 0 . Ahora se realizarn
2

los clculos sobre la funcin y(t) y al finalizar se harn los cambios oportunos.
T

a
y (t )dt 2

T0
T

1
El coeficiente en el origen es d 0
T0

(t

T0
a
)dt .
2
2

Para el resto de coeficientes dn se obtiene,


1
dn
T0

y(t )e

j 2nf 0t

a
dt 2
T0

(t

T0 j 2nf 0t
)e
dt
2

T0 j 2 nf0t 2

(
)e
t

a
a
2
2
2

T0
j 2 nf 0
T0

T 2

j n
j n
e j 2 nf0t 2

a T0e j n
a e e
2
2

2
2
T0 j 2 nf 0 T0 j 2 nf 0
j 2 nf 0 T
2
T

j n
j n

sen( n ) jae j n
ae j n a e e
jae j n
a

j 2 n T02 j 2 nf 0 2
T0 j 2 nf 02 n
2 n
2 n

Puesto que

sen(n)
siempre ser cero para n diferente de cero, que es el caso que nos
n

ocupa.
Bien, ya se han calculado los coeficientes. Ahora nos centraremos en deshacer el efecto del
desplazamiento introducido.
T

j 2nf 0 ( t 0 )
a ja
T
2
x(t ) y t 0 d 0 d n e

2

2 2
n 0

x(t )

a ja

2 2

1 j 2nf 0t j 2n a ja
e
e

2 2
n 0 n

1 jn j 2nf 0t jn
e e
e
n 0 n

1 j 2nf 0t
e
Puesto que e j 2n siempre vale
n 0 n

uno. An se puede arreglar un poco ms.

x(t )

a ja
1 j 2 3 f 0 t 1 j 2 2 f 0 t j 2 f 0 t
1
1

e
e
e j 2 f0t e j 2 2 f0t e j 2 3 f0t .........
....... e
2 2
3
2
2
3

x(t )

a a e j 2f 0t e j 2f 0t e j 2 2 f 0t e j 2 2 f 0t e j 2 3 f 0t e j 2 3 f 0t

........
2
2j
2 j2
2 j3

41

x(t )

a a
1
1

sen(2f 0 t ) sen(2 2 f 0 t ) sen(2 3 f 0 t )........


2
2
3

Finalmente,

x(t )

a
2 1

1 sen(2nf 0 t )
2 n 1 n

Como ya se ha visto el espectro de una seal peridica est formado por una suma de
deltas de Dirac en frecuencia situadas en cada uno de los armnicos de la seal peridica. Es
un espectro discreto. Estas deltas tienen rea igual al valor de su correspondiente dn. Dado que
las deltas tienen amplitud infinita, por convenio se representa el espectro con alturas que
representan estas reas.
En el caso que nos ocupa el espectro es complejo, por tanto se representar mediante su
mdulo y su argumento.

x(t ) d 0

n 0

dn e

j 2 nf 0 ( t

T0
)
2

a
a j 2 n j 2 j 2 nf0t a
a j (2 n 2 ) j 2 nf0t
e
e e
e
e


2 n 0 2 n
2 n 0 2 n

Por tanto el espectro ser,

a
a j (2 n 2 )
X(f ) (f )
e
( f nf 0 )
2
n 0 2 n

Su mdulo,

X(f )

a
a
(f )
( f nf 0 )
2
n 0 2 n

Su fase,

2n 2
arg X ( f )

2n
2

para n positivo
para n negativo

A continuacin se realizan ejemplos que ponen de manifiesto la relacin existente entre


rapidez o pendiente de una seal y su ancho de banda.

Ejercicio

Estudio de ancho de banda y duracin vlido para seales paso bajo.

42

Definicin de una cota superior para el valor absoluto de la derivada de una funcin real

f(t), con un espectro F(f) paso bajo con banda limitada a B.


Solucin:

df (t )
d
F ( f )e j 2ft df

dt
dt

d
F ( f )e j 2ft df

dt
B

B
df (t )
d

F (0) e j 2ft df
dt
dt
B

sen(2 Bt )
d
F (0)2 B

dt
2 Bt

Los ceros de la funcin sinc, estn en t=n/2B, en estos puntos la pendiente es mxima ( y la
mxima de todas se encuentra en el primer cero t=1/2B).
As que, derivando y sustituyendo t=1/2B, se obtendr una cota superior para la derivada
en funcin de F(0) y B.
La derivada de la ltima expresin anterior es:

2 Bt cos(2 Bt ) sen(2 Bt )
F (0)

t 2

Y para t=1/2B obtenemos:


4 F (0) B 2
As que :
.

df (t )
4 F (0) B 2
dt

La pendiente de una funcin siempre ser menor que 4 veces el rea de la funcin y
multiplicado por el cuadrado de su ancho de banda.

Ejercicio

Definicin de una cota inferior para el ancho de banda B (definido con la frecuencia de
corte a la que se cumple

F (0)
2

) de una seal f(t), real, con un espectro F(f) pasobajo, en

funcin del valor absoluto de la derivada mxima de f(t) (

df (t )
dt

max

) y de F(0).

Solucin:

43


df (t )
d

F ( f )e j 2ft df
dt
dt

d B
F ( f )e j 2ft df
dt
B

B
df (t )
d F (0)

e j 2ft df

dt
dt 2 B

d F (0)
sen(2 Bt )

dt 2 2 B 2 Bt

Recuerdese que:

F(f)

F (0)

f (t )dt

F(0)/2

y que

Siguiendo:

df (t ) F (0) 2 Bt cos(2 Bt ) sen(2 Bt )

dt
t 2
2

Las derivadas ms grandes se producen en los ceros del sinc (t=n/2B; n=1,2,3....)
La mxima pendiente se produce en el primer cero.
Por lo tanto, si para cualquier t, el primer miembro de la ecuacin anterior es mayor o igual
que el segundo miembro, entonces para un tiempo t concreto tambin se cumplir. Si este
tiempo es t=1/2B, se puede escribir:
df (t )
dt t 1

2B

F ( 0) 2
4B
2

(2.7)

2
df (t )
B

dt max 4 F (0)

(2.8)

Adviertase, que en la ecuacin 2.7 no queremos decir que la derivada mxima de f(t) se
produzca en t=1/2B, si no nicamente que la derivada en este punto es mayor que una cierta
cantidad constante, por tanto la derivada mxima forzosamente ha de ser tambin mayor y en
consecuencia cumplirse la ecuacin (2.8).

max pendiente de f(t)

2
4 F (0)

Puede comprobarse que esta expresin arroja resultados parecidos a DURACIN EN


TIEMPO * DURACIN EN FRECUENCIA = 1

44

PROBLEMAS PROPUESTOS

2.1.- Demostrar que Sign(t) es igual a -j/f


2.2.-Encontrar la transformada de Fourier de tu(t)
2.3.- Demostrar que la transformada de Fourier de x(at) es igual a

1 f
X
a a

2.4.-Encontrar la transformada de Fourier de una seal peridica.


2.5.- Encontrar la expresin de Sinc(t ) Sinc(t )
2.6.-.Encontrar la expresin de (t ) (t )
2.7.- Encontrar la expresin de

1 1

t t

2.8.- Encontrar la transformada de Fourier de Sinc 2 (t )


2.9.- Calcular x(t ) (t t 0 )
f
2.10.- Un sistema tiene una funcin de transferencia dada por H ( f ) . Si la entrada es
f0
t
, Cual es la expresin, en el tiempo, de la salida del sistema si >>1/f0?

2.11.- Calcular la integral

3Sinc

(t )dt

2.12.- Demostrar que la respuesta y(t) a la funcin impulso (t), es igual a h(t)
(t)
>

h(t)

y(t)
>

45

h(t )dt

2.13.- Demostrar que la respuesta y(t) a la funcin escaln u(t) es igual a

u(t)
>

y(t)
>

h(t)

2.14.- La respuesta impulsional de un sistema es h(t ) 20u (t )e 10t (t ) .


Escriba la expresin de la salida de el sistema si a la entrada se introduce
1)

t nT0
) con T0=0.1 segundos.
p (t ) (
T0 2
n
2) Obtener la respuesta de este sistema si a la entrada se introduce 3Sinc(t ) . Para ello haga
una aproximacin de la respuesta impulsional de este sistema, vlida para frecuencias f
menores que 1 Hz.

t
t
2.15.- El diagrama de bloques siguiente tiene h(t ) a , y h1(t ) y seal de
a
a
entrada x(t)
x(t)
1
x(t)
y(t)
h(t)
d/dt
-1/2

1
h1(t)

t s

-1

1/2

Suponiendo que a>>2, encontrar la expresin de y(t) y dibujar su grfica.

2.16.- El diagrama de bloques de la figura 1 procesa la seal x(t) con ancho de banda B Hz
para dar y(t). Se sabe que h1(t ) t , h 2(t ) t T0 y f0=1/T0 es 2 veces B. Esto
equivale a un sistema como en la figura 2. Se pide:
a) Determine las expresiones H(f) y h(t).
b) Dibujar el espectro de amplitud y de fase de H(f) para un margen de frecuencias entre B y
B Hz.
x(t)

h1(t)

y(t)

x(t)

h(t)

y(t)

h2(t)

Figura 1

Figura 2
46

2.17.- El diagrama de bloques de la figura 1 procesa la seal x(t) con ancho de banda B Hz
para dar y(t). Se sabe que, h1(t ) t T0 y f0=1/T0 es 2 veces B. Esto equivale a un
sistema como en la figura 2.

x(t)

y(t)
x(t)

h(t)

y(t)

h1(t)

Figura 1

Figura 2

Se pide:
a) Determine las expresiones H(f) y h(t). Dibujar el espectro de amplitud y de fase de H(f)
para un margen de frecuencias entre B y B Hz.
b) Si X ( f )

10
f
1 j 2
B 10

calcule la expresin de y(t) haciendo la aproximacin que

considere oportuna.

t T0
t T0

2
2.18.- Si en en el sistema de la figura a) x(t ) 2
y h(t )
T
T
2
0

x(t)
Figura a)

h (t )

y(t)

(t)+(t-T0)+ (ta) Representar grficamente x(t)


b) Calcular la seal de salida y(t) y representarla grficamente.

2.19.- La figura a) muestra la grfica de una seal x(t) peridica con periodo de repeticin T0
segundos.

47

x(t)

g(t)

.
-

Figura a)

T0 +T0/4
T0/4
T0/4+ T0/2

Se pide:
a) Escriba una expresin de la funcin generatriz g(t).
b) Desarrolle en serie de Fourier la seal x(t). Calcule el valor de los coeficientes desde n=-5
hasta n=5. Escrbala como una suma de funciones sinusoidales.
c) Obtenga la transformada de Fourier X(f) de la funcin x(t). Dibuje esquemticamente el
espectro de amplitud de X(f).
d) Con un filtro ideal, tal como se representa en la figura siguiente, se filtra la seal x(t) para
obtener nicamente su primer armnico (seal y(t)) a la frecuencia f0.
x(t)

y(t)

h(t)

1) Escriba la funcin de transferencia H(f) y la respuesta impulsional h(t) de este filtro.


2) Escriba la expresin de la seal y(t).

2.20.- La figura a) muestra la grfica de una seal peridica con periodo de repeticin
T0=0.25 segundos. La figura b) muestra, en lnea gruesa, la seal generatriz generada en base
a una funcin senoidal, siendo T0=10 segundos.
g(t)

p(t)

Figura b)

............
-

.............
-T0

T0

Figura a)

-/2 /2

T0

Se pide:
a) Escriba una expresin de la funcin generatriz g(t).
b) Obtenga la transformada de Fourier G(f) de la funcin generatriz g(t).
48

c) Desarrolle en serie de Fourier la seal p(t). Calcule el valor sus coeficientes desde n=0
hasta n=3. Escriba el desarrollo como una suma de funciones sinusoidales, al menos con los 4
primeros trminos (utilice los coeficientes calculados).
d) Escriba la expresin de P(f). Dibuje esquemticamente el espectro de amplitud de P(f).
Diga tambin que frecuencias armnicas no contendr este espectro.
e) Dibuje un diagrama de bloques que permita obtener una funcin senoidal a 4 Hz y otra a 8
Hz, ambas con la misma amplitud. Si usa filtros y amplificadores especifique sus respuestas
impulsionales y sus funciones de transferencia.

49

CAPTULO 3
3 DENSIDAD ESPECTRAL Y CORRELACIN

3.1 ENERGA DE UNA SEAL

La energa normalizada de una seal es un parmetro til en su procesado. Se define


energa normalizada de una seal x(t) como la energa disipada por una resistencia de un ohm
cuando se le aplica un voltaje x(t) o bien la misma corriente por la resistencia de un ohm. Su
expresin es,

Ex

x(t )

dt

(3.1)

Esta expresin solo tiene significado cuando su valor es finito. Como ya se ha visto en el
captulo uno estas seales se denominan de energa finita. En algunas seales, como por
ejemplo las seales peridicas, esta expresin sera infinita, entonces se considera el promedio
en el tiempo de la energa de la seal. Este promedio resulta ser la potencia media de la seal
o lo que es lo mismo su valor cuadrtico medio. Estas seales se denominan de potencia
media finita. Su expresin es,
T /2

1
Px x(t ) lim
x(t ) 2 dt
T T
T / 2
2

(3.2)

3.2 DENSIDAD ESPECTRAL DE ENERGA

La transformada de Fourier conserva el producto escalar y por tanto la ortogonalidad de las


funciones, entonces si las funciones x(t) e y(t) tienen transformada de Fourier se puede
establecer que:

x(t ) y (t )dt X ( f )Y ( f )df

Esta igualdad es fcilmente demostrable partiendo de,

x(t )

X ( f )e

j 2ft

df y de y (t ) Y ( )e j 2 t d Y ( )e j 2 t d

50

As,

x(t ) y

(t )dt

X ( f )Y

( )e

j 2 ( f ) t

dfddt

X ( f )df Y

X ( f )Y

( ) ( f )d

X ( f )Y

( )dfd e j 2 ( f ) t dt

( f )df

Resulta de especial inters cuando las dos seales son iguales. Es conocido como teorema
de Parseval. En este caso para x(t),

x(t ) dt X ( f ) df
2

(3.3)

La transformacin de Fourier conserva las energas. La energa de la seal puede ser


calculada donde resulte mas adecuado o ms fcil, en el dominio del tiempo o bien en el
dominio de la frecuencia. Tambin tiene aplicaciones concretas distintas; puede calcularse la
energa en un intervalo de tiempo o bien en un intervalo de frecuencia. En este ltimo caso
sealar que las frecuencias matemticamente negativas contribuyen tambin a la energa de la
seal aunque fsicamente solo tengan significado las positivas. La integral se expresara de la
forma siguiente,
Ex

f1

f2

f2

X ( f ) df X ( f ) df
2

f1

Por otra parte la funcin S xx ( f ) X ( f )

se llama densidad espectral de energa, porque

muestra como se distribuye la energa de la seal en el eje de frecuencias. Se puede calcular


como el mdulo al cuadrado de la transformada de Fourier.
Otra situacin interesante se presenta cuando es necesario calcular la densidad espectral de
la seal y(t) que es la salida de un sistema con respuesta impulsional h(t) al ser excitado con
una seal de entrada x(t). La T.F de la respuesta es Y ( f ) H ( f ) X ( f ) , por tanto su energa
ser,

Y ( f )Y

( f )df

H ( f ) X ( f )H

( f ) X ( f )df

H( f )

X ( f ) df

y obviamente su densidad espectral ser,


2

S yy ( f ) Y ( f ) H ( f ) X ( f )

(3.4)

51

3.3 DENSIDAD ESPECTRAL DE POTENCIA

Para determinar alguna expresin de la densidad espectral de potencia de una seal x(t) de
potencia media finita, es decir, de energa infinita, se podra truncar esta seal en un intervalo
de tiempo, por ejemplo entre -T/2 y T/2. Entonces esta seal truncada ya es de energa finita y
es aplicable lo que se expone en el anterior apartado. Llamemos xT(t) a la seal x(t) truncada
entre -T/2 y T/2. As,

2
xT (t ) dt X T ( f ) df
2

Pero como est truncada en este intervalo es posible escribir la igualdad siguiente,
T

xT (t ) dt
2

2
xT (t ) dt X T ( f ) df

Haciendo uso de la expresin 3.3 se obtiene,


T /2

1
1
1
2
2
Px lim
xT (t ) 2 dt lim X T ( f ) df lim X T ( f ) df

T T
T T
T T
T / 2

Por tanto puede afirmarse que la densidad espectral de potencia de x(t) es,
G x lim

1
2
XT ( f )
T

Significa que la densidad espectral de potencia de una seal x(t) se puede obtener
calculando la densidad espectral de energa de la seal truncada, y como esta expresin
depender del intervalo de truncado se debe llevar ste al limite.
De forma semejante la densidad espectral de potencia de la salida de un sistema con
respuesta impulsional h(t) cuando se excita con una seal x(t) de potencia media finita se
puede expresar como,
1
1
2
2
2
2
H ( f ) X T ( f ) H ( f ) lim X T ( f ) H ( f ) G x
T T
T T

G y lim

Ejercicio.

Calcular la densidad espectral de potencia de una seal peridica x(t) con periodo T0=1/f0.
Solucin.

52

Si x(t) es peridica se puede desarrollar en serie de Fourier. Esto es x(t )

espectro X(f) es la suma de la TF de cada uno de sus sumandos. As X ( f )

e j 2nf 0t . Su

d ( f nf

t
Por otra parte se puede truncar x(t) multiplicndola por un pulso de ancho T, . As la
T
t
seal truncada xT(t) ser xT (t ) x(t ) y su espectro X T ( f ) X ( f ) Tsinc (Tf ) , que
T
tambin puede escribirse como,
X T ( f ) Tsinc (Tf )

d ( f nf

) T

sinc( f nf 0 )T

Su densidad espectral de potencia ser entonces,

1 2 *
T d m sinc( f mf 0 )T d n sinc( f nf 0 )T d n2 ( f nf 0 )
T T
m n
n

Gx ( f ) lim

Puesto que cuando T crece la funcin 'sinc' se hace estrecha y adems, al estar multiplicada
por T, se convierte en una funcin delta para T infinito.
As la densidad espectral de potencia de una seal peridica no es ms que el mdulo al
cuadrado de su TF que como ya se ha visto es discreta.

3.4 CORRELACIN DE DOS SEALES DE ENERGA FINITA

Se define la funcin de correlacin de dos seales de energa finita mediante la expresin


siguiente,

Rxy (t )

x( t ) y ( )d

o bien

Rxy (t )

x( ) y ( t )d

(3.5)

Es una funcin del tiempo, no hay cambio de dominio. Es una medida del parecido de una
funcin, que se va desplazando en el tiempo t, con respecto a la otra que permanece fija. Las
ecuaciones (3.5) muestran que en la primera expresin la funcin x(t) se desplaza con
respecto a la funcin y(t) permaneciendo fija esta ltima, mientras que en la segunda
expresin la funcin y(t) se desplaza con respecto a la funcin x(t). Ambas expresiones
proporcionan el mismo resultado. No cabe duda que si en algn instante t existiera un
parecido entre las dos seales el valor de la integral, para este instante, sera elevado
comparado con otros instantes en los que existe menor semejanza.
53

En general no es igual la correlacin de x con y que y con x, su relacin es Rxy (t ) Ryx (t )


Formalmente esta expresin es semejante a la convolucin y en efecto, puede usarse el
concepto de convolucin para su clculo. As,
Rxy (t ) x(t ) y (t )

x(t t ') y (t ')dt '

x(t ) y

( )d

(3.6)

La integral de convolucin se obtiene al hacer la sustitucin t= - lo que no supone ningn


cambio. De otra forma, operando la convolucin en la funcin y* (t ) es an ms sencillo.

Rxy (t ) x(t ) y (t )

x( ) y ( t )d . Igualmente puede mostrarse

Ryx (t ) invirtiendo

la funcin x(t):

Ryx (t ) x(t ) y (t )

x( t ) y ( )d

La TF de la correlacin de dos funciones x(t) e y(t) se denomina densidad espectral


cruzada de energa, S xy ( f ) X ( f )Y ( f ) . Se puede demostrar fcilmente.

y (t )e

Si TF y (t )

j 2ft

dt

(t ' )e

j 2ft '

(dt ' ) y (t ' )e j 2ft ' dt ' Y ( f )

Entonces tomando transformada de Fourier en la ecuacin 3.6 se obtiene,

S xy ( f ) TF R xy (t ) TF x(t ) y (t ) X ( f )Y ( f )

Si la seal x(t) es igual a y(t) entonces la operacin se llama autocorrelacin.

Rxx (t )

x( ) x ( t )d x( t ) x ( )d

La transformada de Fourier de la autocorrelacin es la densidad espectral de energa.

S xx ( f ) TF R xx (t ) TF x(t ) x (t ) X ( f ) X ( f ) X ( f )

Es de notar que el valor de la autocorrelacin en el origen es la energa de la seal. Esto es,


R xx (0)

x( ) x ( )d

X ( f ) X ( f )df X ( f ) df S xx ( f )df E x
2

Se denomina coeficiente de autocorrelacin de la seal de energa finita x(t) a la relacin


c xx

R xx (t )
. Por otra parte dos seales x e y de energa finita son incorreladas si R xy (t ) 0
R xx (0)

y R yx (t ) 0 . Por otra parte el coeficiente de correlacin normalizada se escribe:

54

x(t ) y (t )dt

x(t ) 2 dt

y (t )

dt

3.5 CORRELACIN DE DOS SEALES DE POTENCIA MEDIA FINITA

La expresin es semejante a la definida para seales de energa finita salvo que es


necesario llevar al lmite el intervalo de truncado de las seales del mismo modo que se ha
realizado para densidad espectral de potencia. Se define correlacin cruzada de dos seales
x(t) e y(t) de potencia media finita como,
T

1 2
xy (t ) lim x( ) y ( t )d
T T
T

(3.7)

Tambin de modo semejante a seales de energa finita la igualdad siguiente muestra que
la funcin de auto correlacin en el origen es igual a la potencia media de la seal.

xx (0) G xx ( f )df Px

(3.8)

T de energa

Para verificar que realmente es as, consideremos la seal xT (t ) x(t ) t

finita obtenida a partir del truncado de la seal x(t) de potencia media finita. Su transformada
T

de Fourier es

XT ( f )

x(t )e

j 2ft

dt

y su funcin de auto correlacin ser,

R xT xT (t ) xT (t ) xT (t ) , por tanto su funcin de densidad espectral de energa


2

S xT xT ( f ) TF xT (t ) xT (t ) X T ( f ) .

Su densidad espectral de potencia vendr dada por la expresin siguiente,


X (f)
1
1
GxT xT ( f ) S xT xT ( f ) TF xT (t ) xT (t ) T
T
T
T

Si se hace tender T a infinito entonces,


2

X (f)
1
1
Gxx ( f ) (3.9)
lim GxT xT ( f ) lim S xT xT ( f ) lim TF xT (t ) xT (t ) lim T
T
T T
T T
T
T
Siendo esta ltima la densidad espectral de potencia de la seal x(t)
55

De la ecuacin (3.9) puede deducir que


lim

1
1

TF xT (t ) xT (t ) TF lim ( xT (t ) xT (t )) G xx ( f )
T

T
T

(3.10)

Por tanto la transformada de Fourier de la auto correlacin de una seal de potencia media
finita x(t) es su densidad espectral de potencia media. Esto es conocido como el teorema de
Wiener-Khintchine.
TF xx (t ) G xx ( f )

(3.11)

Este resultado es til cuando se tratan seales aleatorias, que no tiene un expresin
analtica, pero bajo ciertas condiciones disponen de una funcin de auto correlacin como la
expresada permitiendo realizar estudios en trminos de potencia.
Es de notar que G xx ( f ) no puede obtenerse de X ( f )

incluso aunque la seal de

potencia media finita admita espectro.


Se denomina coeficiente de auto correlacin de la seal de potencia media finita x(t) a la
relacin C xx

xx (t )
. Por otra parte dos seales x e y de potencia media finita son
xx (0)

incorreladas si xy (t ) 0 y yx (t ) 0 .

3.5.1 ALGUNAS PROPIEDADES DE LA CORRELACIN Y DE LA DENSIDAD


ESPECTRAL
Las siguientes propiedades son vlidas independiente que las seales sean de energa finita
o de potencia media finita.
Si z (t ) x(t ) y (t ) entonces su auto correlacin es,
R zz (t ) R xx (t ) R xy (t ) R yx (t ) R yy (t ) y por tanto su densidad espectral es,
S zz ( f ) S xx ( f ) S xy ( f ) S yx ( f ) S yy ( f )
Si las seales x(t) e y(t) son incorreladas entonces las anteriores expresiones se simplifican.
R zz (t ) R xx (t ) R yy (t )
y su densidad espectral es,
S zz ( f ) S xx ( f ) S yy ( f )
Esto implica que la energa o potencia de la seal z(t) ser,
E z R zz (0) R xx (0) R yy (0) E x E y
56

As para que las energas puedan ser sumadas es suficiente que las seales componentes
sean incorreladas. No es necesario puesto que bastara que las correspondientes correlaciones
cruzadas en el origen fueran cero. Esto es R xy (0) 0 y R yx (0) 0

Ejercicio.

Una seal con densidad espectral de energa Sxx(f) dada en la figura, se filtra mediante un
f
sistema con funcin de transferencia H ( f ) 4 , para dar Syy(f). Se pide:
10
1.- Cuales son las expresiones de Sxx(f) y Syy(f)?
2.- Cual es la expresin de Ryy(t)? Cuanto vale la energa de la seal de salida?
Sxx(f)

-104

104

Hz

Solucin:
Ante todo son de ayuda las expresiones:

S xx ( f ) TF Rxx (t )

x( t ) x ( )d x(t ) x (t ) ;

Rxx (t )

Rxx (t )

xx

( f )e j 2 ft df ;

S yy ( f ) H ( f ) S xx ( f )

Ex

x (t ) dt

xx

( f )df Rxx (0)

Sxx(f)

Syy(f)

-104

-5103

H(f)2

5103

104

Hz

-5103

5103

Hz

Las expresiones que se piden son:


S xx ( f )

2 104

57

S yy ( f )
R yy (t )

10
4

f f

4 5 103 4 104

Sinc(10 4 t ) 5 10 3 Sinc 2 (5 10 3 t )
R yy (0)

15 10 3

3.6 TRANSFORMADA DE HILBERT Y SEAL ANALTICA

La transformada de Hilbert (TH) de una funcin del tiempo x(t) es otra funcin del tiempo.
Es la convolucin de la funcin 1/t con x(t). Se expresa de la siguiente manera,

hx (t )

1
1 x(t ' )
1 x(t t ' )
x(t )
dt '
dt '
t
t t '
t '

(3.12)

Ejercicio.

Calcular la TH de 1/t.
Solucin.
La TH es

1 1
, para su clculo procederemos del modo siguiente. La transformada de
t t

2
1 1
Fourier de 1/t es -jsign(f), por tanto, TF j sign( f ) 1
t t

As, la TF 1 1 (t )

Ejercicio.

Calcular la transformada de Hilbert de la transformada de Hilbert de x(t)


Solucin.
La expresin que se solicita es hhx (t )

1 1

x(t ) . Para evaluar esta expresin


t t

1 1
2

tomemos su transformada de Fourier TF x(t ) j sign( f ) X ( f ) X ( f )

t t
Tomando la transformada inversa del resultado se obtiene -x(t).
58

Ntese que calcular la transformada de Hilbert de una seal x(t) es equivalente a filtrar la
seal con una funcin de transferencia H ( f ) j sign( f ) . Las grficas siguientes muestran
la forma del mdulo y de la fase de esta funcin de transferencia.
|H(f)|

Fase

/2
f

-/2

a)

b)

Figura 3.1. Transformador de Hilbert a) mdulo. b) fase


1

As, H x ( f ) TF x(t ) j sign( f ) X ( f ) H ( f ) X ( f )


t

Tomar TH de una seal equivale a retrasar /2 todas las componentes positivas de la seal
y adelantar /2 las negativas.
Por tanto no es de extraar que la TH de cos(t) sea igual a sen(t) y la TH de la funcin
sen(t) sea igual a -cos(t).

3.6.1 SEAL ANALTICA


Se llama seal analtica de una seal real x(t) a la funcin del tiempo,
a x (t ) x(t ) jhx (t )
Si la transformada de Fourier de x(t) es X(f) entonces es fcil deducir que el espectro de
ax(t) es Ax ( f ) TF a x (t ) 2 X ( f )u ( f )
Es decir, el espectro de la seal analtica de una seal x(t) solo contiene sus frecuencias
positivas y la energa de la seal x(t) es igual a la de la seal analtica.
Recordemos que si la seal es real su espectro es simtrico, por lo tanto la reconstruccin
de la seal x(t) es obvia.

59

Ejercicio propuesto.

Encontrar la respuesta impulsional h(t) del transformador analtico.


x(t)

h(t)

ax(t)

Figura 3.2

3.6.2 ENVOLVENTE, FASE Y FRECUENCIA INSTANTNEA DE UNA SEAL


REAL
Se define envolvente de una seal real como el mdulo de la seal analtica. Esto es,
e x (t ) a x (t ) x(t ) 2 hx (t ) 2

(3.13)

Se define fase instantnea como el argumento de la seal analtica. Esto es,

x (t ) arga x (t ) tan 1

hx (t )
Imln(a x (t ))
x(t )

(3.14)

Segn todo ello se puede escribir:


a x (t ) e x (t )e j x (t ) e x (t ) cos( x (t )) je x (t ) sen( x (t )) x(t ) jhx (t )
Por tanto hx (t ) e x (t ) sen( x (t )) y x(t ) e x (t ) cos( x (t ))
Por otra parte ex(t) seria la seal de salida de un detector de picos o de envolvente ideal y

x (t ) sera la salida de un detector de fase ideal, si a la entrada de estos dispositivos estuviese


la seal x(t).
Se define frecuencia instantnea como la derivada con respecto al tiempo de la fase de la
seal analtica, todo ello dividido por 2. Esto es,

a (t ) h (t ) x(t ) x(t ) hx (t )
1 d x (t ) 1
d
1
f x (t )

Im ln(ax (t ))
Im x x
2 dt
2
ax (t )
2 ex (t ) 2
dt
2

(3.15)

La frecuencia instantnea sera la salida de un detector de frecuencia ideal a cuya entrada


se aplicase x(t).

60

3.6.3 SEAL REAL PASO BANDA EN FUNCIN DE SEALES PASO BAJO


En el supuesto que la seal x(t) tiene transformada de Fourier con todas sus componentes
frecuenciales concentradas en una determinada banda del espectro, la grfica de esta
transformada podra asemejarse a la figura siguiente 3.3 a).
X(f)

Ax(f)

-f0

f0

-f0

a)

f0
b)

Figura 3.3
En la figura 3.3 b) se representa la TF de la seal analtica de esta seal x(t) paso banda.
Se puede desplazar el espectro de la seal analtica a la zona baja de frecuencias de forma
que su expresin sera B x ( f ) Ax ( f f 0 ) y su TF-1 bx (t ) a x (t )e j 2f 0t .
Luego, a x (t ) bx (t )e j 2f 0t bx (t ) cos(2f 0 t ) jbx (t ) sen(2f 0 t ) . Esto implica que,

x(t ) Re al a x (t ) Re al bx (t )e j 2f 0t

y si bx(t), que no tiene porque ser real, la

descomponemos en sus partes real ix(t) llamada componente en fase, e imaginaria qx(t)
llamada componente en cuadratura, se obtiene,
x(t ) Re al a x (t ) i x (t ) cos(2f 0 t ) q x (t ) sen(2f 0 t )

(3.16)

En el caso que Bx(f) sea par en mdulo e impar en fase, entonces bx(t) sera real y la
expresin (3.16) se escribira como,
x(t ) Re al a x (t ) i x (t ) cos(2f 0 t )

(3.17)

La ecuacin (3.17), como se ver ms adelante es la expresin de una modulacin en


amplitud, en la que ix(t) es la seal de informacin tambin llamada moduladora. La ecuacin
(3.16) representa la expresin de otros tipos de modulacin.

3.7 EL TEOREMA DEL MUESTREO

La pregunta fundamental es cul es la frecuencia mnima a la que se puede obtener


muestras de una seal sin perder la informacin contenida en ella?, o lo que es lo mismo,
cual debe ser la separacin temporal entre muestra y muestra para que esta seal pueda ser
recuperada ntegramente?
61

Para realizar un estudio de esta cuestin, consideremos una seal x(t) de energa finita y del
tipo paso bajo. Es decir cuyo espectro no tiene componentes mas all de una cierta frecuencia
w. Supongamos que la seal x(t) y su espectro X(f) son los de la figura siguiente,
x(t)

X(f)
1
t

-w

a)

b)
Figura 3.4

Muestrear idealmente la seal x(t) a una frecuencia fs Hz es multiplicarla por un tren de


impulsos igualmente espaciados con periodo de muestreo Ts segundos (fs=1/ Ts). Esto es,

xz (t ) x(t ) (t nTs ) x(nTs ) (t nTs )


La seal muestreada idealmente estara formada por un tren de deltas de Dirac con el valor
de cada muestra contenido en el rea de su correspondiente delta. Esto es la representacin
ms exacta de la seal muestreada, puesto que en cada instante de muestreo se ha tomado su
nico valor. Esto no podra realizarse con un tren de pulsos, por ejemplo de onda cuadrada
con el que se producira una ambigedad tanto en el instante de muestreo como en el valor de
la muestra tomada.
El espectro de esta seal muestreada idealmente es:

Xz ( f ) X ( f ) TF (t nTs ) X ( f ) f s ( f mf s ) f s X ( f mf s )
m
m
n

Se muestra grficamente en la figura siguiente.


Xz(f)
fs

..............
-w

..............
w

fs

2fs

Figura 3.5
Se puede observar que el espectro de la seal x(t) se repite infinitamente. Por tanto toda
seal muestreada idealmente tiene un espectro peridico cuya frecuencia de repeticin es la
frecuencia de muestreo de la seal en el tiempo.
62

Si se estuviera interesado en recuperar la seal original x(t) a partir de la seal muestreada


xz(t), nicamente sera necesario filtrar esta seal muestreada con un filtro paso bajo ideal,
H( f )

1 f
, con lo que a su salida solo quedara el espectro de la seal X(f) y por
2w 2w

tanto la seal x(t) recuperada.


Es preciso notar que si la frecuencia de muestreo fs fuera menor que 2w habra un
solapamiento de los espectros que se repiten. Esto es,
fs

Xz(f)
..............

..............
-w

w fs

2fs

Figura 3.6
En este caso al intentar recuperar la seal x(t) a partir de xz(t) mediante el filtro paso bajo
se obtendra una seal distinta de la original. La figura siguiente muestra esquemticamente
esta hecho.
X'(f)
Xz(f)

H(f)

-w
Figura 3.7

Por consiguiente para que una seal x(t) pueda ser recuperada ntegramente a partir de sus
muestras, la frecuencia de muestreo debe ser mayor o igual que dos veces su ancho de banda.
Esto es,
f s 2w

(3.18)

a la cantidad f N 2w se ha dado en llamar frecuencia de Nyquist, que es la frecuencia


mnima de muestreo de una seal.
La mecnica de la recuperacin de la seal, en el supuesto que la frecuencia de muestreo
haya sido correcta, se puede estudiar a partir del espectro de la seal recuperada
f
X ( f ) Ts Xz ( f ) cuya transformada inversa de Fourier es,
fs

63

x(t ) xz (t ) sinc f s t

x(nTs ) (t nTs ) sinc f s t

x(nT )sincf

(t nTs ) (3.19)

Es decir la seal x(t) se reconstruye a partir de una suma o interpolacin de funciones 'sinc'
centradas en los puntos de muestreo, que se anulan en los instantes de muestreo distintos del
central y cuya amplitud es el valor de cada muestra.
x(t)

t
Ts

2Ts

3Ts

...........

Figura 3.8
En la prctica las seales son limitadas en el tiempo y sus espectros no son por tanto de
banda limitada, consecuentemente existira solapamiento en el espectro de la seal
muestreada y el error cometido ser tanto menor cuanto mayor sea la frecuencia de muestreo.
El muestreo ideal no es fsicamente realizable pero resulta una medida acerca de las
posibilidades de muestreo de una seal. Las aproximaciones que pueden obtenerse se conocen
como muestreos prcticos. Tampoco se aborda el muestreo de seales paso banda.

Ejercicio propuesto

Muestrear una seal no significa necesariamente perder parte de ella de una manera
definitiva, puede recuperarse ntegramente si se muestrea a una frecuencia conveniente.
a) Cul debe ser la frecuencia mnima a la que debe muestrear la seal x(t) del tipo sinc de
la figura, para que se pueda recuperar ntegramente?
b) Dibuje un esquema de bloques que recupere la seal inicialmente muestreada (si utiliza
algn filtro, indique cual es su ancho de banda).

64

x(t)
1

-1

2 3

t x 10-3 s

Ejercicio
Los diagramas de bloques de los llamados muestreo natural y muestreo instantneo se
representan en las figuras siguientes:

x(t)

xs(t)
x(t)

p(t)

h(t)

ys(t)

i(t)

muestreador natural

muestreador instantneo

t nT0
t
con
3

,
i
(
t
)

(t nT0 ) ,y h(t )
T



n
n
f
Con objeto de poder dibujar grficas, consideraremos X ( f ) , adems f0=1/T0=2B
B

Con p (t )

Se pide:
a.- Encontrar las expresiones de xs(t) e ys(t)
b.- Encontrar las expresiones de Xs(f), Ys(f) y dibujar sus grficas respectivas.
c.- Son iguales las seales muestreadas xs(t) e ys(t) ? En caso de no serlo cual es su
diferencia.
d.- Como recuperara exactamente x(t) para cada uno de los sistemas de muestreo ? Haga
un diagrama de bloques para cada uno y escriba las expresiones correspondientes de cada
subsistema.
Solucin:
a.
t nT0
xs (t ) x(t )

n

X s ( f ) X ( f ) f0

j 2 nf 0 t
x(t ) f 0 sinc( nf 0 )e

sinc( nf

) ( f nf 0 )

65

X s ( f ) f0

sinc( nf

) X ( f nf 0 )

1
n
sinc( ) X ( f nf 0 )

3 n
3

Xs(f)

f0

2f0

3f0

4f0

b.

t
t nT0
ys (t ) x(t ) (t nT0 ) x(nT0 )

Ys ( f ) X ( f ) f 0 ( f nf 0 ) sinc( f )
n

1
f

Ys ( f ) f 0 X ( f nf 0 ) sinc( f ) sinc(
) X ( f nf 0 )
3
3 f 0 n
n

Fs(f)
Y
(f)

f0

2f0

3f0

4f0

c.- La diferencia es que en el muestreador instantneo el espectro peridico de la seal


muestreada est multiplicado por una funcin sinc que distorsiona cada uno de los espectros
parciales, no ocurriendo lo mismo para el muestreador natural en el que nicamente solo se
ven afectados por un factor de escala.
d.- Para el muestreador natural basta un filtro paso bajo de ancho de banda B.
G(f)

f
f
G ( f ) 3
3
2B
f0

Para el instantneo es necesario compensar la distorsin en la banda baja haciendo pasar la


seal muestreada por el inverso del sinc y despus filtrar paso bajo, es decir si se filtra solo
1
f
X ( f ) , por lo tanto debera filtrar con:
paso bajo no se recupera x(t) sino sinc
3
3 f0

66

f
1
f
f 3 f0
G( f )
3
1

f
2
B

sinc(
2 B sen( f )
)
3
3 f0
3 f0

G(f)

-B

3.8 TRANSFORMADA DISCRETA DE FOURIER

En muchos casos se dispone de una seal determinada x(t) de la que no se conoce su


expresin analtica como puede ser la seal formada por los datos adquiridos durante la
realizacin de un proceso. El clculo de su TF puede ser realizado con ordenador mediante el
procedimiento conocido como Transformada Discreta de Fourier (TDF). Para ello es
necesario:
a) Representar la seal por una serie de muestras x(kTs) que se supondrn ideales. El
muestreo debe haberse realizado a una frecuencia fs correcta.
b) Si realmente la seal proviene de un proceso entonces se ha procedido a la
cuantificacin de los valores de las muestras. Es decir, sustitucin de sus valores por otros
comprendidos en un conjunto finito y discreto. Se comete as un error de cuantificacin que es
posible estudiar y limitar adecuadamente.
c) Considerar x(t) solamente durante un intervalo de tiempo limitado. Esto equivale a
multiplicar la seal por un pulso rectangular o de otro tipo (multiplicar por una ventana
temporal). Si se desea trabajar con N puntos o muestras de la seal, la duracin del pulso
rectangular debe ser T0=NTs. T0 es la longitud de registro y viene limitado por los recursos
disponibles para operar, su inversa f0 ser, como veremos mas adelante, la resolucin de la
TDF.
Hagamos un estudio de las posibilidades de la TDF y de los errores que se cometen,
propios al procedimiento.

67

Estudio de errores.

La figura a) siguiente muestra una seal x(t) junto con el tren de deltas de muestreo a
frecuencia fs y tambin un pulso de anchura T0 como ventana temporal esto es,
T0 Ts
t 2 2
. La figura b) presenta las muestras adquiridas de esta seal.

T

xA(t)

x(t)

Ts
T0=NTs

a)

Ts
T0=NTs

b)
Figura 3.9

As pues el ordenador dispone de un conjunto de valores x(kTs 0

N 1

representativos de la

seal que pueden ser expresados segn,


T0 Ts
t

2 2
x A (t ) x(kTs ) (t kTs )
T0
k

T0 Ts

j 2f

2 2
Su espectro XA(f) ser X A ( f ) f s X ( f nf s ) T0 sinc(T0 f )e
, que tambin
n

puede escribirse como,


T T

j 2f 0 s
2 2
X A ( f ) f s X ( f ) T0 sinc (T0 f )e
( f nf s )

(3.20)

Esta expresin muestra que el espectro X(f) de la seal x(t) que constituye la funcin que se
repite a intervalos fs, est convolucionado con una funcin 'sinc' y por tanto distorsionado.
Esta distorsin ser tanto menor cuanto mayor sea el ancho de pulso por el que se multiplica
x(t), es decir cuanto mayor trozo de seal se tome en la operacin menor ser el error que se
comete por esta causa. Supondremos, para facilitar la comprensin, que el espectro X(f) de la
seal x(t) tiene la forma de un pulso triangular.

68

Ntese que aunque el espectro X(f) de x(t) estuviese limitado en banda, por el hecho de ser
convolucionado con el espectro de la ventana temporal, en este caso el 'sinc', ya no quedar
limitado en banda y por tanto habr solapamiento en las colas de los extremos de cada periodo
del espectro de la seal muestreada. Vase la figura siguiente.
XA(f)

-fs

X(f)

w= fs/2

-fs/2 = -w

fs

Figura 3.10
Si se considera que todos estos errores son suficientemente pequeos entonces se puede
proceder a la operacin de hallar el espectro del trozo de seal x(t) muestreada, con algn
algoritmo del que se hablar mas adelante. Pero la evaluacin de errores todava no ha
terminado. Como que el clculo se realiza con ordenador, y ste solo trabaja con valores o
secuencias de nmeros, el espectro de esta seal muestreada, que es peridico pero continuo,
tambin debe estar muestreado (figura 3.11).
XA(f)

-fs

-fs/2 = -w

f0

fs/2 =w

fs

Figura 3.11
Ahora bien, este espectro peridico y muestreado debe cumplir que su transformada
inversa de Fourier debe ser igual a la seal en tiempo muestreada.
Sistematicemos conceptos. Toda seal muestreada tiene una transformada de Fourier que
es peridica, tambin se sabe que toda seal peridica tiene una transformada de Fourier
discreta.
Ahora recapitulemos. Nuestra seal de partida muestreada en el tiempo tiene un espectro
peridico, este espectro debe ser muestreado y por lo tanto ser discreto, entonces su
transformada inversa de Fourier, que debera ser la seal xA(t) de partida, ser discreta (lo es)
69

por ser el espectro peridico y tambin debera ser peridica (originalmente no lo es) por ser
el espectro discreto. En este ltimo caso el periodo que se repetir ser xA(t) y para que se
repita sin solapamiento en el tiempo es preciso que en el espectro XA(f) se hayan tomado
muestras cada f0 Hz.
Obtencin del algoritmo Transformada Discreta de Fourier (TDF).

La figura siguiente muestra la periodicidad en el tiempo que se supone se ha introducido


tcitamente cuando se pretende calcular la transformada de Fourier de una seal muestreada y
limitada en el tiempo.
xA(t)
...........

...........
T0=NTs

Ts

Figura 3.12
Cul es el algoritmo que proporciona las muestras del espectro de xA(t)? En realidad el
trabajo ya est hecho. Se sabe que toda seal peridica puede desarrollarse en serie de Fourier
y que los coeficientes dn de este desarrollo1 son precisamente las muestras del espectro que,
como ya se sabe, tambin es discreto. Entonces los coeficientes en este caso son,
1
dn
NTs
1

NTs

N 1

T
T0 s

T
s

x A (t )e
2
T
T0 s

x(kTs )
k 0

T
s

j 2 nf 0t

1
dt
NTs

(t kTs )e

j 2 nf 0t

T
T0 s

2 N 1

x(kT ) (t kT )e

T
s

k 0

j 2 nf 0t

dt

1
dt
NTs

N 1

x(kTs )e
k 0

T
T0 s
j 2 knf 0Ts

T
s

(t kTs )dt

kn
N 1
N 1
j 2
1
N
NTs x(kTs )e j 2 knf0Ts x(kTs )e
NTs
k 0
k 0

Por tanto puede escribirse la expresin siguiente:


N 1

X (nf 0 ) x(kTs )e

j 2

kn
N

(3.21)

k 0

Es la transformada discreta de Fourier. Representa las muestras del espectro espaciadas f0


Hz de una seal x(t) muestreada a fs Hz y limitada a N muestras. Ntese que la expresin

Ntese que la expresin dn se encuentra dividida por NTs=T0.

70

(3.21) es peridica cada Nf0=fs, por tanto solo es necesario considerar, por ejemplo, el periodo
obtenido desde n=0 hasta N-1.
No sera difcil llegar a obtener, por los mismos razonamientos, la transformada discreta
inversa de Fourier. Esto es,
x(kTs )

1
N

N 1

X (nf0 )e

j 2

kn
N

(3.22)

n 0

Se puede realizar la transformada discreta inversa de Fourier de un conjunto de muestras


que representan un espectro con el mismo algoritmo proporcionado en la ecuacin (3.21),
dado que la expresin (3.22) se puede escribir tambin como,
kn
j 2

1 N 1
N
x(kTs ) X ( nf 0 )e

N n 0

Significa que bastara conjugar las muestras a tratar, aplicar la TDF, volver a conjugar las
muestras resultado y finalmente dividir cada muestra por N
Segn los planteamientos que se han hecho es fcil deducir que los errores que
inevitablemente se cometen en el proceso se vern minimizados si la frecuencia de muestreo
de la seal temporal aumenta y tambin se aumenta N. Destacar tambin que la resolucin del
espectro aumenta con N.
Ahora se presentar un ejemplo que servir para poner de manifiesto los conceptos
anteriores.

Ejercicio.

Dada la seal x(t)=cos(2*pi*3*t)+cos(2*pi*8*t)+cos(2*pi*15*t), que est formada por tres


funciones sinusoidales a 3, 8 y 15 Hz, se desea encontrar su TDF si se toma una ventana entre
0 y 3 segundos y se muestrea a 50 Hz.
SOLUCIN:
La grfica analgica de la seal x(t) en este intervalo se muestra en la figura siguiente.
Esta seal se muestrea a intervalos de 0.02 segundos, lo que proporcionar 150 muestras.
Su TDF tambin tendr 150 muestras en frecuencia y su resolucin o paso en frecuencia f0
ser 1/3 Hz.
Bajo un punto de vista mas sistemtico, el vector de intervalo de tiempo es ts=0:1/50:31/50, el valor de la ventana temporal o tiempo total que dura el muestreo es
T0=NTs=1501/50=3 s.
71

Esto implica que las muestras del espectro estarn separadas unas de otras 1/T0=f0=1/3 Hz.

x(t)

-1

-2

-3

0.5

1.5
s

2.5

Figura 3.13. Seal x(t)


Aplicando el algoritmo TDF se obtiene un conjunto de valores, que en general sern
complejos, de modo que es conveniente presentarlos mediante lo que se llama espectro de
amplitud y de fase.
La grfica siguiente son las muestras del espectro de amplitud. En esta grfica se ha
exagerado el efecto de ventana, que se comenta mas adelante.
Como era de esperar se pueden apreciar las rayas espectrales a 3, 8 y 15 Hz
correspondientes a los tres tonos de la seal.
80
70
60

amplitud

50
40
30
20
10
0

Figura
Espectro
amplitud
de
10 3.14.
15
20
25 de 30
35
40 x(t)
45

50

Hz

Tambin se puede apreciar que a partir de los 25 Hz las muestras son una imagen especular
de las que aparecen entre 0 y 25 Hz. Esta imagen especular es la correspondiente a la parte
negativa del espectro. Es sabido que toda seal real tiene un espectro de amplitud simtrico
con respecto al origen de frecuencias, as bastara con tomar la primera mitad de los valores
obtenidos para tener la informacin del espectro de amplitud.

72

fase

-1

-2

-3
0

10

15

20

25
Hz

30

35

40

45

50

Figura 3.15. Espectro de fase de x(t)


La grfica anterior representa el espectro de fase y sucede lo mismo que en el espectro de
amplitud, habra que colocar la mitad derecha de la grfica al comienzo para tener la parte
negativa en frecuencias de la fase. Tambin podra omitirse dado que es sabido que en toda
seal real su fase es impar.
Ms detalles a destacar. El mdulo del espectro de tres tonos de igual amplitud debera
consistir en tres rayas espectrales de igual amplitud en sus respectivas frecuencias y ninguna
cosa ms. Sin embargo se puede observar que no son de igual amplitud y que en las
proximidades de estas rayas el espectro no es nulo; es decir existen ms componentes
frecuenciales de las esperadas. La explicacin pasa por observar que no se ha realizado la
transformada de Fourier de tres seales sinusoidales sino de tres seales sinusoidales que han
sido truncadas en el tiempo. Es decir se ha calculado la transformada de Fourier de la seal
x(t) multiplicada por una funcin pulso de duracin 3 segundos y centrada en 1.5 segundos,
provocando esta deformacin del espectro.
Se podra reducir esta deformacin truncando la seal x(t) de forma ms conveniente
multiplicando por otra funcin en lugar de multiplicar por un pulso rectangular. En lenguaje
habitual se ha dado en llamarlo 'elegir la ventana'.
Otro detalle. Recordemos que la frecuencia de muestreo de la seal x(t) ha sido 50 Hz, lo
que significa haber supuesto que el ancho de banda de la seal es 25 Hz. En realidad su ancho
de banda no es mayor que 15 Hz, dado por el tono de ms alta frecuencia, y por tanto se ha
sobre muestreado la seal.

73

Si la frecuencia de muestreo hubiera sido menor que 30 Hz, puede apreciarse en la grfica
de amplitud, no aparecera la raya espectral a 15 Hz y apareceran rayas a otras frecuencias
que no estn contenidas en la seal. Esto dan lugar al fenmeno denominado aliasing, que se
produce cuando se muestrea una seal a un frecuencia menor que el doble de su ancho de
banda.

Ejercicio propuesto
Se define la transformada discreta, directa e inversa de Fourier como:
N 1

X (nf 0 ) x(kTs )e

j 2k n

(directa)

k 0

1 N 1
j 2k n
N
X (nf 0 )e
(inversa)

N k 0
Ts=periodo de muestreo de x(t),

x(kTs )
Con NTs=T0,

N=n de muestras

y f0=1/T0.

* El formato de la funcin y de los datos depender del lenguaje que se utilice, las
especificaciones que se indican pretenden ser solo una gua.
- Realizar una funcin TDF(a(,),b(,),P,S), siendo:
Parmetro de entrada a(,), es una matriz de N filas y 3 columnas ( son las muestras de la
funcin a la que se debe aplicar el algoritmo).
Cada fila es una muestra de x(t) (o X(f)) que se estructura como t0, Real[x(t0)],
Imaginaria[x(t0)]
Parmetro de salida b(,), es una matriz de N filas y 3 columnas ( son las muestras de la
funcin resultado de aplicar el algoritmo).
Cada fila es una muestra de X(f) (o x(t)) que se estructura como f0, Real[X(f0)],
Imaginaria[X(f0)]
El parmetro de entrada P, puede valer 0 o 1 segn se desee la transformada directa o
inversa.
El parmetro de entrada S, puede valer 0 o 1 segn se desee que b(,) contenga las partes
real o imaginaria o bien el mdulo y la fase.

74

3.9 CONVOLUCIN Y CORRELACIN DISCRETAS

Se define la convolucin discreta de dos seales x(t) e y(t) como:


h(mT0 )

x(nT0 ) y(mT0 nT0 )

x(mT

nT0 ) y (nT0 )

Ejercicio propuesto
Basndose en la expresin anterior, realizar una subrutina o funcin con el formato
siguiente:

a.- CONVOLUCION(x,y,h), siendo x e y arrays conteniendo las muestras de las seales de


entrada y h un array conteniendo las muestras de la convolucin.
t
b.- Si y (t )
10T0

t
y x(t ) sinc

2T0

, dibujar h(t) con un nmero de muestras que

considere suficiente, empleando la funcin construida.

75

PROBLEMAS PROPUESTOS.

3.1.- El teorema de muestreo, como ya se sabe, establece que una seal x(t) debe ser
muestreada con una frecuencia mayor o igual al doble de su ancho de banda, si se requiere
recuperar esta seal sin distorsin.
Una seal cuya energa est concentrada en una banda de frecuencias se conoce como seal
pasa banda. La figura (Fig. a) muestra el espectro de una seal x(t) pasa banda.
X(f)
1
Figura a

-f2

-f1

f1

f2

De esta figura se observa que la frecuencia terica de muestreo sera 2f2. Sin embargo existe,
en este tipo seales, la posibilidad de muestrear a una frecuencia menor que la terica. El
diagrama de bloques de la figura b, permite muestrear la seal x(t) con un tren de deltas
p(t )

(t nT )

a la frecuencia (1/T) que se pide y recuperar despus la seal con un

filtro paso banda ideal como se muestra en la figura c.


a) Suponiendo que f1>(f2-f1), encuentre la frecuencia mnima de muestreo y los valores de las
constantes A, fa y fb tal que hacen xs(t)=x(t) del diagrama de bloques de la figura b.
H(f)

p(t)
x(t)

A
H(f)

xs(t)
-fb

Figura b

fa

-fa

fb

Hz

Figura c

3.2.- Considrese un auditorio con un problema de eco. Podemos modelar la acstica del
auditorio como una respuesta al impulso que consiste en un tren de impulsos, con el ksimo
impulso en el tren correspondiente al ksimo eco. Supongamos que en este caso particular la
respuesta al impulso est dada por:

h(t ) e kT (t kT )

donde e kT

representa la atenuacin del ksimo eco.

k 0

Para poder hacer una grabacin de alta calidad en el escenario, los ecos deben ser eliminados,
realizando algn procesamiento de los sonidos detectados por el equipo de grabacin.
76

El sistema total se puede esquematizar segn el diagrama de bloques siguiente:


x(t)

xeco(t)
h(t)

x(t)

g(t)

Donde x(t) es la seal proveniente de la fuente, xeco(t) representa la seal que proviene del
auditorio, modelado por h(t) y que ser procesada por el sistema con respuesta impulsional
g(t) con intencin de suprimir las seales de eco.
a) Escoja g(t) de manera que los ecos sean completamente eliminados y la seal resultante sea
una reproduccin fiel de los sonidos originales de la fuente.
b) Comprobarlo.

3.3.- En el sistema mostrado en la figura, dos funciones de tiempo x(t) e y(t) son multiplicadas
y el producto w(t) se muestrea por un tren de impulsos. x(t) es de banda limitada a f1 e y(t) es
de banda limitada a f2, esto es, X(f)=0 para |f|f1

Y(f)=0 para |f|f2

p(t )

(t nT )

Y(f)

X(f)

x(t)
x

wp(t)

y(t)

-f1

f1

-f2

f2

Determinar el intervalo de muestreo mximo T tal que w(t) se pueda recuperar a partir de
wp(t) mediante un filtro paso bajo ideal.

3.4.- Se dispone de la seal x(t) dada en la grafica siguiente, y se muestrea cada 0.25
segundos, por tanto obtendr nueve muestras.
x(t)
1
1

t (s)

a) Calcular la expresin del espectro discreto X(n) de x(t).


b) Dibujar el espectro de su amplitud y el espectro de fase de X(n), expresando claramente
los valores en los ejes de coordenadas.
77

T
T
1 1
cos(2 f p t ) para p t p
siendo
3.5.- La funcin p(t) se describe como: p(t ) 2 2
2
2

0
en cualquier otro caso

fp=1/Tp=1 Hz
Se pide:
1) Dibuje la grfica y escriba una expresin para la funcin p(t) de forma que sea til en los
apartados siguientes.
2) Escribir la expresin P(f) del espectro de la seal p(t)
3) Dibujar, de manera razonable, la grafica del espectro de amplitud y la grafica del espectro
de fase de P(f) (puede ser conveniente ayudarse de graficas parciales). Calcule sus valores a
las frecuencias de 0, 1, 2, 2.5, 3.5 y 9.5 Hz.

3.6.- Se desea muestrear una seal x(t), cuya amplitud est normalizada a uno, con el tren de
pulsos pm (t )

t nT0
) , siendo la frecuencia de muestreo f0=2b=1/T0 igual a 8 kHz. El
0 10

p( T

diagrama de bloques de este proceso de muestreo se esquematiza en la figura a) y la grfica


del espectro de x(t) con b= 4 kHz se muestra en la figura b).
x(t)

xm(t)
pm(t)
Figura a

X(f)
-b

1
b

Hz

Figura b

Demuestre si es posible o no recuperar la seal x(t). En caso afirmativo proponga un diagrama


de bloques que permita recuperarla a partir de la seal muestreada xm(t). Escriba la/las
expresin/expresiones en tiempo y en frecuencia de cada elemento del bloque.
a) La seal muestreada xm(t) se convoluciona, con h1 (t ) 4b sinc(2bt ) cos(2 10 f 0t ) . Escriba
la expresin resultante de y (t ) xm (t ) h1 (t ) .
78

b) A partir de la seal y(t) del apartado anterior, muestre como recuperar la seal x(t). Escriba
un diagrama de bloques de este procedimiento.

3.7.- Se sabe que la funcin x(t) dada en la figura, en la que se muestran nicamente 12 s, se
expresa como x(t ) 10sinc(10t )* (t ) .
Se pide:
a) Dibuje su espectro de amplitud y su espectro de fase
b) Cual es su frecuencia de muestreo mnima?

3.8.- La figura a) muestra la grfica de una seal peridica con periodo de repeticin T0=4
segundos. La figura b) muestra, en lnea gruesa, la seal generatriz. Las lneas finas, dan una
idea de como podra generarse una expresin de esta seal generatriz a base de sumas y restas
de funciones.

x(t)

Figura b)
Figura a)

p(t)
_1

.............
-4 -3 -2 -1

1 2 3 4 5

2
1

-2

-1

.............
t s

T0=4
Se pide:
a) Escriba una expresin de la funcin generatriz x(t).
b) Obtenga la transformada de Fourier de la funcin generatriz x(t).
c) Desarrolle en serie de Fourier la seal p(t). Calcule el valor sus coeficientes desde n=0
hasta n=4. Escrbala como una suma de funciones sinusoidales, al menos con los 5 primeros
trminos (utilice los coeficientes calculados).
d) Escriba la expresin de P(f). Dibuje esquemticamente el espectro de amplitud de P(f).
Diga tambin que frecuencias armnicas no contendr este espectro.
79

e) Si fuese necesario muestrear esta funcin peridica p(t) explique que pasos seguira, que
frecuencia de muestreo elegira y por que razn. Si no pudiera recuperar ntegramente la seal
original explique tambin que efecto tendra en la seal recuperada.

3.9.- La figura a) muestra un cierto sistema con una seal de entrada x(t). La grfica de la
funcin de transferencia H(f) del sistema se representada en la figura b). Se sabe que la seal
x(t) no contiene frecuencias superiores a f0 Hz.
H(f)
h(t)

x(t)

x(t)

f0

y(t)
- f0

Figura a)

f0

Hz

dt

-1/

y(t)

sen(2f0t)
Figura c)

Figura b)

Opere el sistema h(t) para mostrar que puede ser construido como en la figura c).

t T0
t T0

2
3.10.- Si en en el sistema de la figura a) x(t ) 2
y h(t )
T
2
T
0

y(t)
h (t )
x(t)

Figura a)

(t)+(t-T0)+(t-2T0)

a) Representar grficamente x(t)


b) Calcular la seal de salida y(t) y representarla grficamente.

3.11.- La figura a) muestra la grfica de una seal x(t) peridica con periodo de repeticin T0
segundos.
x(t)

g(t)

.
-

Figura a)

T0 +T0/4
T0/4
T0/4+ T0/2

80

Se pide:
a) Escriba una expresin de la funcin generatriz g(t).
b) Desarrolle en serie de Fourier la seal x(t). Calcule el valor de los coeficientes desde n=-5
hasta n=5. Escrbala como una suma de funciones sinusoidales.
c) Obtenga la transformada de Fourier X(f) de la funcin x(t). Dibuje esquemticamente el
espectro de amplitud de X(f).
d) Con un filtro ideal, tal como se representa en la figura siguiente, se filtra la seal x(t) para
obtener nicamente su primer armnico (seal y(t)) a la frecuencia f0.
x(t)

y(t)

h(t)

3) Escriba la funcin de transferencia H(f) y la respuesta impulsional h(t) de este filtro


4) Escriba la expresin de la seal y(t). Como y a que frecuencia tomara muestras de esta
seal peridica y(t). Por qu?

3.12.- La seal x(t), de entrada al sistema, tiene un ancho de banda b Hz, siendo f0>>b.
Obtener la expresin de y(t), calcular su espectro y dibujar su grafica.
X(f) 1
x(t)

h(t)=j/t

y(t)

-b

Hz
f

cos(2f0t)

3.13.- Se desea que la seal analgica de salida y(t) sea una buena aproximacin a la derivada
de x(t). Muestre en que margen de frecuencias es as y explique las relaciones necesarias entre
frecuencia de muestreo y ancho de banda b. Escriba una respuesta impulsional para el filtro
paso bajo.
x(t)

(t nT )

h(t)=(t-T0)

-
+

Fil. Paso
Bajo

X(f) 1

y(t)
-b

Hz
f

81

h0(t ) t ,

3.14.- En el sistema de la figura

h1(t ) t T0 ,

h 2(t ) t 2T0 ,

h3(t ) t 3T0 y h 4(t ) t 4T0 . La seal de entrada x(t) tiene un espectro genrico X(f)

que tambin se muestra grficamente.


x(t)

h0(t)
X(f) 1

h1(t)
h2(t)

y(t)

h3(t)

-b

Hz
f

h4(t)
Mostrar que si b

2
1 t 2T0
, entonces y(t) se puede escribir como y (t ) x (t )

T0
T0 5T0

3.15.- En el sistema de la figura h(t ) 1 . La seal x(t) tiene un espectro genrico X(f).
t

X(f)
x(t)cos(2f0t

y(t)

h(t)

-b

1
b

Hz
f

Obtener las expresiones y(t) e Y(f)

3.16.- La figura a) muestra un cierto sistema con una seal de entrada x(t) que tiene un ancho
de banda B Hz, y (t ) 2a sinc(2at ) , p (t )

(t nT ) .
0

x(t)
Figura a)

y(t)

v(t)

p(t)

z(t)

z(t)

Diagrama
de
bloques

x(t)

Figura b)

a) Opere las seales para determinar la frecuencia de muestreo mnima a emplear en la


secuencia p(t). Escriba la expresin de la seal z(t). Con un espectro genrico para la seal
v(t), dibuje esquemticamente el espectro de amplitud de la seal z(t).

82

b) Proponga un diagrama de bloques que permita recuperar x(t) a partir de z(t), sabiendo que
x(t) e y(t) son dos seales que estn incorreladas. Escriba las expresiones de los subsistemas
que forman el diagrama de bloques recuperador.

3.17.- A partir de un derivador analgico ideal, cuya funcin de transferencia es H(f)=j2f, se


desea obtener las muestras de la respuesta impulsional h(n) de un derivador discreto. Se sabe
que el mximo ancho de banda de las seales a derivar no excede del valor f 0 2 Hz. Tambin
se sabe que el procesador digital a usar solo dispone de N posiciones de memoria para
almacenar las muestras de la respuesta impulsional de este filtro.
a) Obtenga la expresin discreta del valor de estas N muestras.
b) Suponiendo N=9, y f0=1 Hz obtenga el valor de estas muestras y escriba esta secuencia de
la primera a la ltima Cual es el valor del retardo en el tiempo introducido por este filtro a
una secuencia de entrada?. Puede eliminar este retardo?.Diga cmo hacerlo.
**
H( f )

Notar

d e

j 2 nT0 f

la
T0

TF

diferencia
1

del

cambio

de

dominio

H g ( f )
e j 2 nT0 f
t nT0

83

CAPTULO 4
4 MODULACIONES ANALGICAS

4.1 MODULACIONES

El objeto bsico de la modulacin de una seal es trasladar su contenido frecuencial a otra


posicin o zona del espectro ms conveniente para algn fin, por ejemplo transmitirla va
radio, evitar zonas ruidosas del espectro, etc Esto es una modulacin paso banda. El
procedimiento consiste en modificar la amplitud de una funcin sinusoidal, dando lugar a las
llamadas modulaciones lineales o bien modificar la fase de una funcin sinusoidal, dando
lugar a las llamadas modulaciones angulares. La funcin sinusoidal modificada se llama
portadora y la seal de informacin que la modifica se llamada moduladora.
Existe tambin la posibilidad de modificar algn parmetro de una portadora pero sin
cambiar sustancialmente de banda o zona del espectro de la seal de informacin. En este
caso se denominara modulacin en banda base.
Hay dos grandes categoras: Modulaciones de amplitud y modulaciones angulares. Cada
una de ellas puede dividirse en modulaciones analgicas y modulaciones digitales. En las
modulaciones analgicas la seal de informacin es analgica y en las modulaciones digitales
la seal de informacin es discreta.
En ste captulo se estudiaran las modulaciones analgicas de amplitud y modulaciones
analgicas angulares

4.2 MODULACIONES ANALGICAS DE AMPLITUD

Las modulaciones lineales son aquellas en las que una superposicin de moduladoras da
lugar a una superposicin de seales moduladas. Las seales moduladas linealmente se
obtienen modificando la amplitud A de una portadora (un tono) a una frecuencia fija:
x p (t ) A cos(2f c t )

(4.1)

Los mtodos que se utilizan pueden ser clasificados en:


- Modulacin convencional de amplitud.
84

- Doble banda lateral con portadora (double sideband with transmision carrier
DSB-TC)
- Modulacin de doble banda lateral con portadora en cuadratura (Quadrature
double sideband with transmision carrier QDSB-TC)
- Modulacin en doble banda lateral.
- Doble banda lateral sin portadora (double sideband suppressed carrier DSBSC)
- Modulacin en cuadratura de doble banda lateral sin portadora (Quadrature
double sideband suppressed carrier QDSB-SC)
- Modulacin en banda lateral nica (single sideband suppressed carrier SSB).
- Modulacin en banda lateral asimtrica (asymetric sideband ASB).
- Modulacin en banda lateral vestigial (vestigial sideband VSB).
Seguidamente se analizan estos tipos de modulacin comenzando por la modulacin en
doble banda lateral con portadora suprimida, dada su facilidad conceptual. En este apartado
tambin se estudiar el problema de la sincronizacin de la portadora en el demodulador.

4.2.1 MODULACIN EN DOBLE BANDA LATERAL CON PORTADORA


SUPRIMIDA
La expresin en el tiempo de una seal modulada en doble banda lateral con portadora
suprimida (DSB-SC) es:
x DSB SC (t ) x(t ) cos(2f c t )

(4.2)

Donde x(t) es la moduladora o seal de informacin. fc, es la frecuencia de la seal


sinusoidal llamada tambin portadora; debe ser mucho mayor que la frecuencia mxima del
espectro de la seal x(t).
A modo de ejemplo, supngase que el espectro de x(t) sea X(f):
X(f)
A

-a

Figura 4.1
Entonces la expresin del espectro de la seal modulada ser:
85

X DSB SC ( f )

1
1
X ( f fc ) X ( f fc )
2
2

(4.3)

y su grfica:
XDSB-SC (f)

Banda lateral
inferior

A/2

-fc-a

-fc

-fc+a

fc-a

fc

Banda lateral
superior

fc+a

Figura 4.2. Espectro de x(t) modulada en doble banda lateral con portadora suprimida
En la figura 4.2. se aprecia que el espectro de la seal x(t) queda inalterado pero es
trasladado en frecuencia y centrado a la frecuencia de la portadora. Tambin es preciso notar
que la parte negativa del espectro de la seal x(t) est ahora incorporado a la parte positiva del
espectro de la seal modulada. Por tanto la informacin contenida en la seal x(t), que radica
en las frecuencias positivas (o bien las negativas dado que existe simetra), se encuentra
duplicada en la seal modulada (bandas laterales inferior y superior). Tambin es notorio que
no aparece ningn tono en el espectro de la seal modulada, de aqu procede el significado de
la frase 'portadora suprimida'.

Ejemplo

Modular en doble banda lateral con portadora suprimida la seal x(t ) sin(2 1 t ) con
una portadora fc de 10 Hz.
La seal modulada ser:
x DSB SC (t ) sin(2 1t ) cos(2 10t )
El diagrama de bloques del modulador DSB-SC es:
xDSB-SC(t)

sin(21t)
cos(210t)

86

Su espectro de amplitud terico ser:


XDSB-SC (f)
1/4

-11

-10 -9

10

11

Hz

Figura 4.3. Espectro de amplitud de la seal modulada en doble banda lateral con portadora
suprimida
Y su grfica temporal:
xDSB(t)
0.8
0.6
0.4
0.2
0
-0.2
-0.4
-0.6
-0.8
-1
0

0.2

0.4

0.6

0.8

1.2

1.4

1.6

1.8

(s)

Figura 4.4. Dos segundos de seal modulada en doble banda lateral con portadora
suprimida
En trazo discontinuo se muestra la seal moduladora, que ser preciso recuperar durante el
proceso de demodulacin.

DEMODULACIN
4.2.1.1
Para recuperar la seal x(t) a partir de la seal modulada se podra volver a utilizar el
mismo concepto de modulacin, esto es, trasladar el espectro a su posicin original.
Analticamente en el tiempo es:

87

1 1

xd (t ) x(t ) cos(2f ct ) cos(2f ct ) x(t ) cos2 (2f ct ) x(t ) cos(2 2 f ct )


2 2

xd (t )

x(t ) x(t )

cos(2 2 f c t )
2
2

(4.4)

El espectro Xd(f) de esta seal xd(t) se muestra en la figura siguiente. Se aprecia que esta
seal si se filtra paso bajo con un ancho de banda adecuado (mayor que a Hz y menor que 2fca Hz) se recupera el espectro original (salvo un factor de escala)
Filtro
paso bajo

Xd(f)
A/2
A/4

-2fc-a

-2fc -2fc+a

-a

2fc-a

2fc

2fc+a

Figura 4.5. Espectro de amplitud de la seal xd(t)


Entonces el diagrama de bloques de la demodulacin sncrona o de producto de la DSB-SC
es:
xDSB-SC(t)=x(t) cos(2fct)

x(t) cos2(2fct)

Filtro
paso bajo

x(t)/2

Oscilador local
cos(2fct)
Figura 4.6. Demodulacin sncrona

4.2.1.1.1

SINCRONIZACIN

Necesidad.

Como se ha podido observar, al realizar la demodulacin sncrona, mediante


multiplicacin con un tono, se ha seleccionado ste de igual frecuencia y fase que la portadora
recibida (o la que se recibira si estuviese presente). Estudiemos que podra suceder si el
oscilador local posee una desviacin instantnea de fase (t ) lo suficientemente lenta, es

88

decir que varia mucho ms despacio que la seal moduladora. La funcin del oscilador local
es entonces A cos(2f c t (t )) , que trigonomtricamente es equivalente a:
A cos(2f c t ) cos( (t )) Asin(2f c t ) sin( (t ))
As, en la demodulacin, la salida del multiplicador sera:
xd (t ) Ax(t ) cos(2f c t ) cos(2f c t (t ))
Ax(t ) cos(2f c t ) cos(2f c t ) cos( (t )) sin(2f c t ) sin( (t ))

Ax(t ) cos( (t )) cos 2 (2f c t ) sin( (t )) sin(2f c t ) cos(2f c t )


1
1

Ax(t ) cos( (t )) 1 cos(2 2 f c t ) sin( (t )) sin(2 2 f c t )


2
2

A la salida del filtro paso bajo quedarn nicamente los trminos en baja frecuencia:
x de mod (t )

A cos( (t ))
x(t )
2

(4.5)

La consecuencia evidente es que la amplitud de la seal de informacin varia segn lo hace

(t ) y esto no interesa porque la distorsiona. Adems cuando (t ) valga /2 la seal


demodulada se habr perdido. Por tanto, dado que la atenuacin introducida siempre puede,
en teora, compensarse con una amplificacin, podra admitirse un desfase entre portadora y
oscilador local siempre que sea constante y diferente de /2.

Realizacin

As, para poder demodular una seal sin portadora, es necesario disponer en el receptor de
un tono a la misma frecuencia y fase que la portadora de la seal modulada. El procedimiento
consiste en sintetizarlo a partir de un sistema realimentado que vaya comparando algn
parmetro de la seal de salida con la de entrada, que es normalmente la seal modulada.
En la figura siguiente se presenta el diagrama de bloques del lazo Costas que implementa
este procedimiento.
Supngase que se tiene una portadora de frecuencia fc, entonces el oscilador controlado por
tensin VCO del lazo debera ser diseado para que su frecuencia libre de oscilacin fuese fc,
esto es, cos(2f c t ) . Supngase tambin que existe una diferencia de fase (t ) entre portadora
y

VCO.

Esto

puede

escribirse

considerando

la

seal

de

salida

del

VCO

como cos(2f c t (t )) .

89

xDs(t)

xDSB-SC(t)=x(t) cos(2fct)

Filtro
paso bajo

cos(2fct+(t))

VCO

xc(t)

xD(t)

Filtro
paso bajo
h(t)

-/2
sin(2fct+(t))

xAs(t)

Filtro
paso bajo

xA(t)

Figura 4.7. Lazo Costas


Entonces el funcionamiento del lazo Costas es como sigue:
1
1

x Ds (t ) Ax(t ) cos( (t )) 1 cos(2 2 f c t ) sin( (t )) sin(2 2 f c t )


2
2

y a la salida del filtro paso bajo x D (t )

A cos( (t ))
x(t )
2

1
1

Por otra parte x As (t ) Ax(t ) cos( (t )) sin(2 2 f c t ) sin( (t )) 1 cos(2 2 f c t )


2
2

y a la salida del filtro paso bajo x A (t )


La seal

xc (t )

Asin( (t ))
x(t )
2

A 2 sin( (t )) cos( (t ))
x(t ) 2 h(t) acta sobre el VCO tendiendo a
4

igualar la fase de la portadora, es decir a hacer cero a (t ) . En este caso cos( (t )) =1 y

sin( (t )) =0.
Cuando esto sucede no aparece seal correctora en el VCO, existe sincronismo entre
portadora y demodulador y la seal xD(t) es la seal demodulada con la informacin original.

Ejemplo

La figura muestra otra configuracin para obtener una seal sincronizada a la frecuencia de
la portadora.

90

En el supuesto que el elemento no lineal tuviera una caracterstica como


x D1 (t ) a 2 x DSB SC (t ) 2 , entonces x D1 (t )

a 2 x(t ) 2
1 cos(2 2 f c t )
2

paso banda centrado a 2fc se obtendra x D 2 (t )

Elemento
no lineal
xD1(t)

Filtro
paso banda
h(t)

despus de filtrar

a 2 x(t ) 2
cos(2 2 f c t ) h(t )
2

f 2
xD2(t)

xDSB-SC(t)=x(t) cos(2fct)

Filtro
paso bajo

xD(t)

Aunque la seal x(t) no tuviera componente a la frecuencia nula, su producto si lo tendr.


Por tanto si el filtro paso banda es muy estrecho y centrado a la frecuencia 2fc se obtendr un
tono a esta frecuencia. La amplitud ser estable dado que la transformada de Fourier de x(t)2
es la transformada de Fourier de la potencia de la seal y tomar su componente a frecuencia
cero significa potencia media. Posteriormente se divide esta frecuencia a la mitad.
Seguidamente se muestra una forma ms intuitiva del proceso.
Se intenta mostrar que, an en el supuesto que el espectro de la seal x(t) tenga un vaco
espectral en la banda baja de frecuencias se puede obtener una seal sinusoidal sincronizada
con la portadora. Pinsese que la modulacin en doble banda lateral tambin estar vaca
alrededor de la frecuencia fc Hz y por lo tanto no podra recuperarse ninguna portadora.
En la primera grfica, de la figura siguiente correspondiente a la seal de entrada al
elemento no lineal, parece que hay una frecuencia a fc Hz, pero no es as, la frecuencia es ms
alta. Cada cruce por cero de la funcin moduladora x(t) introduce un cruce por cero adicional
para la funcin sinuosidad y por tanto un aumento de frecuencia. Tambin se observa que no
se producen discontinuidades ni saltos. Est claro que nunca puede haber una funcin
sinusoidal a frecuencia fc Hz a menos que no se produjese ningn cruce por cero de x(t), pero
esto solo sucede en la modulacin en doble banda lateral con portadora (AM convencional).

91

x(t)sen(2fct)
x(t)

Tc/2

Tc

x(t)2sen (22fct)
x(t)2

Tc/2

Tc

En la segunda grfica, para hacerlo evidente, se representa x(t ) 2 sen(2 2 f ct ) en lugar de


1 1

x(t ) 2 sen 2 (2 f c t ) x(t ) 2 cos(2 2 f ct ) . A simple vista puede parecer semejante a la


2 2

grfica anterior pero a frecuencia doble. No es as. Ahora la seal x(t)2 nuca tendr cruces por
cero, nicamente valdr cero donde antes cruzaba. El resultado de multiplicar con la funcin
sinusoidal es el que se aprecia en la figura. No hay cambios de fase en la funcin sinusoidal,
simplemente tomar el valor cero en esos puntos. No introduce cruces adicionales en la
funcin sinusoidal y por tanto no modifica la frecuencia fundamental 2 fc Hz. Por tanto existe
una frecuencia a 2 fc Hz ( y otras).
La manera mas sencilla de obtenerla es filtrando paso banda lo mas estrecho posible. De
este modo se reconstruyen los lbulos de la funcin sinusoidal deteriorados por los valores
cero y tambin se reduce la variacin de amplitud producida por moduladora x(t)2.
Se dice que si el filtro paso banda con respuesta impulsional h(t) es estrecho y centrado a la
frecuencia de 2fc se obtiene una funcin sinusoidal a frecuencia 2fc sincronizada con la
portadora a frecuencia fc Hz
El la grfica siguiente se muestra lo que se quiere decir con esta reconstruccin.
92

x(t)2sen(22fct)

Tc/2

Cuanto ms estrecho sea el filtro baso banda centrado en 2fc mejor ser el
redondeo y menor las variaciones de amplitud

Tc

Finalmente ntese que los cruces por cero de todas las grficas estn relacionados. Si se
produce un desplazamiento en el cruce por cero de la seal en la primera grfica, se traducir
tambin en un desplazamiento del cruce por cero de la segunda y tercera graficas. Por tanto la
seal sinusoidal de la ultima grafica esta perfectamente sincronizada con la de la primera.
En la figura siguiente se muestra una simulacin con una portadora a 1Hz (sen(21t))y la
moduladora a 0.133 Hz (x(t)= cos(20.133t))

x(t)

x(t)sen(21t)

x(t)2

(s)

x(t)2sen2(21t)

(s)

Todo esto representa un intento de explicar intuitivamente algo que no lo es, o que no lo es
a primera vista. Por esta razn, adems del planteo y solucin del problema en el en el
93

dominio de la frecuencia, se ha recurrido a imgenes en el dominio del tiempo para presentar


este interesante concepto, tal como se ha hecho en el anterior apartado.

4.2.2 MODULACIN EN CUADRATURA DE DOBLE BANDA LATERAL SIN


PORTADORA
Se ha comprobado que, admitiendo enganche en frecuencia, un desfase de /2 supone la
prdida de seal. Segn esto es posible enviar sobre el mismo ancho de banda dos
informaciones distintas x(t) e y(t), modulando dos portadoras en DSB-SC (tambin en doble
banda lateral con portadora) con tal que estas portadoras estn en cuadratura y que se realice
una deteccin sncrona para cada seal de informacin (cada canal).
La expresin de una modulacin en cuadratura de doble banda lateral sin portadora
(Quadrature double sideband suppressed carrier QDSB-SC) es:
xQDSB SC (t ) x(t ) cos(2f c t ) y (t ) sin(2f c t )

(4.6)

Tal como se observa esta modulacin se obtiene como suma de dos modulaciones en
amplitud. Las seales x(t) e y(t) tienen un ancho de banda limitado a b Hz. Su transformada
de Fourier es:
X QDSB SC ( f )

1
1
1
X ( f f c ) X ( f f c ) Y ( f f c ) Y ( f f c ) (4.7)
2
2
2j

y el mdulo de esta transformada podra tener un aspecto semejante al de la figura 4.8.


|XQDSB-SC (f)|
X(f-fc)/2

-fc-b

-fc

-fc+b

fc-b

Y(f-fc)/2

fc

fc+b

Figura 4.8. Espectro de amplitud de una modulacin en cuadratura


Ntese que ambas seales de informacin ocupan la misma zona del espectro y
aparentemente no sera posible volver a separar estas seales. Sin embargo segn el diagrama
de bloques siguiente esta informacin es recuperable.

94

xDs(t)

xQDSB-SC(t)=x(t) cos(2fct)+ y(t) sin(2fct)

Filtro
paso bajo

xD(t)

cos(2fct)
cos(2fct)
(sincronizado)
-/2
sin(2fct)

Filtro
paso bajo

xAs(t)

xA(t)

Figura 4.9. Diagrama de bloques de demodulacin en cuadratura


En la primera rama a la salida del multiplicador se obtiene:
x Ds (t ) x(t )

1
1 cos(2 2 f c t ) y(t ) sin(2f c t ) cos(2f c t )
2

A la salida del filtro paso bajo de ancho de banda b se obtendra:


x D (t )

x(t )
2

En la otra rama:
x As (t ) x(t ) cos(2f c t ) sin(2f c t ) y (t )

1
1 cos(2 2 f c t )
2

Y en la salida del filtro paso bajo se obtendra: x A (t )

y (t )
2

4.2.3 MODULACIN EN DOBLE BANDA LATERAL CON PORTADORA


Tambin llamada modulacin convencional de amplitud. La caracterstica esencial de una
modulacin convencional de amplitud es que, como se ver mas adelante, en su espectro
destaca la presencia de la portadora, indicando que parte de la energa empleada en generar
esta seal se utiliza en mantener la portadora en detrimento de la seal moduladora.
La expresin en el tiempo de una seal modulada en doble banda lateral con portadora
(DSB-TC) es:
x DSB TC (t ) A1 mx(t ) cos(2f c t )

(4.8)

95

Donde x(t) es la moduladora o seal de informacin. El parmetro m debe tener un valor


tal que 1 mx(t ) siempre sea mayor que cero.
Es fcil suponer que la expresin en el tiempo de una modulacin en doble banda lateral
con portadora en cuadratura (QDSB-TC) es:
xQDSB TC (t ) AF 1 mx(t ) cos(2f c t ) AC 1 my(t ) sin(2f c t )

(4.9)

En esta expresin x(t) e y(t) son dos moduladoras distintas. No se har hincapi en este
tipo de modulacin puesto que, con las salvedades para DSB-TC, es conceptualmente idntico
a lo expuesto para DSB-SC en el apartado anterior.
A modo de ejemplo y siguiendo las mismas pautas que en la modulacin DSB-SC,
supngase que el espectro de x(t) sea X(f):
X(f)
A

-a

Figura 4.10
Entonces la expresin del espectro de la seal modulada en DSB-TC ser:
X DSB SC ( f )

Am
X ( f f c ) X ( f f c ) A ( f f c ) ( f f c )
2
2

(4.10)

y la grfica que muestra su espectro de amplitud:


XDSB-SC (f)

Banda lateral
inferior

A/2

-fc-a

-fc

-fc+a

fc-a

fc

Banda lateral
superior

fc+a

Figura 4.11. Espectro de x(t) modulada en doble banda lateral con portadora

96

Como se puede observar en el espectro, existe una delta de Dirac a la frecuencia de la


portadora. El ejemplo siguiente servir para fijar conceptos.

Ejemplo.

Modular la seal x(t ) sin(2 1 t ) en doble banda lateral con portadora probando con
amplitudes de portadora e ndices de modulacin diferentes.
Primeramente con una portadora de amplitud 2, frecuencia fc de 10 Hz y un ndice de
modulacin m=0.5.
La seal modulada ser:
x DSB TC (t ) 2(1 0.5sin(2 1t )) cos(2 10t )
El diagrama de bloques del modulador DSB-TC es:
0.5sin(21t)

xDSB-TC(t)
1

2cos(210t)

En la figura siguiente se muestran 2 segundos de esta modulacin

3
2
1
0
-1
-2
-3
0

0.2

0.4

0.8

0.6

1.2

1.4

1.6

1.8

Figura 4.12
La misma modulacin pero con una amplitud de tono y un ndice de 1;
x DSB TC (t ) (1 sin(2 1t )) cos(2 10t )
97

2
1
0
-1
-2

0.2

0.4

0.6

0.8

1.2

1.4

1.6

1.8

2 s

Figura 4.13
Ahora

con

una

amplitud

de

tono

de

3/5

un

ndice

de

1.5;

3
xDSB TC (t ) (1 1.5sin(2 1t )) cos(2 10t )
5
1.5
1
0.5
0
-0.5
-1
-1.5

0.2

0.4

0.6

0.8

1.2

1.4

1.6

1.8

2 s

Figura 4.14
Ntese que cuando el ndice de modulacin es mayor que 1, la forma de onda descrita por
los picos superiores o inferiores de la portadora (esta seal se llama envolvente) ya no es la
misma que la seal moduladora. Esta situacin no es deseable en una modulacin de amplitud
con portadora; anula la ventaja esencial que tiene este tipo de modulacin para recuperar la
seal moduladora.
El siguiente ejercicio propuesto muestra un modulador prctico del tipo llamado
balanceado.

Ejercicio propuesto

98

Determinar la expresin de m(t)


e1(t)

Elemento
no lineal

s1(t)

+
kx(t)

cos(2fct)
-

e2(t)

s(t)=a1 + a2 e(t) + a3 e(t)2

m(t)

Elemento
no lineal

s2(t)

El siguiente ejercicio propuesto muestra un modulador prctico del tipo llamado no lineal o
de gran seal.
Ejercicio propuesto

Demostrar que m(t) es una seal modulada en amplitud por x(t) que tiene un ancho de

t nT0
banda a siendo h(t ) 4a sinc(2at ) cos(2f 0 t ) y p (t )
n T0
2

x(t)
+
h(t)
mA
+

m(t)

p(t)

Demodulacin de la DSB con portadora

Por supuesto la demodulacin de AM convencional llamada tambin doble banda lateral


con portadora, puede ser realizada sncronamente igual que la DSB-SC. Es decir
multiplicando por un tono de la misma frecuencia y fase que la portadora y filtrando paso
bajo.
xDs(t)

xDSB-TC(t)=A(1+mx(t)) cos(2fct)

xDc(t)
Filtro
Bloqueo de
paso bajo
continua

xD(t)

cos(2fct)
cos(2fct)
(sincronizado)
Figura 4.15

99

x Ds (t ) (1 mx(t ))
x D (t )

A
1 cos(2 2 f c t ) ;
2

x Dc (t )

A(1 mx(t ))
2

finalmente

Amx(t )
2

Ejemplo

En el diagrama de bloques de la figura siguiente se muestra una manera de obtener un


seal sincronizada con la portadora. Se llama demodulacin homodina.

cos(2fct)

Filtro
paso banda

xDs(t)
xDSB-TC(t)

xDc(t)
Filtro
Bloqueo de
paso bajo
continua

xD(t)

El filtro paso banda muy estrecho est centrado a la frecuencia de la portadora. Este
procedimiento, adems de requerir un vaco espectral en la baja frecuencia de la moduladora,
hace llegar al multiplicador las perturbaciones aadidas a la seal transmitida.
Un diagrama de bloques que proporciona mejores resultados prcticos se muestra
seguidamente.

limitador
Filtro
paso banda

e(t)

Filtro
paso banda

cos(2fct)

xDSB-TC(t)

xDs(t)

xDc(t)
Filtro
Bloqueo de
paso bajo
continua

xD(t)

El primer filtro paso banda recoge una portadora sinusoidal que puede ser ruidosa. El
segundo filtro paso banda recogera el primer armnico de la seal e(t). Por tanto se sintetiza
una portadora libre de perturbaciones.

100

Pero la ventaja principal de esta modulacin consiste en la posibilidad de utilizar un


detector de envolvente como demodulador, que en la practica es muy sencillo de implementar,
se muestra en la figura siguiente.

xDSB-TC(t)

xD'(t)

Figura 4.16
Es un circuito de carga rpida y descarga lenta del condensador.
Envolvente obtenida xD'(t)

Envolvente real xD(t)

3
2
1
0
0

0.2

0.4

0.6

0.8

1.2

1.4

1.6

1.8

2 s

Figura 4.17

Se carga rpidamente al valor de pico de la seal y se descarga lentamente mientras el


diodo no conduce entre pico y pico. El resultado es un seguimiento xD'(t) de la envolvente real
o seal moduladora xD(t).
Los valores de R y C deben ser elegidos de tal manera que la descarga no sea demasiado
rpida, ni demasiado lenta. En el primer caso se producira un rizado demasiado acusado de la
envolvente recuperada y en el segundo caso la descarga no quedara controlada por los picos,
tal como esta a punto de suceder entre los segundos 1.4 y 1.6 de la figura 4.17. Se puede
llegar a la conclusin que

1
1
RC
siendo b el ancho de banda de la moduladora y fc
b
fc

la frecuencia de la portadora. La seal xD'(t) puede ser mejorada filtrando paso bajo para
eliminar las componentes de alta frecuencia generadas.

101

Ntese que si el ndice de modulacin es mayor que 1, como en la figura 4.14, ya no es


posible demodular por envolvente. La seal obtenida en la demodulacin ya no es la misma
que la moduladora.

Ejercicio propuesto

Demostrar que al multiplicar xm(t) por p(t) con T0

1
y despus filtrar paso bajo
fc

convenientemente se obtiene x(t), por tanto el proceso puede servir para simular un detector
de envolvente.
xm (t ) A(1 mx(t )) cos(2 f 0t ) y p (t )

t nT0
)
T0 2

4.2.4 MULTIPLEXADO POR DIVISIN EN FRECUENCIA


Del estudio de la modulacin de amplitud y del concepto de filtrado, se puede inferir que
es posible enviar varias seales simultneamente compartiendo un mismo medio de
comunicacin siempre que sus espectros no se solapen (incluso si se solapan como en QDSB)
y posteriormente recuperar cada una de estas seales. El diagrama de bloques siguiente es ms
explicativo.
xm1(t) Filtro
x1(t)
paso bajo

x1(t)
cos(2f1t)

cos(2f1t)
x2(t)
xm2(t) Filtro
paso bajo

x2(t)
cos(2f2t)

xm(t)
.
.
.
.

.
.
.
.

xn(t)
cos(2fnt)

cos(2f2t)
xmn(t) Filtro
xn(t)
paso bajo
cos(2fnt)

Figura 4.18.

102

Se suman todas las seales de informacin que han sido moduladas (en el diagrama se a
supuesto DSB-SC ) y se transmite esta seal suma xm(t) por un determinado medio de
comunicacin. x m (t ) x1 (t ) cos(2f 1t ) x 2 (t ) cos(2f 2 t ) .......x n (t ) cos(2f n t ) es la seal
multiplexada. Su espectro de amplitud podra ser el que se muestra en la figura 4.19. Se ha
supuesto, cosa habitual, que todas las seales moduladoras x1(t), x2(t), ... xn(t) tienen el
mismo ancho de banda b:
|Xm (f)|
X2(f-f2)/2

X1(f-f1)/2

.........

.........
-fn+b -fn -fn-b

Xn(f-fn)/2

-f2

-f1

f1-b

f1

f2

fn

Figura 4.19. Espectro de amplitud de la seal multiplex.


Para que no se solapen los espectros se eligen las frecuencias de las portadoras con una
separacin entre ellas al menos 2 veces el ancho de banda de la seal moduladora, por tanto
aunque las seales se encuentren mezcladas en el tiempo sus espectros son perfectamente
distinguibles y podrn ser recuperadas

Ejercicio propuesto

Demostrar que la seal de informacin procedente de n sistemas, puede transmitirse


mediante una sola lnea, si las seales de informacin, cuyo ancho de banda mximo es B, se
preparan al principio y recuperan al final como indica el esquema de bloques de la figura
4.18, siendo h(t ) 2 Bsinc(2 Bt ) la respuesta impulsional de los filtros paso bajo.

En alguna ocasin se ha mencionado que el espectro de la seal moduladora o de


informacin se duplica al modular en amplitud. La parte negativa del espectro de la seal de
informacin se hace positiva al modular. Por tanto se tiene una redundancia de informacin en
la seal modulada; cualquiera de las bandas laterales, bien la superior o bien la inferior lleva
la misma informacin. Eliminar una de ellas significara un ahorro del espectro y la
disponibilidad de esta zona frecuencial para ser ocupada por otra seal de informacin.

103

4.2.5 MODULACIN EN BANDA LATERAL NICA


La modulacin en banda lateral nica (single sideband suppressed carrier SSB) sin
portadora se basa en eliminar una de las bandas laterales, superior o inferior, en una
modulacin DSB
Supngase una seal real x(t) cuyo espectro sea como el de la figura siguiente, en el que se
distingue, con fines didcticos, la parte de frecuencias negativas con relleno y la positiva sin
relleno.

X(f)

-b

Figura 4.20
El espectro de la modulacin en doble banda lateral sin portadora es:
X(f+fc)/2

-fc-b

-fc

|XDSB (f)|

X(f-fn)/2

-fc+b

fc-b

fc

fc+b

Figura 4.21
Eliminando mediante filtrado la banda lateral inferior del espectro de la modulacin en
doble banda lateral se obtiene el espectro de la modulacin en banda lateral nica.
|XSSB (f)|

-fc-b

-fc

fc-b

-fc+b

fc

fc+b

Figura 4.22
Su expresin en el tiempo es:
x SSB (t ) x(t ) cos(2f c t ) hx (t ) sen(2f c t )

(4.11)

siendo hx(t) la transformada de Hilbert de la seal x(t)


Si se hubiera seleccionada la banda lateral inferior la expresin en el tiempo sera:
x SSB (t ) x(t ) cos(2f c t ) hx (t ) sen(2f c t )

(4.12)
104

Para llegar a estas expresiones se debe hacer uso del concepto de seal analtica y de la
transformada de Hilbert, apartado 3.6.3. Recurdese que la transformada de Hilbert hx(t) de
una seal x(t) es:
hx (t ) x(t )

1
t

Y la expresin de la seal analtica es:


a x (t ) x(t ) jhx (t )
Como ya es sabido, esta ltima expresin tiene la particularidad de tener, cuando j es
positivo, un espectro con valor cero para frecuencias negativas e igual al de la seal x(t) para
las frecuencias positivas. Si el signo de j es negativo la situacin es la inversa.
As la transformada de Fourier de a x (t ) x (t ) jhx (t ) es Ax ( f ) 2 X ( f )u( f ) y la
transformada de Fourier de a x (t ) x (t ) jhx (t ) es Ax ( f ) 2 X ( f )u( f ) . Grficamente es:
2X(f)u(f)

-b

2X(f)u(-f)

-b

a)

b)

Figura 4.23. a) Espectro de x(t ) jhx (t ) , b) espectro de x(t ) jhx (t )


No cabe duda que 2 X ( f ) Ax ( f ) Ax ( f ) y que 2 x (t ) a x (t ) a x (t ) . Ahora el
propsito es bien sencillo. Desplazar el espectro de la figura 4.23 a) hacia frecuencias
positivas una cantidad igual a fc (frecuencia de la portadora) y desplazar el espectro de la
figura 4.23 b) hacia frecuencias negativas tambin una cantidad fc. Despus sumar estos
espectros para que quede como en la figura. Esto es 2 X BLU ( f ) Ax ( f f 0 ) Ax ( f f 0 ) .
2 |XSSB (f)|

-fc-b

-fc

fc-b

-fc+b

fc

fc+b

Figura 4.24

105

Lo dicho es equivalente en el dominio del tiempo a:


2 xBLU (t ) x (t ) jhx (t ) e j 2 fct x (t ) jhx (t ) e j 2 fct

(4.13)

e j 2 f c t e j 2 f c t
e j 2 f c t e j 2 f c t
jh
t
j

2 xBLU (t ) 2 x (t )
(
)
2
x

2
2j

(4.14)

Y como:

Entonces:
x SSB (t ) x(t ) cos(2f c t ) hx (t ) sen(2f c t )
Es evidente que la modulacin en banda lateral nica es una modulacin en cuadratura,
solo que en este caso las seales de informacin, sus componentes, estn relacionadas.
Tambin se suelen llamar a x(t) componente en fase y hx(t) componente en cuadratura. Estos
nombres no son accidentales, tienen que ver con la realidad de estas seales. Recurdese que
hacer la transformada de Hilbert de una seal x(t) es retardar noventa grados todas las
componentes de frecuencia de la seal x(t). Realizar un dispositivo que haga esto presenta
dificultades y nicamente se pueden lograr aproximaciones.
Tambin se presentan dificultades al intentar eliminar mediante filtrado cualquiera de las
bandas, porque como los filtro no son ideales siempre quedaran residuos de la banda que se
pretende eliminar. La modulacin SSB se adapta bien a seales que no tienen componentes
espectrales en la banda baja, como es la seal de voz.

Demodulacin

Para demodular la seal SSB y obtener x(t) es preciso reconstruir el espectro de x(t). Con
un demodulador para DSB es suficiente.
x(t) cos2(2fct) hx(t) sin(2fct) cos(2fct)
xSSB(t)=x(t) cos(2fct) hx(t) sin(2fct)

Filtro
paso bajo

x(t)/2

Oscilador sncrono
cos(2fct)
Figura 4.25

106

Igual que con las seales DSB se puede aadir portadora a la seal banda lateral nica para
ser SSB-TC. Si la portadora aadida tiene una amplitud suficientemente grande, se puede
recuperar la seal de informacin mediante deteccin por envolvente.
Supngase que a la seal SSB se ha aadido una portadora de amplitud A:
x SSB TC (t ) A cos(2f c t ) x(t ) cos(2f c t ) hx (t ) sen(2f c t )

(4.15)

La envolvente de la expresin anterior es:


e(t )

A x(t )2 hx (t )2

e(t ) A 1

2 x(t ) x(t ) 2 hx (t ) 2
2
A
A
A2

Si la portadora es mucho mayor que |x(t)| entonces:


e(t ) A 1

2 x(t )
x(t )
A(1
) A x(t )
A
A

Se puede obtener x(t) de forma correcta detectando por envolvente.


Las seales se podran recibir en un receptor de AM comn y se requerira la mitad del
ancho de banda de la transmisin presente en transmisiones de AM.
MODULACIN EN BANDA LATERAL RESIDUAL
Se ha mencionado que en la modulacin SSB se presentan dificultades al intentar eliminar
mediante filtrado cualquiera de las bandas, porque al ser los filtros no ideales quedan residuos
de la banda que se pretende eliminar. La modulacin en banda lateral residual (vestigial
sideband VSB) se basa precisamente en no intentar eliminar este residuo, simplemente hacer
que sea de tal manera que no produzca distorsin al demodular. Intentemos fijar los conceptos
de forma intuitiva. Supngase que se dispone de un filtro paso alto o paso banda con un
flanco poco abrupto en su espectro H(f) y con el se pretende obtener la banda lateral superior
de una modulacin en doble banda lateral. Esto se muestra en la figura siguiente.
H (f)
XDSB(f)

-fc

-fc+r

Banda lateral
inferior

fc-r

Banda lateral
superior

fc

Figura 4.26

107

Despus de filtrar con H(f) se obtendr un espectro semejante a lo que se muestra en la


figura siguiente.
XVSB (f)

-fc-b

-fc

Banda lateral
superior

Residuo de la banda
lateral inferior

-fc+r

fc

fc-r

fc-b

Figura 4.27
Que supuestamente sera el espectro de la seal modulada en banda lateral vestigial (o
residual). Sin embargo, al demodular, es decir al trasladar las bandas del espectro hacia la
parte baja de frecuencias se obtendr un espectro semejante a lo que se muestra en la figura
siguiente.
XVSB (f)

Espectro recuperado
Espectro original

-2fc -2fc+r

-b

2fc-r

2fc

Figura 4.28
Es evidente que el espectro recuperado no ser el original a menos que las colas de las
bandas laterales se sumen de tal manera que lo reconstruyan. Esto sucede cuando el filtro
utilizado para eliminar la banda lateral es de fase lineal y cumple |H(fc)|=1/2
H (f)

H (fc)=1/2
1

-fc-r

-fc

-fc+r

fc-r

fc fc+r

Figura 4.29
La expresin de este tipo de modulacin para la banda lateral superior es:

108

xVSB (t ) x(t ) cos(2f c t ) hx (t ) 2 jx(t ) d (t )sen(2f c t )


Siendo d(t) la transformada inversa de Fourier de D(f).
D(f) se obtiene como se muestra en la figura siguiente:
D(f)

Filtro ideal
u(f-fc)

Filtro real

1/2

1/2
fc-r

fc

Diferencia entre
real e ideal

-1/2
fc+r

-r

Figura 4.30
La demodulacin ser sncrona, pero si se aade una portadora de suficiente amplitud se
pueden hacer las mismas consideraciones que para SSB.
La modulacin en banda lateral residual se usa en los sistemas de televisin pblica porque
la seal compuesta de vdeo, con un ancho de banda de 6 MHz, requerira 12 MHz en
transmisin. Adems tiene componentes apreciables en las frecuencias bajas y no sera
posible filtrar adecuadamente para modular en banda lateral nica.

109

4.3 MODULACIONES ANALGICAS ANGULARES

Las modulaciones angulares se caracterizan porque la informacin se encuentra en la fase


de la seal modulada. Las seales moduladas angularmente se obtienen modificando la fase
de una portadora. Esto es :
x p (t ) A cos( i (t ))

(4.16)

Dentro de todas las posibles leyes que pueden relacionar la fase instantnea i(t) con una
seal de informacin x(t) existen dos de especial inters.
- Modulacin de fase (PM): la fase instantnea varia proporcionalmente con la informacin
x(t).

PM i (t ) 2f c t d x(t )

(4.17)

Con lo que la seal modulada ser:


x PM (t ) A cos(2f c t d x(t ))

(4.18)

d se llama constante de desviacin de fase. Para que no haya ambigedad debe ser

d x(t ) max . La frecuencia instantnea fi de esta fase instantnea se puede obtener como la
derivada de PMi(t) con respecto al tiempo y dividido por 2.

dx(t )
1 d PM i (t )
f PMi f c d
2
2 dt
dt
- Modulacin de frecuencia (FM): la frecuencia instantnea varia proporcionalmente con la
informacin x(t).
f FMi f c f d x(t )

(4.19)

fd es la constante de desviacin de frecuencia. No es necesaria una restriccin parecida a la


de desviacin de fase; siempre se puede distinguir entre dos frecuencias.
Para determinar la fase instantnea se integra la frecuencia instantnea y se multiplica por
2.
t

FM i (t ) 2 f c dt ' 2f d x(t ' ) dt '


t

FM i (t ) 2f c t 2f d x(t ' ) dt '


0

Finalmente la seal modulada en FM ser:

110

x FM i (t ) A cos 2f c t 2f d x(t ' ) dt '


0

(4.20)

Ejemplo

Modular en FM y en PM la seal x(t ) sin(2 1 t ) con constante de desviacin de


frecuencia de 3 Hz y constante de desviacin de fase de 3 radianes. La portadora ser de
amplitud 1, y frecuencia fc de 10 Hz.
La frecuencia instantnea de la seal modulada en FM ser:
f FMi 10 3 sin(2 1 t )
Su fase instantnea ser:
t

FM i (t ) 2 f c dt ' 2 f d x(t ') dt ' 2 10 t 3 3 cos(2 1 t )


Y la seal modulada en FM:
xFM i (t ) cos 2 10 t 3 3 cos(2 1 t )

En la figura anterior se pueden apreciar 2 segundos de esta modulacin. En lnea


discontinua, a modo de comparacin, se muestra la seal moduladora x(t).
La fase instantnea de la seal modulada en PM ser:

PM i (t ) 2 10 t 3 sin(2 1 t )
La seal modulada en PM:
x PM i (t ) cos2 10 t 3 sin(2 1 t )
111

Igual que en FM, en la figura siguiente se aprecian 2 segundos de esta modulacin PM. En
lnea discontinua se muestra la seal moduladora x(t).

La frecuencia instantnea de esta modulacin en PM es:


f PM i (t ) 10 3 cos(2 1 t )
Obsrvese la similaridad entre las dos modulaciones salvo un desfase.

Hay una relacin entre FM y PM:


x(t)

x(t)

Modulador
de PM

.....dt

d.../dt

Modulador
de FM

xFM(t)

xPM(t)

Se puede usar un modulador de fase para obtener modulacin de frecuencia o bien usar un
modulador de frecuencia para obtener una modulacin de fase.
Como las modulaciones de frecuencia y de fase estn muy relacionadas, cualquier
variacin de fase producir una variacin de frecuencia y viceversa. Que estas modulaciones
no son lineales, puede comprobarse en el razonamiento siguiente.
En general una modulacin angular xm(t) con moduladora x(t) puede expresarse de la
forma:

x m (t ) Re al Ae ji ( t ) Re al Ae j 2f ct e jkx ( t )

112

Pues

bien,

1 jkx(t )

tomando

el

trmino

e jkx (t )

desarrollndolo

en

serie

1 2
1
k x(t ) 2 j k 3 x(t ) 3 ......... , por tanto xm(t) puede escribirse como:
2!
3!

1
1

xm (t ) Re al Ae j 2 fct 1 jkx(t ) k 2 x(t ) 2 j k 3 x(t )3 .........


2!
3!

(4.21)

En la expresin anterior se puede observar que a menos que kx(t ) 1 ste tipo de
modulacin no es lineal. Por tanto parece lgico que las bandas laterales de la modulacin
angular no cumplan el principio de superposicin, as el anlisis espectral debera hacerse, si
es posible, para la seal moduladora que se elija.
De la expresin anterior es destacable tambin que si la desviacin de frecuencia o de fase
es lo suficientemente pequea la modulacin angular puede asociarse o tratarse como una
modulacin de amplitud. Esto se ver en lo que, mas adelante, llamaremos modulacin de FM
de banda estrecha.

4.3.1 ANLISIS ESPECTRAL


Ante todo debe decirse que, a diferencia de una seal modulada en amplitud, la
transformada de Fourier de una seal de modulacin angular en general no puede evaluarse.
En algunos casos la operacin puede realizarse numricamente o bien por aproximaciones.
Por ello conviene exponer algunas ideas de principio.
Hay dos mecanismos identificables en la comprensin del espectro de una seal modulada
angularmente. Uno se debe a la velocidad de cambio de la seal moduladora, que es
semejante a hablar de su contenido frecuencial, totalmente unido al concepto de ancho de
banda. El otro se debe a la amplitud de la seal moduladora que influye en la frecuencia
instantnea de la portadora. Por ejemplo, cuando la desviacin de frecuencia es pequea
predomina el primer mecanismo y podra adelantarse que el ancho de banda de la seal
modulada sera semejante al de una modulacin de amplitud, es decir 2 veces el ancho de
banda de la moduladora. Cuando la desviacin de frecuencia no es pequea, comparada con el
ancho de banda de la moduladora, predomina el segundo mecanismo y podra decirse que el
ancho de banda de la seal modulada sera, aproximadamente, 2 veces la desviacin de
frecuencia de la modulacin.
Como ya se ha mencionado no es posible evaluar la transformada de Fourier de una seal
FM en general. Se realizar un anlisis para una seal x(t)= a cos(2fmt) que se modular en
113

FM. Las ideas expuestas y este anlisis permitirn extraer algunas conclusiones relativas al
ancho de banda de la modulacin angular.
La frecuencia instantnea es:
f i f c f d a cos(2f m t ) f c f cos(2f m t )

con f f d a

La fase instantnea se obtendr integrando fi y multiplicando por 2.

i 2f c t

f
f
sen(2f m t ) siendo
el llamado ndice de modulacin.
fm
fm

La seal modulada en frecuencia es x FM (t ) A cos2f c t sen(2f m t ) , pero por facilidad


de operacin se escribir en forma compleja. Esto es:

x FM (t ) Re al Ae j 2f ct e j sen( 2f mt )

(4.22)

La segunda exponencial es una funcin peridica del tiempo con frecuencia fundamental fm
Hz y en consecuencia puede desarrollarse en serie de Fourier.
e

j sen( 2f m t )

j 2nf m t

1
con d n
Tm

Tm

T
m

j sen( 2f m t )

e j 2nf mt dt

Haciendo el cambio de variable 2f m t se obtiene: d n

1
2

j sen( ) n

Esta integral puede evaluarse en trminos de los parmetros n (entero)y (continuo) y


adems est tabulada. Jn() se llama funcin de Bessel de primera clase de orden n y
argumento .

Figura 4.31. Funciones de Bessel


Por tanto la expresin de la seal de FM se puede escribir como:

x FM (t ) Re al Ae j 2f ct J n ( )e j 2nf mt )
n

114

x FM (t ) A J n ( ) cos2 f c nf m t

y finalmente

De este resultado es evidente que una seal en FM con modulacin senoidal tiene un ancho
de banda infinito, puesto que est formado por infinitos tonos mltiplos de fm pero con unas
amplitudes que decrecen a medida que se separan de la frecuencia de la portadora.
XFM (f)

.......

.......

.......

.......

-fc

Hz

fc

Figura 4.32

Entonces, a efectos prcticos, el ancho de banda puede considerarse finito. Ahora habra
que decidir donde cortar.

Ejemplo

Con una moduladora x(t ) cos(2 1 t ) , es decir con un tono de fm =1 Hz, representar el
espectro de la seal modulada en FM con constantes de desviacin de frecuencia fd 1, 3 y 5.
La portadora ser de amplitud 1, y frecuencia fc de 10 Hz.
La frecuencia instantnea de la seal modulada en FM ser:
f FMi 10 f d 1cos(2 1 t ) con f f d 1
Su fase instantnea ser:
t

FM i (t ) 2 f c dt ' 2f x(t ' ) dt ' 2 10 t f sen(2 1 t )

700
600
500
400
300
200
100
0
2

10

2f

12

14

16

18
Hz

20

115

x FM i (t ) cos2 10 t f sen(2 1 t )

Y la seal modulada en FM:

Los espectros de las grficas corresponden a 20 segundos de esta seal modulada en FM


Para fd=1, en la grfica anterior corresponde a un ndice de modulacin = f/ fm= 1.
La siguiente grfica, para fd=3, correspondiente a una = f / fm= 3:

450
400
350
300
250
200
150
100
50
0
0

10

2f

12

14

16

18
Hz

20

Para fd=5 correspondiente a una = f / fm= 5:

350
300
250
200
150
100
50
0
0

10

12

2f

14

16

18
Hz

20

La cantidad de banda necesaria depender de los requisitos de calidad o fidelidad en la


transmisin. Pero una regla usada, que fue propuesta por J.R. Carson es:
BWFM 2(f f m )

En general, para una modulacin en FM de una seal x(t) con ancho de banda b la
expresin anterior es:
BWFM 2(f b)

(4.23)
116

Para la modulacin de fase se puede obtener una expresin semejante. Suponiendo que la
seal moduladora x(t) tenga una amplitud mxima Am, la desviacin mxima de fase sera

=d Am, esto es:


PM i (t ) 2f c t d Am sen(2f m t )
PM i (t ) 2f c t sen(2f m t )
Su frecuencia instantnea ser:
1 d PM i (t )
f PMi f c f m cos(2f m t )
dt
2

Asociando ahora la formula de Carson se obtiene:


BWPM 2( f m f m ) 2( 1) f m
Si la seal x(t) tiene un ancho de banda b, el ancho de banda para una modulacin de fase
puede escribirse como:
BWPM 2( 1)b

(4.24)

4.3.2 MODULADORES DE FM Y DE PM
Tericamente puede utilizarse un modulador de FM en lugar de PM con solo derivar
previamente la seal moduladora y al revs utilizar un modulador de PM en lugar de FM
integrando la seal moduladora.
Se pueden dividir los moduladores en tres grupos:
- Mtodos directos. Tpicamente el oscilador controlado por tensin (VCO). Obtienen la
modulacin angular variando la frecuencia instantnea de algn dispositivo.
- Generacin de PM o FM de banda estrecha y posterior multiplicacin de frecuencia.
- Generacin de seales de FM por mtodos no lineales. Se refiere la obtencin de una
forma de onda de FM con portadora cuadrada o triangular; posteriormente se genera una
portadora sinusoidal por filtrado.
No se har hincapi en los mtodos directos y no lineales, pero s en el de generacin de
FM o PM de banda estrecha por su inters conceptual.

117

MODULACIN DE PM DE BANDA ESTRECHA


4.3.2.1
En la seal modulada en PM, el valor absoluto mximo de la constante de desviacin de
fase d, suponiendo x(t) normalizada a uno, est limitada a . Para valores de d << se
pueden hacer en la expresin de la modulacin de PM ciertas aproximaciones.
x PM (t ) A cos(2f c t d x(t )) A cos(2f c t ) cos( d x(t )) A sen(2f c t ) sen( d x(t ))
Como que d << se puede aproximar cos( d x(t )) a 1 y sen( d x(t )) por d x(t ) . As:
x PM (t ) A cos(2f c t ) A d x(t ) sen(2f c t )

(4.25)

Su espectro sera muy similar a la de una modulacin lineal en AM. Un diagrama de


bloques que genera sta modulacin se muestra en la figura siguiente.
-

d x(t)

xPM(t)
+
/2

Asin(2fct)
Figura 4.33
Si la seal x(t) fuese integrada a la entrada, se estara generando una modulacin de FM de
banda estrecha.

Validez

Se calcular ahora la condicin que debe cumplir d

para que pueda realizarse la

aproximacin de banda estrecha. La expresin anterior puede ponerse de forma:


y (t ) A1 cos(2 f c t ) A2 sen(2 f c t )

(4.26)

Donde A1 A y A2=-A d x(t). Para que y(t) fuese una modulacin en PM debera ser de la
forma: y ' (t ) C cos(2f c t d x(t )) siendo C una constante. Pero la expresin y(t) puede
agruparse y expresarse como:
y (t )

A
A12 A2 2 cos 2 f c t arctg( 2 )
A1

y (t ) A 1 d x(t ) 2 cos2f c t arctg( d x(t ))


2

118

Teniendo en cuenta que arctg( x) x

x3 x5


3
5

se puede aproximar la desviacin

de fase arctg( d x(t )) d x(t ) con menos de 1% de error (

arctg d x(t ) d x(t )


100 ) para
arctg d x(t )

valores de d menores que 0.13 radianes (7.45). La ligera modulacin en amplitud de la onda
puede eliminarse con un elemento recortador y despus filtrar paso banda para eliminar los
armnicos indeseables.
La figura siguiente muestra el diagrama de bloques de un modulador basado en esta
modalidad.

x(t)

Modulador
de PM B.E.

.....dt
0

xFM(t)

Cadena de
multiplicacin
de frecuencia

Modulador de FM de banda
estrecha

Amp.

Oscilador
foL

Figura 4.34
Antes de la modulacin la seal de informacin se integra; de este modo la salida estar
modulada en FM. Un modulador de FM de banda estrecha produce una desviacin de
frecuencia muy pequea, comparada con la frecuencia de la portadora; por ello es necesario
que esta sea tambin pequea para que la informacin no se confunda con las propias
inestabilidades del oscilador. Es necesario, por tanto, incrementar la frecuencia de la
portadora fc y la razn de desviacin fd / fc. Esto se consigue con multiplicadores de frecuencia
y la multiplicacin, al final, por una funcin sinusoidal de la frecuencia adecuada.
Un multiplicador de frecuencia opera sobre la frecuencia instantnea, de modo que si la
entrada tiene una frecuencia instantnea fi(t) = fc1 + fd1 x(t) a la salida despus de multiplicar
por n habr una frecuencia instantnea nfi(t) = nfc1 + nfd1 x(t).
La figura siguiente muestra el diagrama de bloques de un multiplicador de frecuencia por
n.
xFMBE(t)

Limitador
amp. salida
amp. entrada

xR(t)

Fil. Paso banda


centrado a nfc1

Amp.

xFMnfc1 (t)

119

Para realizar fsicamente un dispositivo de este tipo, simplemente pinsese en una seal
sinusoidal, con frecuencia instantnea fundamental f (t )i f c1 f d 1 x(t ) , a la que se recorta
su amplitud para generar armnicos a frecuencias multiplos de la fundamental. Esta seal
cuasi peridica se puede expresar en serie de Fourier y estar formada por infinitos armnicos
mltiplos de la frecuencia instantnea fundamental.
t

xR (t ) a1sen(2 f c1 t 2 f d 1 x(t ')dt ') a2 sen(2 2 f c1 t 2 2 f d 1 x(t ')dt ') ......an sen(2 nf c1 t 2 n f d 1 x(t ') dt ') ...

Por tanto solo habra que filtrar paso banda con el ancho adecuado en el armnico que se
desea recoger y amplificarlo convenientemente. Como se trata de una seal sinusoidal cuya
frecuencia varia al ritmo de una moduladora, cada uno de los armnicos tambin variaran al
ritmo de esta moduladora con lo que se dispone de una modulacin de FM en cada armnico.
Esta multiplicacin afecta tanto a la desviacin de frecuencia como a la frecuencia de la
portadora y no vara el cociente fd / fc.

Para aumentarlo ser necesario un batido o

multiplicacin con un tono.

Ejemplo

Con un modulador de FM de banda estrecha se obtiene una seal xFMBE(t) con una
constante de desviacin de frecuencia fd1=25 Hz y una portadora fc1 = 200 kHz. Sin embargo
se desea una constante de desviacin de frecuencia fd =75 kHz y una emisin en la banda
comercial de 88 a 108 MHz.
xFMBE(t)

Cadena de
multiplicacin
de frecuencia

xFMs(t)

xFM(t)

xFMb(t)
Amp.

Oscilador
foL
Segn las especificaciones se dispone a la salida del modulador de banda estrecha de una
frecuencia instantnea tal como f i 200 10 3 25 x(t ) , por tanto se necesita un factor de
multiplicacin n=75 103/25= 3000, que podra realizarse con una cadena formada por 3
multiplicadores por 5, 3 multiplicadores por 2 y 1 multiplicador por 3. As la frecuencia
instantnea en la seal xFMs(t) despus de la multiplicacin ser:
nf i 200 10 3 3000 25 3000 x(t ) 600 10 6 75000 x(t )

120

y la seal x FMs (t ) A cos 2 600 10 6 t 2 75000 x(t ' ) dt '


0

Se ve que la frecuencia de la portadora es ahora 600 MHz, totalmente fuera del margen de
los 88 a 108 MHz fijados.
La seal xFMb(t) = xFMs(t) cos(2foLt) puede desarrollarse trigonomtricamente para
comprobar que est formada por un coseno de la suma de sus argumentos mas otro coseno de
la diferencia de sus argumentos. Si el amplificador final es adems un fitro paso banda que
elimina el coseno de la suma de argumentos, entonces con un oscilador regulable entre 688 y
708 MHz se podra obtener la seal modulada en la banda de frecuencias especificada.

4.3.3 DEMODULACIN DE FM Y PM
La frecuencia (fase) instantnea de una seal modulada en FM (PM) varia linealmente con
la seal moduladora. Por tanto el procedimiento de recuperacin deber basarse en un
sistema que produzca una seal de salida proporcional a las variaciones de la frecuencia (fase)
de la entrada.
En general para demodular una seal modulada angularmente el mejor procedimiento es
derivarla. As para x p (t ) A cos( i (t )) su derivada es:
dx p (t )
dt

di (t ) d cos(i (t ))
d (t )

A i sen(i (t ))
dt
di (t )
dt

Caso PM:
Su fase instantnea es PM i (t ) 2f c t d x(t )
d PM i (t )
dx(t )
2f c d
Por tanto
dt
dt

dx PM (t )
dx(t )

A 2f c d
sen 2f c t d x(t )
dt
dt

(4.27)

Caso FM:
t

Su fase instantnea es FM i (t ) 2f c t 2f d x(t ' ) dt '


0

d FM i (t )
2f c 2f d x(t )
dt

121

dx FM (t )
A2 f c f d x(t ) sen 2f c t 2f d x(t ' ) dt '
dt
0

f d x(t )
dx FM (t )

A2f c 1

sen
2

f
t
2

f
x
(
t
'
)
dt
'
c
d

f
dt
c

(4.28)

En ambos casos la informacin de la fase se ha trasladado a la amplitud, siendo claramente


una modulacin de amplitud con portadora (si bien esta portadora tambin est modulada)
Con un detector de envolvente a la salida del derivador y despus un bloqueo de continua
se recupera a x(t) en FM y su derivada en PM. En este ltimo caso sera necesaria una ltima
integracin.
Existen tres mtodos que pueden permitir implementar esta derivada:
- Derivacin en el tiempo.
- Derivacin en el dominio de la frecuencia.
- Derivacin por retardo.
El primer mtodo consiste en utilizar un sistema que haga la operacin de derivacin
exactamente.

Ejemplo

Un derivador prctico.
Vcc

dx (t )
En donde I (t ) C FM
dt

I(t)

y la salida
V (t ) Vcc I (t ) R Vcc RC

dx FM (t )
dt

I(t)
C

I(t)

V(t)

xFM(t)

El segundo mtodo se basa en que idealmente un derivador es un dispositivo lineal cuya


funcin de transferencia tendr una pendiente lineal sobre la banda de frecuencias ocupada

122

por la seal de entrada modulada angularmente. Convierten variaciones de frecuencia


instantnea de la seal de entrada en variaciones de amplitud de la seal de salida.

Ejemplo

En el diagrama de bloques de la figura, la salida de H(f), que puede ser un filtro paso
banda, ser una seal modulada en amplitud xFMAM(t) con portadora, dado que acta como un
derivador.

xFM(t)

H(f)

xFMAM(t)

Detector de
envolvente

x(t)

La explicacin del proceso de demodulacin es el siguiente:


Si la frecuencia de la portadora de la modulacin angular es fc y el ancho de banda de la
seal modulada es 2a, entonces con una funcin de transferencia del sistema H(f) como se
muestra en la figura, se obtendra a la salida una seal modulada en amplitud proporcional a la
seal moduladora, en el supuesto que el flanco del espectro de H(f) fuese lineal.
H (f)

-fc-a -fc

-fc+a

fc-a

fc

fc+a

El tercer mtodo se basa en un dispositivo lineal que realiza una aproximacin a la


derivada.
dx FM (t ) x FM (t ) x FM (t T0 )

dt
T0

123

A la seal xFM(t) se le resta la misma seal ligeramente retardada y se divide la diferencia


por el valor del retardo. El retardo debe ser reducido para que la aproximacin sea buena.
Un diagrama de bloques se muestra en la figura siguiente.

xFM(t)

1/T0

d xFM(t)/dt

T0

Figura 4.35
Este mtodo es muy til para demoduladores de FM de muy altas frecuencias (microondas
y laser) porque puede obtenerse la diferencia a travs de caminos de distintas longitudes. Para
frecuencias mas bajas el retardo puede obtenerse con un circuito con una funcin de
transferencia de amplitud constante y fase lineal en toda la banda de trabajo. La fase debe ser
tal que su derivada con respecto a la frecuencia sea 2T0.

Ejemplo

El llamado demodulador por conteo de pulsos utiliza el concepto de derivacin por retardo.
En la figura siguiente se muestra su diagrama de bloques.

xFMl(t)
xFM(t)
A

xFMr(t)

xFMd(t)

Rectificador de
media a onda

+
-

-B
T0

Filtro paso
bajo

x(t)

xFM2(t)

La seal de FM se amplifica fuertemente y despus es recortada por el limitador para


producir una seal de FM de onda cuadrada xFM1(t). Vase la secuencia de grficas que
describe el proceso.

124

xFM1(t)
trazo continuo

xFM2(t)
trazo discontinuo

t
xFMd(t)
t
xFMr(t)
t
x(t)
t
La seal diferencia xFMd(t) est formada por una secuencia de pulsos cuya posicin
depende linealmente de la seal moduladora. El rectificador prepara esta secuencia para ser
filtrada paso bajo y obtener el mensaje x(t).

4.3.4 MTODOS DE EXTENSIN DE UMBRAL.


Es evidente que la seal de FM, a la entrada de cualquier demodulador, estar acompaada
de perturbacin. Est demostrado terica y prcticamente (ver Carlson) que si la relacin
potencia de seal a potencia de perturbacin no supera un cierto valor (umbral), que depende
en parte de ese demodulador, no es posible recuperar la informacin con ese demodulador.
Existen unos demoduladores de FM mucho ms sofisticados que permiten recuperar la
informacin de seales recibidas que presentan relacin potencia de seal a potencia de
perturbacin menores que los demoduladores vistos hasta ahora. Estos circuitos presentan la
caracterstica de permitir umbrales mucho ms pequeos; de ah la expresin de "extensin de
umbral".
Todos los circuitos que presentan esta caracterstica son mucho ms complejos que los
discutidos previamente". Sin embargo ya que todos los elementos individuales en cada
diagrama de bloque han sido ya estudiados, el comportamiento global no ser difcil de
comprender.

125

El primer mtodo de extensin de umbral es el receptor de FM con realimentacin. Se trata


de un receptor superheterodino, como puede apreciarse en la figura siguiente, en el que se ha
reemplazado el oscilador local por un VCO cuya frecuencia instantnea est controlada por la
seal demodulada. Cualitativamente la salida del VCO ser una seal de FM muy parecida a
la de la entrada, excepto que la frecuencia de la portadora ser fc-fFI , siendo fFI la frecuencia
intermedia, que corresponde a la frecuencia central del filtro HFI(f).

xFM(t)

HFI(f)

y(t)

Demodulador
de FM

Filtro paso
bajo

xD(t)

VCO
Figura 4.36
La operacin de multiplicacin (heterodinacin) nos dar una seal de FM con portadora
fFI Y una desviacin de fase que ser la diferencia de desviaciones de fase de la seal de
entrada del receptor y de la procedente del VCO, adems de otras seales que son bloqueadas
par el filtro HFI(f). De esta forma la desviacin efectiva de frecuencia a la entrada de HFI(f) es
reducida, por lo que la anchura de banda de este filtro puede ser mucho ms pequea que la de
transmisin de F.M.. Si esto es as, la potencia de perturbacin a la entrada del demodulador
es mucho menor, y de aqu que sea posible demodular seales que por el efecto umbral no
podran recuperarse con los procedimientos vistos anteriormente.
En la figura anterior, la salida del VCO es igual a 2 cos2 f c f FI t k D (t ) con
t

D (t ) 2 x D (t ' )dt '


0

En ausencia de perturbacin la seal de FM recibida es:


t

x FM (t ) A cos2f c t FM (t ) con FM (t ) 2f d x(t ' )dt '


0

Por tanto suponiendo que HFI(f) elimine trminos de suma de frecuencias de la salida del
multiplicador, la entrada al demodulador ser la seal diferencia de frecuencias:
y (t ) A cos2f FI t FM (t ) k D (t )

As:

126

x D (t )

1 d
FM (t ) k D (t ) f d x(t ) kx D (t )
2 dt

y despejando xD(t) de la anterior ecuacin:


x D (t )

fd
x(t )
1 k

de esta forma la desviacin de frecuencia ha sido reducida par el factor 1/(1+k) debido al
efecto de la realimentacin. Y esta reduccin de la desviacin permite que la salida del
multiplicador tenga una seal con desviacin de fase FM (t ) k D (t )

FM (t )
con lo
1 k

que la anchura de banda del filtro, MFI(f) puede ser mucho menor que la de transmisin de
FM (pero siempre mayor que 2b) eligiendo k adecuadamente.
Un circuito muy parecido al anterior, pero sin el discriminador de FM, es el "Phase Looked
Loop" (P.L.L.) cuyo diagrama de bloques se muestra en la figura siguiente.
xFM(t)

y1(t)

y2(t)

Filtro paso
bajo

xD(t)

VCO
Figura 4.37

En l se hace funcionar al VCO como un generador de FM. que tiende a dar una seal
igual a la de entrada del P.L.L. Lgicamente, para que esto ocurra, a la salida del VCO debe,
haber la frecuencia instantnea de la seal de entrada. El filtro paso bajo se utiliza para
eliminar las frecuencias indeseables del producto de la seal de FM de entrada con la
procedente del VCO. El problema del P.L.L. es que se precisa de un enganche inicial; es decir
se necesitan unas ciertas condiciones iniciales para que el sistema P.L.L. empiece a funcionar,
siendo necesario un tiempo para que se llegue al funcionamiento adecuado.

127

PROBLEMAS PROPUESTOS

4.1.- El esquema de bloques de la figura representa un modulador de amplitud del tipo


balanceado, llamado as porque su configuracin permite eliminar trminos indeseables de la
modulacin.
e1(t)
s1(t)
+
Elemento
no lineal
+
y(t)
x(t)
+
cos(2fct)
2

s(t)=a e(t)

e2(t)

Elemento
no lineal

s2(t)

a) Cual es la expresin de y(t) ?.


b) Si X ( f ) ( f

) , calcular el espectro de Y(f) y hacer una grfica.

c) Dibujar un esquema de bloques para recuperar x(t) (si utiliza algn filtro, debe indicar su
ancho de banda)
4.2.- Cual debe ser la frecuencia mxima del espectro de x(t) para que al atravesar el
sistema pueda ser recuperada ntegramente?
Cual debe ser el valor del amplificador final para que la seal recuperada y(t) sea igual a
x(t)?.
x(t)
> A

> x
^

>

cos 2f0 t

h(t)
1
>

> x>
^

h(t)
2

> B

y(t)
>

Datos:
h1 (t ) sinc(t ) cos(2f 0 t )

h2 (t ) sinc (t )

4.3.- Una seal de control conteniendo informacin acerca del estado de un sistema, debe
hacerse llegar a un controlador para que ste decida la accin a tomar.
El entorno del sistema es muy ruidoso, para ello debe trasladarse la banda de la seal a una
zona del espectro menos ruidosa, disendose para ello el sistema de la figura.
128

x(t)

e(t)

elemento
no lineal

+
cos(2f0t)

s(t)

y(t)

h(t)

a) Demuestre que x(t) a la salida del sistema ha sido trasladada de banda


Datos:

, h(t ) 2 Bsinc(2 Bt ) e j 2f 0t e j 2f 0t

s (t ) 1 a e(t ) a 2 e(t ) 2

El ancho de banda de x(t) es menor o igual a B


4.4.- Demostrar que y(t) es una seal modulada en doble banda lateral, siendo el ancho de
banda de x(t) menor o igual a B.

x(t) > x
^

>

y(t)
>

h(t)

t nT0
con T0 10

Datos:
h(t ) 2 Bsinc(2 Bt )(e j 2f 0t e j 2f 0t ) ,

f0

1
T0

4.5.- Cual es la expresin de S(t)


x(t)
> m

> +

>

>

1v
x
>
^

x >+
^

>

<
cos 2 f0 t

S(t)
>

1
v
2

Es algn tipo de modulacin de x(t) que conozca?

129

4.6.- Del diagrama de bloques de la figura


x(t)
>

filtro ?

> x >
^

filtro

>

paso bajo

cos 2f0 t

> x>
^
^
h(t)
+

cuantificador

>

codificador

>

memoria

y(t)
Con:

h(t )

y (t ) 1 z (t ) cos(2 7 10 3 t ) ,

(t nT ) ,

sen(2 2 103 t )
cos(2f 0t ) .
x(t ) 4 103
t

Y con el espectro de la seal z(t) dado por :


Z(f)
1

3
- 10 Hz.

espectro de z(t)

3
10 Hz.

Se pide:
a) El primer bloque del diagrama Que clase de filtro y cual debe ser su ancho de banda
mnimo para que no deteriore la seal x(t) ?
b) Dibujar lo mas exactamente posible el espectro de la seal x(t) en el punto a.
c) Que ancho de banda mnimo debe tener el filtro paso bajo para recuperar la seal
moduladora en el punto b y cual es la expresin en el tiempo de la funcin de transferencia de
este filtro?
d) Cual es el espectro de la seal existente en el punto c?, dibujarla lo mas exactamente
posible. Escribir su expresin en el tiempo.
e) Cuanto debe valer T0, como mximo, para muestrear correctamente la seal existente
en c?
f) Dibujar el espectro de la seal existente en d, lo ms exactamente posible. Escribir su
expresin.
g) Si el codificador es de 256 niveles, Cuantos bytes de memoria ocuparan 20 minutos de
proceso?

130

4.7.- Demostrar que si una seal m(t) modulada en frecuencia atraviesa un sistema con una
funcin de transferencia cuyo espectro de amplitud y de fase es semejante al de la figura, la
seal de informacin x(t) ser retardada un tiempo tg.

|H (f)|

-fc

Espectro
de m(t)

fc

(f)

4.8.- Demodular la seal x(t)cos(2f0t), considerando que x(t) tiene un ancho de banda B.
(Realizar un diagrama de bloques y si se utiliza algn filtro, especifquelo dando una
expresin)
4.9.- Un sistema empleado comnmente para asegurar la privacidad en la comunicacin
hablada es un codificador de voz. Como se ve en la figura a, la entrada al sistema es una seal
de voz normal x(t) y la salida es la versin codificada y(t). La seal y(t) es transmitida y luego
decodificada en el receptor.
X(f)
x(t)

Codificador

y(t)

y(t)

Decodificador

x(t)
-fc

fc

Hz

Figura b

Figura a

Suponiendo que la seal x(t) es real y de banda limitada a fc y que nuestro codificador lo que
hace realmente, es permutar las diferentes bandas del espectro de la seal de entrada, es decir
el espectro de salida Y(f) del codificador puede expresarse como:
Y(f)= X(f-fc) para f>0

Y(f)= X(f+fc) para f<0

Si X(f) est dada por el espectro mostrado en la figura b, se pide:


a) Dibujar el espectro de la seal codificada y(t), teniendo en cuenta que debe tener el mismo
ancho de banda que x(t).

131

b) Usando amplificadores, multiplicadores, sumadores, osciladores y filtros ideales, dibuje un


diagrama de bloques para el codificador ideal, justificndolo tericamente.
c) Usando amplificadores, multiplicadores, sumadores, osciladores y filtros ideales, dibuje un
diagrama de bloque para el decodificador ideal asociado, justificndolo tericamente.
4.10.- Encontrar la expresin de y(t) en el diagrama de bloques (sistema realimentado) de
la figura, si x(t)=cos 2100t, y h(t)= sinc t, con =1000 y A=1/2100.

x(t) >+ +
- ^

>

>

h(t)

>

d
dt

<

y(t)
>

<

4.11- En el diagrama de bloques de la figura, la seal x(t) que tiene un ancho de banda 'a'

t nT0
siendo h(t ) 4a sinc(2at ) cos(2f 0 t ) y p (t )
n T0
2

x(t)
mA

con f0=1/T0

h(t)

y(t)

+
A

p(t)

Se pide:
1) Dibujar el espectro de la seal existente inmediatamente despus del sumador. Suponga
una forma arbitraria del espectro de x(t).
2) Dibujar el espectro del ltimo bloque del diagrama Que clase de filtro es y cual es su
ancho de banda?
3) El primer bloque es un amplificador que amplifica la seal por mA. Cul es la expresin
de su respuesta impulsional y cual es la expresin de su espectro?
4) Escribir la expresin del espectro de la seal p(t). Dibujarla lo ms exactamente posible.
5) Escribir y dibujar la expresin del espectro de la seal existente inmediatamente despus
del multiplicador.
6) Dibujar el espectro de la seal y(t). Escribir su expresin.
132

7) Escribir la expresin de la seal y(t). Que tipo de modulacin es.


4.12.- Determinar la expresin de m(t) si la entrada y la salida de los elementos no lineales
est relacionada como s(t)=a1 + a2 e(t) + a3 e(t)2.
e1(t)

s1(t)

Elemento
no lineal

+
kx(t)

cos(2fct)
-

e2(t)

4.13.p (t )

Demostrar

que

al

m(t)

s2(t)

Elemento
no lineal

multiplicar

x m (t ) A(1 mx(t )) cos(2f 0 t )

por

t nT0
) y despus filtrar paso bajo convenientemente (expresar el filtro como
T0 2

un paso bajo ideal) se puede recuperar la seal x(t) que tiene un ancho de banda a mucho
menor que f0.
4.14.- Las seales x(t), y(t) y z(t), del diagrama de la figura, son del tipo paso bajo, tienen
un ancho de banda wa y se desean modular en doble banda lateral para multiplexarlas
posteriormente. Si p (t )

(1) (
n

T0

nT
0
4
2)
T0 2

x(t)

a1
p(t)

y(t)

h2(t)

h3(t)

a2
z(t)
a3

s1(t)

h1(t)

s2(t)

d(t)

s3(t)

a) Explique cual es funcionamiento bsico de este diagrama.


b) Escriba las expresiones de las respuestas impulsionales hn
c) Escriba tambin las expresiones de d(t) y de D(f) y dibuje esta ltima.
133

d) Que valores deben tener cada uno de los coeficientes an para que todas las seales
multiplexadas tengan el mismo nivel
t'

4.15.- La expresin de la entrada del sistema y (t ) A cos(2 f 0t 2 k x(t ')dt ') es una seal
0

modulada en frecuencia, teniendo un ancho de banda 6B centrada en f0. La amplitud de la


sea x(t) toma valores entre 1 y su ancho de banda B es mucho menor que f0.

Atencin f c

f0
2

s2 (t )

y(t)

s1(t) Filtro paso


banda h1

cos(2fct)

s3(t)

s6(t)

sen(2fct)

/2

d/dt

t'
A
f
cos(2 0 t 2 k x(t ')dt ')
2
0
2

Filtro paso
s4(t) banda h2

Filtro paso
bajo h3

s(t)

-1
s5(t)

a) Encontrar las expresiones de s1(t), s3(t), y s4(t).


b) Si el filtro paso banda h1es ideal escribir la expresin de su respuesta impulsional.
c) Si la seal s5(t) debe ser una modulacin en AM convencional, escribir la expresin de
la respuesta impulsional del filtro paso banda h2.
d) Escribir las expresiones y dibujar los mdulos de los espectros de las seales s5(t) y
s6(t).
e) Escribir la expresin de la respuesta impulsional del filtro paso bajo adecuado h3.
Escribir la expresin de s(t).

4.16.- Las seales x(t), y(t) y z(t) son del tipo paso bajo, tienen un ancho de banda wa y
estn

multiplexadas

en

frecuencia

formando

la

seal

d (t ) x(t ) sen(2 f 0t ) y (t ) sen(2 3 f 0t ) z (t ) sen(2 5 f 0t ) .

Si p(t )

(1) (

T0

nT
0
4
2 ) teniendo en cuenta adems que f 2w .
0
a
T0 2

a) Explique cual es funcionamiento bsico de este diagrama de la figura.

134

b) Escriba las expresiones de cada una de las respuestas impulsionales de los filtros ideales
h0(t), h1(t), h2(t) y h3(t) que le permitirn obtener el resultado que se pretende. (especificar
el ancho de banda y porque)
c) Escriba tambin la expresin de D(f) y dibuje su grfica, especificando claramente los
valores importantes tanto en ordenadas como en abscisas.
d) Que valores deben tener cada uno de los coeficientes a1, a2, a3 para que a la salida de los
amplificadores se recuperen exactamente las seales que fueron multiplexadas

p(t)

h1(t)
h2(t)
h3(t)

d(t)

h0(t)

a1

x(t)

h0(t)

a2

y(t)

h0(t)

a3

z(t)

4.17.- El diagrama de bloques de la figura representa la generacin de la seal estereo que


posteriormente ser modulada en FM y transmitida. Los canales izquierdo y derecho se filtran
paso bajo con banda de 0 a 15 kHz y despus se combinan. La frecuencia f0, de 19 kHz, se
denomina frecuencia piloto; se usa para modular en doble banda lateral sin portadora la
combinacin diferencia y tambin es necesaria en la demodulacin de la seal estereo.
Canal
izquierdo Filtro paso x (t)
I
bajo h1
Canal
derecho Filtro paso x (t)
D
bajo h1

xD(t)+xI(t)

xD(t)-xI(t) Modulador
DBL sin
portadora
cos(22fot)

x2

xbb(t) Modulador xFM(t)

de FM
cos(2fot)

a) Escriba la expresin de xbb(t) en funcin de xD(t) y xI(t)


b) Si el filtro paso bajo h1 es ideal escribir la expresin de su respuesta impulsional.
c) Con la forma de los espectros de xD(t) y xI(t) que considere, escriba la expresin de
Xbb(f) y dibuje su grfica, poniendo de manifiesto el ancho de cada una de las bandas
parciales que pudieran coexistir en la banda total.
135

d) Escribir la expresin de la seal xFM(t). Si la seal xbb(t) est normalizada a 1,


determine, usando la regla de Carson, cual sera el ancho de banda de la seal FM si la
constante de desviacin de frecuencia del modulador es de 75 kHz.
e) A partir de la seal xbb(t) dibuje un diagrama de bloques que represente la obtencin de
los canales izquierdo y derecho separadamente, es decir realice la operacin inversa que se
efectuara en el receptor una vez demodulado en FM.
f) Escriba las expresiones tanto en tiempo como en frecuencia de todos los filtros que
considere en el apartado e).
g) Se podra escuchar la informacin transmitida en un receptor que no fuese estereo?
Raznelo.

4.18.- El diagrama de bloques de la figura representa un sistema para codificar una seal y(t).
Se supone que esta seal no tiene un ancho de banda superior a 4 kHz, no obstante los dos
filtros paso bajo a 4 kHz de la primera etapa lo asegura. El filtro paso bajo con respuesta
impulsional h1(t) tiene una banda de paso de 2 kHz y el filtro paso banda con respuesta
impulsional h2(t) tiene una banda de paso de 2 kHz a 4 kHz.
Atencin considere Y ( f ) (

f
) , adems todos los filtros son ideales.
4000
cos(22000t)

F.P bajo
0-4 kHz

x(t)

y(t)

s1(t)

Filtro paso s3(t)


bajo h1

cos(24000t)
F.P bajo
0-4 kHz

x(t)

s2(t)

s(t)

Filtro paso s4(t)


banda h2

a) Escriba la expresin de la respuesta impulsional de los dos primeros filtros del diagrama
de bloques y la expresin de la seal x(t).
b) Escriba las expresiones de s1(t), s2(t), S1(f), S2(f) y dibuje las grficas del modulo de
estas dos ltimas mostrando claramente las amplitudes y las frecuencias.
136

c) Escribir las expresiones de las respuestas impulsionales de h1 y de h2.


d) Dibujar los mdulos de los espectros S3(f), S4(f) y S (f), mostrando claramente las
amplitudes y las frecuencias.
e) Dibuje un diagrama de bloques del decodificador, es decir que recupere la seal y(t) y
escriba las expresiones de las respuestas impulsionales de todos los filtros que use. (Suponga
que dispone del oscilador necesario)

4.19.- Considere el sistema mostrado en la figura. Suponga que las salidas del comparador
son 1, dependiendo de la polaridad de la entrada y que la puerta NOR exclusiva da +1 si las
entradas son de la misma polaridad y -1 si son de distinta polaridad.
a) Demuestre que la tensin de salida z(t) contiene un termino proporcional a cos() y por
tanto el sistema se puede usar como un detector de fase.
b) Considerando el filtro paso bajo ideal, escriba su respuesta impulsional suponiendo el
mximo ancho de banda posible.
cos(2fct)

comparador

cos(2fct+)

Filtro paso
bajo

z(t)

comparador

** Se sabe que /4=1-1/3+1/5-1/7+......

4.20.- La seal de entrada al diagrama de bloques es una seal modulada en amplitud con
portadora, esto es xAM (t ) (1 mx(t )) sen(2 f 0t ) , y x(t) una seal cuyo espectro se extiende
desde los 400 Hz hasta los 15 kHz.

Amplitud salida

Caracterstica del comparador

xAM(t)

c(t)
Comparador

1
-1

Amplitud entrada

d(t) Filtro h1

z(t)

a) Explique razonadamente cual es el objeto de este diagrama de bloques


137

b) Escriba la expresin de c(t) y su desarrollo en serie de Fourier


c) Escriba la expresin de d(t).
d) Escriba las expresiones tanto en tiempo como en frecuencia del filtro h1 que cree
precisar.
e) Si en lugar de un comparador se coloca un filtro paso banda, escriba las expresiones en
tiempo y en frecuencia del filtro y explique con que problema debera enfrentarse si se
pretende obtener un sistema util.
f) Si ahora el diagrama de bloques es el de la figura siguiente,
xAM(t)

c(t) d(t)=c(t)2

h(t)=1/t

d(t)

e(t)=xAM(t)2 e(t)

g(t)

Raiz
cuadrada

z(t)

Escriba las expresin de c(t).


g) Escriba las expresin de g(t). Es z(t) la seal que originalmente se desea? En cualquier
caso especifique claramente que es z(t).
h) Explique razonadamente la utilidad, en cuanto a una implementacin fsica del diseo,
de cada uno de los dos diagramas de bloques.

4.21.- Si p (t )

(1) (
n

t T0 4 nT0 2
) , con f0=1/T0 igual a 1 MHz, x(t) tiene un ancho
T0 2

de banda B= 4 kHz, la respuesta al impulso h(t)=1/t, la entrada y la salida de los elementos


no lineales est relacionada como s(t)= e(t)2, y todos los filtros paso banda son ideales
centrados a la frecuencia f0 Hz con ancho de banda 8 kHz, determinar:
e1(t)

x(t)

p(t)

F. P.
Banda

Elemento
no lineal

s1(t)

F. P.
Banda
y(t)

a(t)

h(t)
e2(t)

Elemento
no lineal

s2(t)

F. P.
Banda

*Suponga cualquier forma para el espectro de amplitud de x(t)

138

a) La expresin de a(t). Justificarla.


b) Las expresiones de e1(t) y E1(f) (espectro de e1(t)). Dibujar la grafica del espectro de
amplitud de e1(t).
c) La expresin de la respuesta impulsional de los filtros paso banda
d) Dibuje el espectro de amplitud de la seal s1(t)
e) Calcule la expresin de e2(t)
f) Determinar la expresin de y(t)
g) Que tipo de modulacin es y cual es el objeto de la seal a(t). Plantee un diagrama de
bloques que describa como recuperar la seal x(t) sin olvidar la cuestin del sincronismo de
portadora.
h) Justificar con precisin el apartado de recuperacin de la portadora
* Para los apartados g) y h) haga un dibujo del espectro de amplitud de la seal y(t)
manteniendo la forma del espectro de x(t) que ha usado en los apartados anteriores. Haga un
breve comentario explicativo de cada uno de sus elementos. Utilice este espectro como
referencia en sus contestaciones.

4.22.- Las seales x1(t), x2(t)..... x8(t) representan 8 seales normalizadas en amplitud, y se
desea multiplexarlas en frecuencia para ser transmitidas conjuntamente en una nica seal
y(t). Cada una de ellas tiene un ancho de banda de B=50 Hz (de 0 a 50 Hz). El diagrama de
bloques que se propone para este multiplexor se muestra en la figura siguiente.
h1(t)
Con p(t ) ( t nT0 )
T0 10
n

a1

y
f0

1
100 Hz .
T0

x1(t)

p(t)

h2(t)

a2

.
.
.
.
h8(t)

c1(t)

. x2(t)
.
.
.
a8

c2(t)

s(t)

y(t)

c8(t)

x8(t)

a) Explique cual es el objeto de la seal p(t) y de los filtros paso banda cuya respuesta
impulsional es hn(t). A su parecer de que tipo de modulacin son las seales c1, c2, c3,.....c8.
139

b) Escriba las expresiones de las respuestas impulsionales hn(t) de los filtros ideales que le
permitirn obtener el resultado que se pretende (escriba solo el h3(t) y el h8(t)), especifique el
ancho de banda y raznelo. Escriba y dibuje el espectro de amplitud de H3(f) y H8(f)
c) Que valores deben tener cada uno de los amplificadores a1, a2,...... a8 para que en la seal de
salida y(t) exista uniformidad en la amplitud de las seales multiplexadas
d) Escriba las expresiones c1(t) y c8(t). Escriba tambin las expresiones C1(f), C8(f) y dibuje su
grfica de amplitud, especificando claramente los parmetros importantes. Especifique cual
debera ser la frecuencia de muestreo de cada una de estas dos seales.
e) Escriba la expresin de y(t) y dibuje su espectro de amplitud (solo para frecuencias
positivas), especificando claramente los parmetros importantes. Cul es objeto de la seal
s(t)?. En el caso de ser necesario A que frecuencia debera muestrear la seal y(t)?
f) Proponga un diagrama de bloques que recupere cada una de las seales de informacin xn.
Debe obtener el sincronismo de portadora correctamente.

4.23.- Sea y (t ) A sen(2f 0 t 1 (t )) , con 1 (t ) 2 f d x(t )dt . En condiciones normales de


0

funcionamiento, la seal sinusoidal v(t ) B cos(2 f 0t 2 (t )) que es la salida del oscilador


controlado por tensin (V.C.O.) sigue con muy poca diferencia de fase a la seal y(t), es decir

(t ) 1 (t ) 2 (t ) 1 radian . Suponiendo el filtro paso bajo ideal y con un ancho de banda


tal que solo deje pasar las variaciones de sen ( (t )) , demostrar como recuperar la seal x(t).
y(t)

v(t)

F.P.B

(AB/2)sen (t)

V.C.O.

140

4.24.- La seal xFM (t ) A sin(2 f 0t 2 k y (t ')dt ') , con una seal de informacin y(t)
0

normalizada a uno y valor k=104, se recibe en el punto de entrada a del diagrama de la


figura.
a

xFM(t)

d/dt
Desfase
/2

*El desfasador convierte


un seno en un coseno

Y(f)

F. P.
Bajo

x1(t)
Diagrama de
bloques

x2(t)
x3(t)

*Regla de Carson: BW=2(f+fm); f =k1

*Diagrama espectral de la seal y(t)

* Los espectros de las seales de informacin


x1, x2 y x3 tienen formas arbitrarias

Xm1(f)

Xm2(f)

f (kHz)

Xm3(f)

a) Segn la regla de Carson cual es el ancho de banda de la seal xFM(t). Escribir la expresin
existente en el punto b.
b) Escribir la expresin de la funcin de transferencia H(f) del Filtro Paso Bajo ideal
(F.P.Bajo) que permite recuperar la seal y(t). Escribir tambin su respuesta impulsional.
c) Escribir la expresin en el tiempo xm1(t), xm2(t) y xm3(t) de cada una de las seales
moduladas Xm1(f), Xm2(f) y Xm3(f) que estn multiplexadas en frecuencia. .Cual es el ancho
de banda de cada una de ellas? A que frecuencia muestreara cada una de ellas?Cul es el
tipo de modulacin?
d) Si en el punto c se dispone de la seal y(t), proponga un diagrama de bloques (justificado
con expresiones) que permita recuperar las seales de informacin x1(t), x2(t) y x3(t) que se
han multiplexado en frecuencia.

4.25.- Se desea muestrear la seal x(t), cuya amplitud est normalizada a uno, con el tren de
pulsos triangular p (t )

t nT0
) , siendo la frecuencia de muestreo f0=2b=1/T0 igual a 8
0 10

( T

141

kHz. El diagrama de bloques de este proceso de muestreo se esquematiza en la figura a) y la


grfica del espectro de x(t) con b= 4 kHz se muestra en la figura b).
x(t)

X(f)

xm(t)
p(t)

-b

Figura a

1
b
Figura b

Hz

Se pide:
1) Desarrollar p(t) en serie de Fourier. (Expresar el desarrollo con funciones sinusoidales
indicando claramente el valor o la expresin de sus amplitudes).
2) Escribir la expresin Xm(f) del espectro de la seal muestreada xm(t) y dibujar una grafica
que de idea de cmo es su espectro de amplitud, especificando claramente los valores de los
parmetros de inters.
3) Proponga un diagrama de bloques que permita recuperar la seal x(t) a partir de la seal
muestreada xm(t). Escriba la/las expresin/expresiones en tiempo y en frecuencia de cada
elemento del bloque.
4)

La

seal

muestreada

xm(t)

se

convoluciona,

en

un

caso,

con

h1 (t ) 4b sinc(2bt ) cos(2 10 f 0t ) y, en otro caso, con h2 (t ) 4b sinc(2bt ) cos(2 15 f 0t ) .


Escriba la expresin resultante de xm (t ) h(t ) para cada caso.
5) Opere la expresin xm (t ) 4b sinc(2bt ) cos(2 15 f 0t ) cos(2 15 f 0t ) 2b sinc(2bt ) y
muestre el espectro de la expresin resultante. Reconoce en la expresin a operar, algn
procedimiento estndar empleado en procesado de la seal? Escriba un diagrama de bloques
de este procedimiento.
6) Si ahora x(t) es la seal moduladora en una modulacin en FM, cuya frecuencia instantnea
es fi (t ) 15 f 0 2000 x(t ) y desea muestrear esta seal modulada en FM Cul seria el valor
de la frecuencia de muestreo que tomara? Justifique su respuesta y apyela con diagramas de
bloque y las expresiones necesarias.

4.26.- x(t) e y(t) son seales de audio, cada una con ancho de banda igual a 15 kHz. Suponga
que representan el canal izquierdo y derecho de una grabacin. Si se procesan segn el

142

diagrama de bloques de la figura, en donde el interruptor se puede simular con ayuda de las
seales z1 y z2, cuya f 0

1
38 103 Hz. determinar:
T0
z1(t)

z1(t)

x(t)

..........
z2(t)

T0

............
t

T0/2

........

............
t

F. P. Bajo
ideal a 53 kHz

d(t)

e (t)

z2(t)

y(t)

Oscilador
senoidal a 38
kHz

m(t)

Divisor de
frecuencia
por 2

*Suponga cualquier forma para los espectros de amplitud de x(t) e y(t)


1) Desarrollar en serie z1(t) y z2(t).
2) La expresin d(t). Justificarla.
3) La expresin de la respuesta impulsional del filtro paso bajo
4) Las expresiones e(t) y E(f) (espectro de e(t)). Dibujar la grafica del espectro de amplitud de
e(t).
5) Las expresiones m(t) y M(f). Dibujar la grafica del espectro de amplitud de m(t).
6) En su opinin, que utilidad tiene la seal m(t) Conoce alguna circunstancia en la que se
usa este procedimiento?. Plantee un diagrama de bloques que describa como recuperar las
seales x(t) e y(t) a partir de m(t), justificando con precisin el objetivo de los bloques.
4.27.- Se dispone de una seal peridica z(t) que se usa como entrada en un sistema para
generar una seal continua y tres tonos, tal como se muestra en la figura siguiente. El filtro
paso banda H1 es ideal, centrado en f0 Hz y de ancho de banda 1 Hz.
xb(t)

z (t)
H1

2/4

xa(t)

H2

a1

H3

a2

H4

a3

H5

a4

1
cos(2f0t)
cos(22f0t)
cos(23f0t)

143

Con cos(a b) cos(a) cos(b) sen(a) sen(b)

a) Escriba z(t) como una suma de funciones senoidales.


b) Calcule y exprese la seal xa(t).
c) Calcule y exprese la seal xb(t).
d) Escriba las expresiones de las respuestas impulsionales h(t), y de los espectros H(f) de los
filtros ideales H2, H3, H4 y H5 (el primer filtro es paso bajo y los restantes filtros son paso
banda). Razone los anchos de banda que considera y dibuje las graficas de los espectros de
amplitud.
e) Calcule el valor de los amplificadores de salida para obtener las seales deseadas.

4.28.- Se dispone de las seales de informacin x(t) e y(t), cada una con ancho de banda B
Hz, y cumplen la relacin tan(k (t ))

y (t )
: La seal k(t) es peridica y se muestra
x(t )

grficamente. H1 y H2 son filtros paso banda centrados a f0 Hz, el primero con ancho de
banda 2B Hz (B>>1) y el segundo con ancho de banda 1 Hz. El diagrama de bloques que
procesa estas seales es el siguiente.
x(t)

H1

a(t)

limitador

1/2f0

H3

-1

H2

d (t )

(t nT0 )

Desfase
/2

1/2f0

y(t)

k(t)

1s

..........

-0.5

s(t)

0.5

..........
1.5

2.5

t (s)

a) Escribir la expresin en el tiempo de a(t) Qu tipo de modulacin es?


b) H3 es un filtro paso banda centrado en f0. Explique cual es su objeto, escriba la expresin
de H3(f) y razone el ancho de banda que considera.
c) Escribir la expresin de s(t) y la expresin de su frecuencia instantnea.
144

4.29.- En el diagrama de bloques de la figura las seales x(t) e y(t) son de tipo paso bajo, la
forma de su espectro es arbitraria pero tienen un ancho de banda de 4 kHz y puede tomarse
como referencia la figura dada. La frecuencia f0 del oscilador, en el diagrama de bloques, es
mucho mayor que el ancho de banda de estas seales y la respuesta impulsional del bloque
que se encuentra a la salida del oscilador es h(t ) (t
1

X(f)

-4

s3(t)

cos(2f0t)
+

1 Y(f)
-4

x(t)
f kHz

T0
).
4

z(t)

h(t)
f kHz

y(t)

s1(t)

s2(t)

a) Demuestre que s1(t) es la funcin sen(2 f 0t ) . Escribir las expresiones s2(t) y s3(t).
b) Escribir las expresiones de z(t) y Z(f) y dibujar, esquemticamente, el espectro de amplitud
de este ltimo. Cul es el ancho de banda de la seal z(t)? Explquese.
c) Dibuje un diagrama de bloques que permita recuperar las seales x(t) e y(t), teniendo en
cuenta que en el receptor no dispone de una oscilador sinusoidal sincronizado a la frecuencia
de la portadora. Escribir las expresiones de los filtros que use tanto en tiempo como en
frecuencia.

4.30.- Se desea modular en FM, con una portadora de f0 Hz una seal vocal x(t) con x(t ) 1 ,
cuyo ancho de banda es 15 kHz.
a) Cual debe ser el valor de la constante de desviacin de frecuencia fd si se requiere una
ocupacin en la banda espectral de la seal FM a transmitir de 160 kHz
b) Escriba la expresin en el tiempo de esta seal de FM.
c) Proponga un diagrama de bloques que permita recuperar la seal x(t) y escriba las
expresiones de los filtros y/o procedimientos que utilice.
145

4.31.- La seal x(t), en el diagrama de bloques de la figura, es de tipo paso bajo y est
normalizada a uno. La forma de su espectro X(f) es arbitraria pero tienen un ancho de banda
de 4 kHz y puede tomarse como referencia la figura dada. El periodo de repeticin del tren de
impulsos i (t )

(t nT ) , es T

1
103 segundos y la respuesta impulsional del ltimo
8

bloque es h(t ) 16 103 sinc(8 103 t ) cos(2 10 f 0t ) .

x(t)

a(t)

b(t)

h(t)

z(t)

i(t)

a) Escribir las expresiones a(t) y A(f) y dibujar, esquemticamente, el espectro de amplitud de


esta ltima. Cul es el ancho de banda de la seal a(t)?
b) Expresar i(t) en funcin de seales sinusoidales. Escribir la expresin I(f).
c) Escribir las expresiones b(t) y B(f) y dibujar, esquemticamente, el espectro de amplitud de
esta ltima. Cul es el ancho de banda de la seal b(t)?
d) Escribir la expresin H(f) y dibujar su mdulo.
e) Escribir las expresiones z(t) y Z(f) y dibujar, esquemticamente, el espectro de amplitud de
esta ltima. Cul es el ancho de banda de la seal z(t)? Explquese.
f) Dibuje un diagrama de bloques que permita recuperar la seal x(t) a partir de la seal z(t),
teniendo en cuenta que nicamente dispone de multiplicadores, sumadores, filtros y
amplificadores. Escribir las expresiones, tanto en tiempo como en frecuencia, de los
amplificadores y de los filtros que use.

4.32.- Una seal x(t) con ancho de banda 4 kHz y normalizada en el tiempo a 2, se muestrea
a la frecuencia mnima f0 con un tren de pulsos p(t) muy estrechos. El objetivo consiste en

146

obtener una modulacin en amplitud con portadora a 80 f0 Hz, segn el diagrama de bloques
siguiente:
a(t)
x(t)

H1

H2

p(t)

b(t)

X(f)

xm(t)
-4

a
4

f kHz

c(t)

p(t )

T0
t nT0
siendo 100 s

a) Escribir la expresin p(t) y desarrollarla en serie de Fourier expresndola en funcin de


senos y/o cosenos.
b) Escribir la expresin P(f) del espectro de la seal p(t) y dibuje de forma razonable su
grfica.
c) Escribir la expresin a(t) y tambin el espectro de amplitud de, nicamente, el armnico 80

f0. Dibuje este espectro de forma razonable (rotular correctamente los ejes, amplitudes etc)
d) Escribir las expresiones H2(f) y h2(t) del filtro, dejando bien claro cual es su ancho de
banda, si lo que se pretende es disponer a su salida de una funcin sinusoidal a la frecuencia
80 f0.
e) Escribir las expresiones H1(f) y h1(t) del filtro, dejando bien claro cual es su ancho de
banda, si lo que se pretende es disponer a su salida de una modulacin en amplitud de la seal
x(t).
f) Calcule el valor A del amplificador para que la funcin sinusoidal de salida tenga amplitud
1.
g) Debe ser A=B?. En cualquiera caso raznelo y escriba el valor de B y la expresin de
xm(t).
h) Suponiendo que no dispone de osciladores, dibuje un diagrama de bloques que permita
recuperar la informacin x(t), explique como opera, y escriba las expresiones en tiempo y en
frecuencia de todos los filtros que use.

147

4.33.- Dos seales x(t) e y(t), con anchos de banda a 4 kHz, se usan como moduladoras en un
cierto tipo de modulacin en amplitud con portadora f0, segn el diagrama de bloques
siguiente:

h(t )

y(t)

x(t)

h(t)

a(t)

h(t)

c(t)

cos(2f0t)

b(t)

X(f)

m(t)

- +
d(t)

-4

Desfase
-/2

Y(f)

1
t

f kHz

f kHz

e(t)
-4

a) Escribir las expresiones A(f) y B(f) realizando la transformada de Fourier de a(t) y b(t).
b) Dibujar las grficas de amplitud y de fase de A(f) y B(f).
c) Escribir las expresiones c(t), C(f) y dibujar esquemticamente el espectro de amplitud este
ultimo.
d) Escribir la expresin m(t), y explique claramente que tipo o tipos de modulacin
representa.
e) A partir de la expresin m(t), escriba separadamente las expresiones de las modulaciones
mx(t), my(t), correspondientes a cada una de las seales x(t) e y(t).
f) Escriba las expresiones Mx(f) y My(f). Dibuje el espectro de amplitud de cada una de estas
modulaciones.
g) Escriba la expresin M(f) y dibuje su espectro de amplitud (rotulando convenientemente el
eje de frecuencia)
h) Suponiendo que no dispone de osciladores, dibuje un diagrama de bloques que permita
recuperar las seales x(t) e y(t) separadamente, explique como opera, y escriba las
expresiones en tiempo y en frecuencia, as como el dibujo de los espectros, de todos los filtros
que use.

148

4.34.- Las seales x(t) e y(t), con anchos de banda a 4 kHz, que representan el canal izquierdo
y el canal derecho de una seal de audio, se procesan segn el diagrama de bloques siguiente:

x(t)

c(t)

x(t)+y(t)

h2(t)

b(t)

h3(t)

e(t)

m(t)

h4(t)

i(t)
h1(t)
a(t)

+ x(t)-y(t)

y(t)

Y(f)

Con: i (t )

d(t)

X(f)

t nT
0

-4

kHz

-4

kHz

Siendo h1 (t ) t T0 , h2 (t ) 2 sinc(t ) cos(2 f 0t ) , h3 (t ) 16 103 sinc(8 103 t ) cos(2 f 0t ) ,


4

h4 (t )

1
t
2 f0

las respuestas impulsionales de los bloques. Adems T0= 10-6 segundos (con

f0=1/T0).

a) Escribir las expresiones del desarrollo en serie de Fourier de i(t) y a(t).


b) Escribir las expresiones de I(f) y A(f).
c) Escribir las expresiones de H2(f), H3(f) y H4(f). Dibujar esquemticamente las grficas de
sus espectros de amplitud.
d) Escribir las expresiones de c(t) y d(t) y dibujar esquemticamente las grficas de sus
espectros de amplitud.
e) Escribir las expresiones b(t) y B(f), y dibuje la grfica de su espectro de amplitud.
f) Escribir las expresiones e(t) y E(f), y dibuje la grfica de su espectro de amplitud.
g) Escribir la expresin de m(t). Que tipo de modulacin representa? Comntelo.
h) Suponiendo que no dispone de osciladores, dibuje un diagrama de bloques que permita
recuperar las seales x(t) e y(t) separadamente, explique como opera, y escriba las
149

expresiones en tiempo y en frecuencia, as como el dibujo de los espectros, de todos los filtros
que use.
i) Que ventajas o inconvenientes cree que tiene este sistema respecto a una modulacin en
estereofona estndar.

4.35.- Una seal x(t) que tiene un ancho de banda b Hz y est modulada en doble banda
lateral, tiene que ser derivada. Para ello, primero debera ser demodulada, y despus de
derivar x(t) volver a modularla. Demuestre qu d(t) del diagrama de bloques de la figura (a) es
una buena aproximacin a la modulada de la derivada de x(t), realizando el proceso de un a
vez. En la figura (b) se muestran espectro de amplitud y de fase del filtro paso banda h(t), en
el que ta es arbitrariamente pequeo.
|H(f)|

x(t) cos(2f0t)
h(t)

1/ta

d(t)

-f0- -f0 2f0ta


(b)

(a)

-f0

f0-b f0
(f)

Hz

f0

Hz

4.36.- En el diagrama de bloques siguiente, las seales x(t) e y(t) tienen, cada una, un ancho
de banda B=4 kHz y tambin se conocen la expresin de la respuesta impulsional
T
h 2(t ) t 0 . Adems 1/T0=f0 es mucho mayor que B. El objetivo consiste en obtener
4

la expresin xm(t) de un cierto tipo de modulacin en amplitud.


a(t)

p(t)

Y(f)

x(t)

c(t)

e(t)

h1(t
h2(t

b(t)

d(t)
y(t)

h3(t

xm(t)

-4

-0.4 0.4 4 f kHz


X(f) 1

-4 -0.4 0.4

4 f kHz

150

p (t )

t nT0

T0 2

a) Escribir la expresin p(t) expresndola en funcin de senos y/o cosenos. Escribir la


expresin P(f) del espectro de la seal p(t) y dibuje de forma razonable la grfica de |P(f)|.
b) Escribir la expresin b(t) Son iguales los mdulos del espectro de amplitud de la seales
a(t) y b(t)? Son iguales sus espectros de fase?
c) Dibujar, de forma razonable los espectros de amplitud de las seales c(t) y d(t) (rotulando
correctamente los ejes y mostrando claramente lo que se desea expresar).
d) Escribir las expresiones H1(f) y h1(t) del filtro, dejando bien claro cual es su ancho de
banda, si lo que se pretende es disponer a su salida de una funcin sinusoidal a la frecuencia 3

f0 .
e) Escribirla seal e(t)
f) Escribir las expresiones H3(f) y h3(t) del filtro ideal paso banda a 8 kHz centrado a 3 f0.Hz.
g) Escriba exactamente la expresin xm(t) (valores de amplitud correctos). Explique que tipo
de modulacin representa.
h) Suponiendo que no dispone de osciladores, dibuje un diagrama de bloques que permita
recuperar las informacin x(t) e y(t), explique como opera, y escriba las expresiones en
tiempo y en frecuencia de todos los filtros que use.

4.37.- La seal xm(t) es la seal modulada en frecuencia de una informacin estereofnica. En


este diagrama de bloques las seales de audio x(t) e y(t), cada una con un ancho de banda de
15 kHz, representan los canales izquierdo y derecho respectivamente. Se sabe que la
frecuencia para formar la seal moduladora z(t), que est normalizada a uno, es f0=38 kHz.
Tambin se sabe que la constante de desviacin de frecuencia del oscilador controlado por
tensin (VCO) es k=75000 y su frecuencia central es fm=100 MHz.
(No importan las formas de los espectros de la suma y de la resta de las seales x(t) e y(t))

151

x(t)

x(t)+y(t)
cos(2f0t)

y(t)

p(t)

Divisor
de frec.

x(t)-y(t)

z(t)

VCO v(t) h(t)


fm

xm(t)

r(t)

a) Dibuje esquemticamente el espectro de amplitud de la seal r(t) (rotule correctamente los


ejes)
b) Escriba la expresin de la seal z(t) y dibuje su espectro de amplitud.
c) Escriba la expresin de la frecuencia instantnea fi(t) que tendr la seal v(t).
d) Escriba la expresin de la seal v(t).
e) Escriba la expresin en tiempo y en frecuencia del filtro paso banda ideal necesario para
limitar la banda de paso. Razone su eleccin.
f) Proponga un diagrama de bloques que permita recuperar la seal moduladora z(t). Escriba
las expresiones en tiempo y en frecuencia de los filtros que use.
g) Proponga un diagrama de bloques que permita recuperar, separadamente, x(t) e y(t).
Escriba las expresiones en tiempo y en frecuencia de los filtros que use. (No puede disponer
de osciladores).

4.38.- Se desea generar una seal z (t ) y (t ) cos(2 f 0t ) cos(2 f 0t 2 k x(t )) formada por
una modulacin en doble banda lateral para la informacin y(t), sumada a una modulacin de
fase para la informacin x(t). Si x(t ) max 1 y la constante de desviacin de frecuencia es
k<<0.01 entonces, para generar la seal z(t) y recuperar despus las informaciones x(t) e y (t),
se propone el diagrama de bloques de la figura, teniendo el oscilador controlado por tensin
(VCO) un frecuencia instantnea f (t ) f 0 k a (t ) , con f0 mucho mayor 4 kHz.

152

x(t)

d(..)/dt

a(t)

Y(f)

b(t)
VCO
h(t)
c(t)
y(t)

xm(t)

-0.4 0.4 4 f kHz

-4

X(f) 1

d(t)
-0.4 0.4

-4

4 f kHz

a) Obtener la expresin xm(t) mostrando que z(t) puede ser aproximada por xm(t)
( z (t ) xm (t ) ). Para ello deber obtener la expresin de la respuesta impulsional h(t) del filtro
y las expresiones de las seales a(t), b(t), c(t) y d(t), aportando las explicaciones que crea
necesarias.
b) Suponiendo que, en recepcin, no dispone de osciladores, dibuje un diagrama de bloques
que permita recuperar las informacin x(t) e y(t), explique como opera, y escriba las
expresiones en tiempo y en frecuencia de todos los filtros que use.

4.39.- En el diagrama de bloques de la figura 1, las seales de voz x(t) e y(t) tienen, cada una,
un ancho de banda B=4 kHz y tambin se sabe que |x(t)|max<1 y f0>>B. El elemento no lineal
ENL tiene una caracterstica tal que la seal de salida es el cuadrado de la seal de entrada (
s(t)=e(t)2). El objetivo consiste en obtener la expresin xm(t) de un cierto tipo de modulacin
en amplitud.
X(f)
1+x(t)

ENL

cos(2f0t)
-/2
sen(2f0t)
y(t)

e (t)

ENL

s1(t)
-4 -0.4 0.4 4
s3(t)

s2(t)

h(t)

xm(t)
h1(t)
xm(t)

figura 1

x(t)

A
?

y(t)

Y(f)

kHz
f

1
kHz

-4 -0.4 0.4

4 f

figura 2

a) Escribir las expresiones s1(t), s2(t) y s3(t).

153

b) Dibujar esquemticamente el espectro de amplitud de la seal s3(t), rotulando


correctamente los ejes y mostrando claramente lo que se desea expresar.
c) Escribir las expresiones H(f) y h(t) del filtro ideal, dejando bien claro cual es su ancho de
banda, que finalmente permitira generar la seal modulada xm(t).
d) Escriba exactamente la expresin xm(t) (valores de amplitud correctos). Explique que tipo
de modulacin representa.
e) El diagrama de bloques de la figura 2 del enunciado representa el demodulador de esta
seal modulada xm(t). A la vista de la expresin xm(t) obtenida en el apartado anterior,
explique cual es el objetivo del bloque con respuesta impulsional h1(t), y escriba sus
expresiones en tiempo y en frecuencia.
f) En consecuencia con el apartado anterior e), dibuje el diagrama de bloques A que permita
recuperar la informacin x(t) e y(t), explique como opera, y escriba las expresiones en tiempo
y en frecuencia de todos los filtros que use. Escriba tambin las expresiones en el tiempo de
las seales a la entrada de estos filtros, dejando constancia de los trminos que filtran.

4.40.- En el diagrama de bloques de la figura, la seal de voz x(t) tiene un ancho de banda

t nT0
B=4 kHz y tambin se sabe que |x(t)|max<1, z (t )
y 2B=f0=1/T0. Se pide:
n
T0 2

x(t)

s1(t)

z(t)

h1(t)
h2(t)

s3(t)

s2(t)

xm(t)

X(f)

-4 -0.4 0.4 4

kHz
f

a) Desarrolle en serie de Fourier la seal z(t) y exprsela como una suma de funciones
sinusoidales.
b) Escriba este desarrollo de z(t) calculando sus coeficientes hasta el termino n=11.
c) Si se conoce que la respuesta impulsional h1(t ) 4 B Sinc(2 Bt ) cos(2 9 f 0t ) , Escriba la
expresin H1(f) y dibuje su espectro de amplitud, rotulando convenientemente los valores en
los ejes de coordenadas.

154

d) Dibuje esquemticamente el espectro de amplitud de la seal s1(t) hasta la frecuencia 9f0


Hz. Escriba las expresiones s3(t) y S3(f). Dibuje le espectro de amplitud de S3(f).

f 9 f0
f 9 f0
e) Si se conoce que la funcin de transferencia H 2( f )

,
2 B 100
2 B 100
Escriba la expresin h2(t). Explique que funcin cumple este bloque y escriba la expresin
s2(t).
f) Escriba exactamente las expresiones xm(t) y Xm(f) y dibuje |Xm(f)| (con valores de amplitud
correctos). Explique que tipo de modulacin representa.
g) En consecuencia con el apartado anterior f), dibuje el diagrama de bloques que permita
recuperar la informacin x(t), explique como opera, y escriba las expresiones en tiempo y en
frecuencia de todos los filtros que use.

4.41.- En el diagrama de bloques de la figura, las seales de voz x(t) e y(t) tiene un ancho de

t nT0
banda B=4 kHz y tambin se sabe que |x(t)|max e |y(t)|max <1, p (t )
y
n
T0 2
2B=f0=1/T0. Se pide:

x(t)
p(t)

s1(t)

s2(t)

h1(t)
h2(t)

s3(t)

h3(t)

X(f)

s4(t)

xm(t)

-4 -0.4 0.4 4

Y(f)

kHz
f
-4 -0.4 0.4 4

kHz
f

y(t)

a) Desarrolle en serie de Fourier la seal p(t) y exprsela como una suma de funciones
sinusoidales hasta el sexto trmino (n=6) y con sus correctas amplitudes. Calcule la
transformada de Fourier de p(t) y dibuje esquemticamente su espectro de amplitud.
b) Si se conocen las respuestas impulsionales h1(t ) 4 B Sinc(2 Bt ) cos(2 5 f 0t ) y

h 2(t ) 2 Sinc(t ) cos(2 5 f 0t ) , escriba las expresiones H1(f) y H2(f) y dibuje sus espectros
de amplitud, rotulando convenientemente los valores en los ejes de coordenadas.
c) Dibuje esquemticamente el espectro de amplitud de la seal s1(t) hasta la frecuencia 6f0
Hz. Escriba las expresiones s2(t) y S2(f). Dibuje le espectro de amplitud de S2(f).
155

d) Escriba la expresin s3(t).


e) Si se conoce que la respuesta impulsional h3(t )

1
, calcule y escriba la expresin s4(t).
t

f) Escriba exactamente las expresiones xm(t) y Xm(f). Que tipo de modulacin representa.
g) En consecuencia con el apartado anterior f), dibuje el diagrama de bloques que permita
recuperar la informacin x(t), explique como opera, y escriba las expresiones en tiempo y en
frecuencia de todos los filtros que use.

4.42.- Se desea sustituir, mediante una aproximacin, un bloque integrador (figura 1) por dos
bloques pero que uno de ellos sea un derivador como en la figura 2.

x(t)

y (t ) x(t ')dt '


0

figura 1

y(t)

x(t)
figura 2

h(t)

a
b
x2(t)

d x2 (t )
y2 (t )
dt

y(t)

y2(t)

Teniendo en cuenta que x(t) es una seal modulada en amplitud a la frecuencia de portadora f0
se pide:
a) Primero, dibujar esquemticamente el espectro de amplitud y de fase de la funcin de
transferencia del bloque integrador (figura 1).
b) Usando el desarrollo en serie de Taylor de una funcin f(z) en un punto z=a, esto es:
f ( z ) f (a ) f '(a )( z a )

f ''(a )
( z a ) 2 ........ y tomando solo los dos primeros trminos, calcule
2!

las constantes a, b y la respuesta impulsional h(t) correspondientes, para obtener esta


aproximacin.

4.43.- En el diagrama de bloques de la figura, la seal de voz x(t) tiene un ancho de banda

t nT0
T0
B=4 kHz, p (t )
, y tambin se sabe que h1(t ) (t ) ,
4
n
T0 2

h 2(t ) 4 B Sinc(2 Bt ) cos(2 3 f 0t ) y B<<f0=1/T0.


156

x(t)
p(t)

s1(t)
h1(t)

s2(t)

h2(t)

X(f)

y(t)

-4 -0.4 0.4 4

p1(t)

kHz
f

Se pide:
a) Desarrolle en serie de Fourier la seal p(t) y escrbala como una suma de funciones
sinusoidales. Calcule el valor de sus coeficientes para los 5 primeros trminos.
b) Escriba la expresin de s1(t). Escriba la expresin de S1(f). Dibuje esquemticamente el
espectro de amplitud de s1(t).
c) Escribir la expresin de la funcin de transferencia (H2(f)) de h2(t). Dibuje su espectro de
amplitud. Diga que representa este sistema.
d) Escribir la expresin de la funcin de transferencia (H1(f)) de h1(t). Dibuje su espectro de
amplitud y su espectro de fase. Explique que operaciones realiza este sistema a cada una de
las seales sinusoidales que entran en l.
e) Opere y escriba la expresin p1 (t ) p(t ) h1(t ) p(t ) (t

T0
).
4

f) Dibuje esquemticamente el espectro de amplitud de la seal s2(t)


g) Escriba la expresin de y(t) y diga en que tipo de modulacin la clasificara.
h) En consecuencia con el apartado anterior g) y suponiendo que no dispone de ningn tipo de
oscilador, dibuje el diagrama de bloques que permita recuperar la informacin x(t), explique
como opera, y escriba las expresiones en tiempo y en frecuencia de todos los filtros que use.
y) Si fuese la seal |x(t)|max<< radianes, proponga una expresin alternativa para aproximar
y(t) y explique porque la propone. En este caso, escriba las expresiones de la frecuencia
instantnea y de la fase instantnea de la seal aproximada y(t).

4.44.- En el diagrama de bloques de la figura, las seales x(t), y(t) y z(t) tienen un ancho de
1
, h 2(t ) 2 B Sinc(2 Bt ) cos(2 f 0t ) ,
banda B=4 kHz. Tambin se sabe que h1(t )
t
h3(t ) 400 Sinc(100t ) cos(2 2 f 0t ) y B<<f0=1/T0.

157

z(t)

x(t)

e(t)

s=e2
s1(t)

cos(2f0t)

h1(t)

s(t)

h2(t)

h3(t)

-4 -0.4 0.4 4

s3(t)

p(t)

kHz
f

espectro genrico de las seales de entrada

s2(t)

y(t)

Se pide:
a) Escriba la expresin de s(t). Dibuje esquemticamente el espectro de amplitud de s(t),
especificando claramente cual es el ancho de banda de cada una de sus componentes.
b) Escribir las expresiones de las funciones de transferencia (H2(f) y H3(f)) de h2(t) y de
h3(t). Dibuje sus espectros de amplitud. Diga que representan estos sistemas. Especifique
claramente cuales son sus anchos de banda
c) En consecuencia con el apartado anterior b), escriba la expresin de la seal s3(t)
especificando claramente las amplitudes resultantes.
d) En consecuencia con el apartado anterior b), escriba la expresin de la seal s4(t)
especificando claramente las amplitudes resultantes. Dibuje su espectro de amplitud. No
olvide rotular los valores del eje de frecuencias.
e) Escribir la expresin de la funcin de transferencia (H1(f)) de h1(t). Dibuje su espectro de
amplitud y su espectro de fase.
f) Opere y escriba la expresin s1 (t ) cos(2 f 0t ) h1(t ) .
g) Escriba la expresin de la seal s2(t) y dibuje esquemticamente su espectro de amplitud.
h) Escriba la expresin de p(t) y explique que tipo o tipos de modulacin se usa.
y) En consecuencia con el apartado anterior h) y suponiendo que no dispone de ningn tipo de
oscilador, dibuje el diagrama de bloques que permita recuperar solo las seales x(t) e y(t)
explique como opera, y escriba las expresiones en tiempo y en frecuencia de todos los filtros
que use.
j) A partir de la seal p(t) y mediante la recuperacin de la portadora a f0 Hz, dibuje el
diagrama de bloques que permita multiplicar esta frecuencia por dos y despus recuperar la
seal z(t). Explique como opera (sobre todo el multiplicador), y escriba las expresiones en
tiempo y en frecuencia de todos los filtros que use.

158

4.45.- En el diagrama de bloques de la figura las seales x(t) e y(t) con una amplitud < 1, tiene
un

ancho

de

banda

B=4

kHz.

Tambin

se

sabe

que

i (t )

t nT ,
0

h1(t ) 4 B Sinc(2 Bt ) cos(2 6 f 0t )


2B=f0=1/T0.

h 2(t ) 4 B Sinc(2 Bt ) cos(2 5 f 0t )

X(f)

x(t)

s1(t)

s2(t)

y(t)

h1(t)

i(t)

h2(t

z(t)

-4 -0.4 0.4 4
Y(f)

siendo

kHz
f

p(t)
-4 -0.4 0.4 4

kHz
f

Se pide:
a) Desarrolle en serie de Fourier la seal i(t) y escrbala como una suma de funciones
sinusoidales. Especifique el valor de sus amplitudes. Escriba tambin la expresin de I(f).
b) Escriba la expresin de s1(t). Escriba la expresin de S1(f). Dibuje esquemticamente el
espectro de amplitud de s1(t).
c) Escriba la expresin de s2(t). Escriba la expresin de S2(f). Dibuje esquemticamente el
espectro de amplitud de s2(t).
d) Escribir las expresiones de las funciones de transferencia (H1(f) y H2(f)) de h1(t) y de
h2(t). Dibuje sus espectros de amplitud. Diga que representan estos sistemas.
e) Escriba la expresin de s3(t). Escriba la expresin de S3(f). Dibuje esquemticamente el
espectro de amplitud de s3(t).
f) Escriba la expresin p(t). Escriba la expresin de P(f). Dibuje esquemticamente su
espectro de amplitud.
g) Escriba la expresin de z(t) y dibuje esquemticamente su espectro de amplitud. Diga en
que tipo de modulacin la clasificara.
h) Suponiendo que recibe z(t) y que no dispone de ningn tipo de oscilador, dibuje el
diagrama de bloques que permita recuperar la informacin x(t) e y(t), explique como opera, y
escriba las expresiones en tiempo y en frecuencia de todos los filtros que use.

159

y) Si fuese necesario utilizar menos ancho de banda para la seal z(t), proponga otro diagrama
de bloques para generar z(t) con la mitad de su ancho de banda. En este caso explique su
diagrama y escriba las expresiones en tiempo y en frecuencia de los subsistemas que use.

4.46.- La seal de entrada al diagrama de bloques de la figura es p (t )

t nT0
y las
20
0

seales tambin de entrada x(t), y(t) y z(t) tiene un ancho de banda b=f0/2 Hz cuyos espectros
genricos estn representados en las figuras. Tambin se conocen las respuestas al impulso
h1(t )

f0
f
f
f
sinc 0 t cos 2 f 0t , h 2(t ) 0 sinc 0 t cos 2 2 f 0t
10
10
20
20

y h10(t ) f 0 sinc f 0 t cos 2 10 f 0t .


10

1+x(t)
1+y(t)
p(t)

h1(t)
h2(t)
h10(t)

c1(t)
c2(t)
c10(t)

a1

1+z(t)

a2

d(t)

20

X(f) 1

v(t)

-b

Y(f) 1

-b Z(f)

a10
-b

Hz
f

Hz
f

Hz
f

a) Exprese p(t) en una suma de funciones sinusoidales.


b) Escriba la expresin de las funciones de transferencia H 1( f ) , H 2( f ) y H 10( f ) . Dibuje
el espectro de H10(f). Diga que representan estos tres diagramas de bloques. Escriba la
expresin de las funciones c1(t), c2(t) y c10(t).
c) Calcule los valores de la amplificacin a1, a2, y a10 para que las amplitudes de las seales de
salida de los tres amplificadores tengan el mismo nivel.
d) Escriba las expresiones d(t) y D(f). Dibuje el mdulo de D(f).
e) Escriba las expresiones v(t) y dibuje el mdulo de V(f).
f) En consecuencia con el apartado anterior que describe la seal v(t) y su espectro, y
suponiendo que no dispone de ningn tipo de oscilador, proponga la primera accin que
realizara para obtener un seal sinusoidal que lleve a recuperar, ms tarde, la informacin
x(t) y(t) y z(t). Escriba la/s expresin/es en tiempo y en frecuencia del/los subsistema/s que
use.

160

g) En consecuencia con el apartado anterior dibuje el diagrama de bloques que permita


recuperar la informacin x(t) y(t) y z(t) , explique como opera, y escriba las expresiones en
tiempo y en frecuencia de todos los filtros ideales que use.

4.47.-

La

seal

de

entrada

al

diagrama

de

bloques

de

la

figura

es

z (t ) cos 2 f 0t 2 f d x(t ')dt ' . Se sabe que el ancho de banda de la seal x(t) es b Hz, fd=b,
0

f0>>b, T0=1/f0 y |x(t)|max=1. Tambin se conoce la respuesta al impulso h(t ) t T0 . Por


4

comodidad use y (t ) x(t ')dt '


0

z(t)

a1(t)

h1(t)

b1(t)

d(..)/dt

c1(t)

d1(t)

cos(2f0

+ e(t)
-

h(t)

a(t)

a2(t) h1(t)

b2(t)

d(..)/dt

c2(t)

d2(t)

h2(t)

X(f) 1
-b

Hz
f

a) Calcular la expresin de a(t).


b) Escribir las expresiones a1(t) y a2(t). Dibuje esquemticamente el espectro de amplitud de
de la seal a1(t) pero de forma que se ponga de manifiesto su ancho de banda ( si lo cree
necesario de una explicacin).
c) Obtenga la funcin de transferencia H 1( f ) y la respuesta impulsional h1(t) de los filtros
paso bajo ideales. Explique como elige su ancho de banda.
d) Escribir las expresiones b1(t) y b2(t),c1(t) y c2(t).
e) Escribir las expresiones d1(t), d2(t) y e(t).
f) Escribir la expresin e(t). Tambin obtener la respuesta impulsional h2(t) para que la seal
de salida m(t) sea exactamente x(t).
g) Encontrar m(t) si ahora z(t) fuera una modulacin de FM de banda estrecha. Es decir
z (t ) cos 2 f 0t 2 f d y (t ) sen 2 f 0t con fd<<b. No se olvide de explicar como elige el ancho

de banda de los filtros H1(f).


161

4.48.- Las seales de entrada x(t) e y(t) al diagrama de bloques tiene un ancho de banda
b=1/2T0 Hz cuyos espectros genricos X(f) e Y(f) estn representados en las figuras. Tambin
se

conocen

las

respuestas

al

impulso

h1(t ) 2 f 0 sinc f 0t cos 2 10 f 0t ,

h 2(t ) 2 cos 2 10 f 0t .

x(t)
y(t)

h(t)=j/t

h(t)=j/t

+
+
-
+

X(f) 1

s1(t)

s(t)

s2(t)

p (t )

m(t)

z1(t)

h1(t)

z(t)

-b

Hz
f

Hz
f

Y(f) 1

h2(t)

(t nT )

z2(t)

-b
Figura a

a) Escriba las expresiones de las funciones s1(t) y s2(t). Diga que representan. Escriba las
expresiones de las funciones S1(f) y S2(f). Dibuje sus grficas con ayuda de la figura a.
b) Escriba las expresiones de las funciones s(t) y S(f). Dibuje su espectro de amplitud.
c) Exprese p(t) en una suma de funciones sinusoidales. Tambin exprese P(f)
d) Escriba las expresiones m(t) y M(f) (hgalo en funcin de s(t) y S(f)). Dibuje el mdulo de
M(f).
e) Escriba la expresin de las funciones H 1( f ) y H 2( f ) . Dibuje sus espectros de amplitud.
Que representan estos dos diagramas de bloques. Escriba la expresin de la funcin z2(t).
f) Escriba la expresin z1(t). Finalmente escriba la expresin z(t) y dibuje el mdulo de Z(f).
Es z(t) una seal real?
g) En consecuencia con el apartado anterior que describe la seal z(t) y su espectro, y
suponiendo que no dispone de ningn tipo de oscilador, proponga la primera accin que
realizara para obtener un seal sinusoidal como inicio de la recuperacin de las seales
originales. Dibuje el diagrama de bloques que permita recuperar la informacin x(t) e y(t),
explique como opera, y escriba las expresiones en tiempo y en frecuencia de todos los filtros
ideales que use.

4.48.- La Figura a representa el diagrama de bloques de un modulador de fase de banda


estrecha que contiene un multiplicador de frecuencia por 10000. Se sabe que
162

T
h(t ) t 0 ,
4

y (t ) x (t ')dt ' , que 2 f d y (t ) max 1 radian, la constante de


0

desviacin de frecuencia es fd=b/100 y el espectro genrico de la seal x(t), que est


normalizada a 1, es el que se muestra en X(f).
Por otra parte, la Figura b representa el diagrama de bloques de un demodulador de frecuencia
que contiene un comparador (si la amplitud de entrada es positiva la salida vale 1 voltio si es
negativa -1 V) y un filtro paso bajo ideal con respuesta impulsional h1(t).
2fdy(t)

xFME(t)

multiplicador
f x 1000

xFM(t)
X(f)

h(t)

A sen(2 f 0t )

xFM(t)

d(..)/dt

Figura a

d(t)

m(t)

-b
z(t)

h1(t)

1
b

Hz
f

s(t)

Figura b

a) Escriba la expresin de la funcin xFME(t), siguiendo estrictamente el diagrama de bloques,


con objeto de mostrar que se puede asociar a una modulacin de amplitud. Dibuje su espectro.
b) Segn el apartado anterior, si tuviera que recuperar la seal x(t) a partir de xFME(t), dibuje
un diagrama de bloques que lo muestre y escriba las expresiones que crea oportunas para
apoyar su eleccin
c) Reescriba la expresin del apartado a) de forma que muestre que xFME(t) es tambin una
modulacin en frecuencia. Escriba las constantes de desviacin de fase y de frecuencia. Haga
una estimacin razonable de su ancho de banda.
d) Escriba la expresin de la funcin xFM(t). Obtenga la constante de desviacin de frecuencia.
Si tuviera que filtrar la seal de salida xFM(t) para estandarizarla que ancho de banda usara.
e) Ahora respecto a la Figura b: Escriba la expresin d(t). Escriba m(t) hasta el tercer
armnico.
f) Escriba z(t) hasta el tercer armnico. Escriba la expresin de la respuesta al impulso h1(t) y
la funcin de transferencia H1(f) del filtro que conduzca a la demodulacin de x(t).

163

g) Suponga que x(t) es una seal estereofnica estndar. Dibuje la distribucin de bandas del
espectro de amplitud de esta seal y proponga los dos diagramas de bloques que conduzcan a
generar y despus a recuperar los dos canales. De una explicacin razonable de cada bloque
relacionada con el espectro.

4.49.p(t )

Se

efecta

un

muestreo

no

ideal

de

la

seal

x(t)

con

la

funcin

t nT0

para obtener la seal muestreada xs(t). Despus se procesa esta
n T0 2

seal xs(t) tal como se muestra en la figura a). Se sabe que f0>>b y que h(t) es la respuesta al
impulso de un filtro paso banda ideal centrado en 3f0 Hz y de ancho 2b Hz. Tambin se sabe
que h1 (t ) k (t ) y h2 (t ) t T0 12 . Los espectros de las seales de entrada x(t) e y(t)
tienen un ancho de banda b Hz, y pueden tomarse para fines grficos tal como se representan
en la figura b).
x(t)

h(t)
h(t)

p(t)

h1(t)
h2(t)

a(t)

b(t)
Figura a

X(f)

v(t)

y(t)

s(t)

-b
Y(f)

Figura b

Hz
f

Hz
f

-b

a) Opere la seal xs(t) y exprsela de forma que contenga funciones senoidales.


b) Escriba la expresin de la funcion Xs(f). Dibuje esquemticamente su espectro de amplitud.
c) Escriba la expresin de las funciones h(t) y H ( f ) . Dibuje su espectro de amplitud.
Exprese tambin H1(f) y H2(f) y dibuje sus espectros de amplitud y de fase.
d) Escriba las expresiones de v(t) y V(f). Dibuje su espectro de amplitud.
e) Escriba la expresin de s(t). Que margen de frecuencias ocupa en el espectro. Aydese
con un dibujo esquemtico de la grfica de su espectro de amplitud.
f) Suponiendo que no dispone de ningn tipo de oscilador, proponga la primera accin que
realizara para obtener una seal sinusoidal como inicio de la recuperacin de las seales
originales. Dibuje el diagrama de bloques que permita recuperar la informacin x(t) e y(t),
164

explique cmo opera, y escriba las expresiones en tiempo y en frecuencia de todos los filtros
ideales que use.

4.50.-

La

entrada al diagrama de bloques de la figura es


z (t ) A cos 2 f 0t 2 k x (t ')dt ' . Se sabe que el ancho de banda de la seal x(t) es b Hz, k=b,
0

seal

de

1
t .
A2 2 k

f0>>b, y |x(t)|max=1. Tambin se conoce la respuesta al impulso h2 (t )

Por

comodidad use y (t ) x(t ')dt '


0

espectro genrico de x(t)


X(f)

h(t )

d (t )
dt

b2(t)

z(t)

b3(t)

h1 (t )

1
t

a1(t)

h(t )

d (t )
dt

a2(t)

b1(t)

-b

b(t)

h2(t)

Hz
f

m(t)

f0 /k

a3(t)

a) Calcular la expresin de a1(t).


b) Demuestre que la seal de salida m(t) es igual a x(t).

4.51.- La figura a) muestra la grfica de una seal peridica con periodo de repeticin T0
segundos. La figura b) muestra, en lnea gruesa, la seal generatriz basada en una funcin
senoidal.

Figura b)
p(t)

1
-T0/4

............
-2T0

Figura a)

g(t)

-T0/4
-T0

0 T0/4

T0

.............
2T0
t s

T0/4

T0

Se pide:
a) Escriba la expresin de P(f) y dibuje esquemticamente su mdulo.
b) La seal x(t) tiene un ancho de banda b Hz, h2(t) es la respuesta impulsional de un filtro
paso banda ideal de b Hz centrado en 2f0 Hz y f0>>b. Obtener las seales c(t), d(t), e(t), m(t),
r(t) y final s(t). (1.5)
165

x(t)
p(t)

c(t)

h2(t)

h(t)=1/t

r(t)

h(t)=1/t
-

e(t)

d(t)

h1(t)=(t-T0/4)

h2(t)

p(t)

m(t)

s(t)

c) Dibuje un diagrama de bloques que permita recuperar la seal x(t) (a efectos grficos use el
espectro genrico X(f) del ejercicio 2) a partir de la seal s(t). Especifique las respuestas al
impulso y funciones de transferencia de los bloques que use.

4.52.- Se desea sustituir, mediante una aproximacin, un bloque integrador (figura 1) por dos
bloques pero que uno de ellos sea un derivador como en la figura 2.
x(t)

y (t ) x(t ')dt '


0

y(t)

h(t)

a
x(t)

y2 (t )

figura 1

figura 2

x2(t)

d x2 (t )
dt

y(t)

y2(t)

Teniendo en cuenta que x(t) es una seal modulada en amplitud a la frecuencia de portadora f0
se pide:
a) Primero, dibujar esquemticamente el espectro de amplitud y de fase de la funcin de
transferencia del bloque integrador (figura 1).
b) Usando el desarrollo en serie de Taylor de una funcin f(z) en un punto z=a, esto es:
f ( z ) f (a ) f '(a )( z a )

f ''(a )
( z a ) 2 ........ y tomando solo los dos primeros trminos, calcule
2!

las constantes a, b y la respuesta impulsional h(t) correspondientes, para obtener esta


aproximacin.

166

CAPTULO 5
5 MODULACIONES DE PULSOS

5.1 INTRODUCCIN

Cuando se muestrea idealmente una seal x(t) con el propsito de ser transmitida, cabe
preguntarse por el tipo de modulacin que va a ser usada, dada la imposibilidad fsica del
manejo de las funciones delta. Una posibilidad, es la manipulacin por las muestras que
conforman el mensaje, de alguna de las caractersticas de un tren peridico de pulsos.
Los tres tipos estandarizados principales se designan por lo general como modulacin en

amplitud de pulsos (PAM), modulacin en duracin de pulsos (PDM) y modulacin en


posicin de pulsos (PPM).
x(t)

T=periodo de muestreo

PAM

PDM

PPM

Tipos de modulacion analogica de pulsos

Figura 5.1. Enumeracin de tipos de modulacin analgica de pulsos


Como se puede observar en la figura, el parmetro que varia la caracterstica del pulso
(amplitud, duracin o posicin), lo hace en proporcin directa a la seal moduladora. Sin
embargo, en la modulacin de pulsos en amplitud y en duracin, los valores muestra que son
iguales a cero se suelen representar por medio de una amplitud o duracin diferentes de cero.
Esto se hace para conservar una velocidad de pulsos constante, cosa til para propsitos de
sincronizacin.

167

5.2 MODULACIONES ANALGICAS DE PULSOS

Se pueden establecer tres observaciones generales acerca de la modulacin analgica de


pulsos.
Las ondas moduladas por pulsos tienen componente continua y un contenido de baja

frecuencia apreciables, en especial cerca de los primeros armnicos de fs= 1/Ts. Por lo
que la transmisin directa puede ser difcil o imposible.
Cuando se transmiten las ondas moduladas por pulso, se debe tener cuidado en prevenir

el traslape de los pulsos, ya que se destruira la informacin. El ancho de banda


requerido es mayor o igual a 1/2 siendo la duracin nominal del pulso.
De manera conceptual los valores muestra originales se extraen de la onda modulada,

convertidos en impulsos ponderados y filtrados paso bajo.


Con respecto al primer punto, la transmisin de pulsos a corta distancia puede ser hecha
sobre circuitos almbricos o por cable coaxial, pero una transmisin eficiente requiere una
traslacin de frecuencia adicional. En consecuencia muchos sistemas de pulsos tienen un paso
de modulacin, en el cual los pulsos se convierten a pulsos de radio frecuencia (si xp(t) es la
onda modulada por pulsos la seal que se transmite es xc(t)=xp(t)Accos(wct)),generalmente a
doble banda lateral.
Dado que se escoge la duracin del pulso mucho menor que el tiempo entre ellos (<<Ts) y
Ts1/2W con W el ancho de banda de la informacin, el ancho de banda de la modulacin de
pulsos (transmisin en banda base BTBB) resulta ser mucho mayor que W (BTBB1/2>>W)
,teniendo en cuenta adems que si se modula en doble banda lateral (ancho de banda BTrf)
entonces se duplica BTBB (BTrf=2BTBB). Resumiendo, el ancho de banda que se requiere es
grande en comparacin con el ancho de banda del mensaje, pero resulta ventajoso en cuanto a
reduccin de ruido. Sin embargo cuando no se precisa mantener la forma del pulso, se puede
obtener una drstica reduccin en el ancho de banda de la seal modulada, hasta el punto de
conseguir eficiencias semejantes a la modulacin de banda lateral nica. Esto se pondr de
manifiesto en los apartados siguientes, muy concretamente al hablar de multiplexado por
divisin en el tiempo (MDT).

168

Se suele utilizar poco la modulacin analgica de pulsos en amplitud en transmisin de


mensajes, pero es utilizado en sistemas de control o como un paso intermedio en la
reconstruccin del mensaje de otros tipos de modulaciones.

5.2.1 MODULACIN EN AMPLITUD DE PULSOS (PAM)


La forma de onda usual est compuesta de pulsos unipolares y no necesariamente deben
ser rectangulares. La expresin general de PAM es:
x p ( t ) A0 1 mx ( kt s ) p ( t kTs )

(5.1)

donde A0 es la amplitud del pulso no modulado, p(t) es la forma del pulso y m es el ndice
de modulacin (comparable a AM).
Encontrar el espectro de esta expresin es un ejercicio sencillo de aplicacin que, adems
de cuantificar el ancho de banda, permitir hacer algunas puntualizaciones.

Ejercicio

Los dos diagramas de bloques en las figuras siguientes representan dos moduladores PAM:
xs(t)

x(t)
p(t)

modulador PAM sin cresta plana

x(t)

h(t)

ys(t)

i(t)
modulador PAM con cresta plana

t nT0
t
Con p(t )
con T0 3 , i ( t ) ( t nT0 ) ,y h(t )


n
n

f
Con objeto de poder dibujar grficas, consideraremos el espectro X ( f ) de la
B
seal a modular, adems la frecuencia de muestreo f0=1/T0=2B ser la justa; el doble del
ancho de banda de la seal a modular.
Se desea:
a.- Encontrar las expresiones de xs(t) y ys(t). Encontrar tambin las expresiones de Xs(f),
Ys(f) y dibujar sus grficas respectivas.

169

b.- Saber si son iguales las seales moduladas xs(t) y ys(t). En caso de no serlo cual es su
diferencia.
c.- Recuperar exactamente x(t) para cada uno de los sistemas de modulacin. Hacer un
diagrama de bloques para cada uno y escriba las expresiones correspondientes de cada
subsistema.
Solucin:
a.

t nT0
j 2 nf 0t
(
)
xs (t ) x(t )

x
t
f
0 Sinc ( nf 0 )e


n
n

X s ( f ) X ( f ) f 0 Sinc( nf 0 ) ( f nf 0 )
n

X s ( f ) f 0 Sinc( nf 0 ) X ( f nf 0 )
n

1
n
Sinc( ) X ( f nf 0 )

3 n
3

Xs(f)

f0

2f0

3f0

4f0

Para la seal ys(t).

t
t nT0
ys (t ) x(t ) (t nT0 ) x(nT0 )

Ys ( f ) X ( f ) f 0 ( f nf 0 ) Sinc( f )
n

1
f

Ys ( f ) f 0 X ( f nf 0 ) Sinc( f ) Sinc(
) X ( f nf 0 )
3
3 f 0 n
n

Fs(f)

f0

2f0

3f0

4f0

b.- La diferencia es que en el modulador PAM con cresta plana el espectro peridico de la
seal est multiplicado por una funcin Sinc que distorsiona cada uno de los espectros
parciales o de informacin, no ocurriendo lo mismo para el modulador PAM sin cresta plana
en el que nicamente solo se ven afectados por un factor de escala.
c.- Para el modulador PAM sin cresta plana basta un filtro paso bajo de ancho de banda B.

170

>

G(f)

>

f
f
G ( f ) 3
3
2B
f0

Para el modulador PAM con cresta plana es necesario compensar la distorsin en la banda
baja haciendo pasar la seal muestreada por el inverso del Sinc y despus filtrar paso bajo, es
1
f
decir si se filtra solo paso bajo no se recupera x(t) sino Sinc(
) X ( f ) , por lo tanto debera
3
3 f0
filtrar con:

f
1
f
f 3 f0
G( f )
3
1

2 B Sinc( f )
2 B Sen( f )
3
3 f0
3 f0
G(f)

-B

Del ejercicio anterior se desprende que el ancho de banda de una seal modulada en PAM
es infinito y por tanto, en teora, el canal y los elementos de transmisin y de recepcin
necesarios tambin deberan serlo. Esta cuestin se abordar mas adelante.
Surge ahora la razonable pregunta, Porque hay inters en que los pulsos tengan cresta
plana?. La respuesta es que no es necesario utilizar pulsos con cresta plana, otro tipo de pulso
conduce igualmente la informacin, es ms, los pulsos con cresta plana distorsionan el
espectro de la seal de informacin. Sin embargo la razn de peso que aboga por los pulsos de
cresta plana es de orden prctico; se precisa una circuitera mucho ms sencilla y econmica
para generarlos. Adems, cuando las seales se transmiten a distancias largas, se necesitan
repetidores que compensen las atenuaciones producidas. Estos repetidores deberan amplificar
y filtrar el ruido que se ha ido aadiendo a la seal modulada. Ahora bien como la forma del
pulso no es importante estos repetidores nicamente deben regenerar los pulsos en lugar de
amplificarlos, es decir, crear uno nuevo. Por ejemplo puede regenerarse un nuevo pulso si su
amplitud se hace proporcional al rea, detectada en un ancho o intervalo de tiempo fijo, del
pulso de entrada, y una vez ms prima la sencillez en la circuitera.

171

5.2.2 MODULACIN PULSOS EN DURACIN (PDM)


La duracin de un pulso ((k)) es proporcional a la muestra del mensaje, as:
( k ) 0 1 mx ( kTs ) , con lo que la seal es:
t kTs
x p (t ) A0 p

k
( k ) / 0

(5.2)

Con p(t) un pulso rectangular.


El espectro de xp(t) es difcil de encontrar pero si 0<< Ts entonces se puede aproximar
por el de PAM con idntica demodulacin.

x(t)

diente de sierra
periodo T de muestreo

nivel recorte
x(t)+diente de sierra

Seal PDM
t

Figura 5.2. Generacin de la seal PDM


En la figura se puede ver un diagrama de seales de generacin de PDM.

5.2.3 MODULACIN DE PULSOS EN POSICIN (PPM)


La expresin de la modulacin PPM es:
x p ( t ) A0 p ( t t k ) A0 p ( t ) ( t t k )
k

Con

t k kTs t 0 x ( kTs )

La expresin de su espectro es tambin complicada. Consiste en bandas laterales de AM y


PM centradas en todos los mltiplos de fs. La mejor forma de demodular es convertir a PAM.
172

5.3 MULTIPLEXADO POR DIVISIN EN EL TIEMPO

El uso de pulsos muy estrechos en el tiempo de una seal PAM deja suficiente espacio
entre ellos para la insercin de pulsos procedentes de otras seales PAM.
La figura siguiente muestra dos seales x1(t) y x2(t), cada una de ellas muestreada a T0
segundos y moduladas en PAM, que han sido combinadas formando un entrelazado.

x1(t)

T0
x2(t)

2T0

t
T0/2

Figura 5.3. Combinacin de dos seales PAM


La seal resultante representa otra seal, muestreada a una frecuencia doble que la usada
para muestrear cada una de sus componentes, modulada en PAM. Es obvio que el mtodo
puede aplicarse, en teora, a un nmero ilimitado de componentes. En general, el multiplexado
por divisin en el tiempo (MDT) se aplica, por razones de eficiencia, a seales paso bajo y
con el mismo ancho de banda.
La seal as multiplexada podra enviarse por una lnea de transmisin o bien utilizarse
como moduladora en algn tipo de modulacin de onda continua. Sin embargo su ancho de
banda es tericamente infinito, y por tanto se debe decidir cuanto ancho de banda es
realmente necesario para transmitir la informacin y recuperarla correctamente despus en el
receptor.
La grfica siguiente muestra la modulacin PAM, y la correspondiente seal x(t)
moduladora, resultante de la multiplexacin por divisin en el tiempo (MDT) de las
modulaciones PAM de las seales x1(t) y x2(t) de la grafica anterior.

173

x(t)

t
T0/2

Figura 5.4. MDT de dos modulaciones PAM y seal moduladora resultante x(t).
En general, si se multiplexan en el tiempo las modulaciones PAM de N seales (x1(t),
x2(t),....... xN(t)), cada una de ellas con una frecuencia de muestreo de f0 =1/T0 Hz, entonces su
ancho de pulso debe ser menor o igual T0/N.
El espectro YM(f) de las modulaciones PAM multiplexadas en funcin del espectro de la
seal moduladora resultante X(f) ser:

YM ( f ) Nf 0 Sinc( f ) X ( f nNf 0 )
n

y su representacin grfica,
YM(f)

Sinc(f)

-3Nf0 -2Nf0

X(f) Sinc(f)

-Nf0

Nf0

2Nf0

3Nf0

(Hz)

Figura 5.5. Espectro de la seal PAM multiplexada.


Representa que la seal multiplexada x(t) est muestreada a Nf0 Hz y por tanto tiene un
ancho de banda de Nf0/2 Hz.

Consideraciones del MDT en el transmisor

Como se estar interesado en transmitir una seal muntiplexada, ocupando el menor ancho
de banda posible, est claro que debera enviarse la seal analgica x(t) cuyo espectro es X(f).
El diagrama de bloques siguiente representa un multiplexor por divisin en el tiempo, con
174

pulso de cresta plana. Las seales a multiplexar (y0(t), y1(t), y2(t),. yN-1(t)) modulan en
PAM a las seales x0(t), x1(t), x2(t),. xN-1(t). Es de notar que los impulsos de muestreo en
cada modulador PAM esta retardado segundos con respecto al impulso del modulador de la
rama superior con objeto de permitir el entrelazado de las seales PAM cuando se combinen
en el sumador.

i(t )

i(t)

x0(t)

( t nT )
0

h=(t/)

y0(t)

x1(t)
y1(t)

h1=(t-)

h=(t/)
x2(t)

h2=(t-2)

h=(t/)

.
.
.
.

y2(t)

xN-1(t)

Lnea de
transmisin

yN-1(t)

hN=(t-(N-1))

h=(t/)

hb(t)
yM(t)

Figura 5.6. Diagrama de bloques de un multiplexor de seales PAM.


Obviamente, el espectro YM(f) de las seal yM(t) es el que se muestra en la grfica 5.5. As
para transmitir la informacin con el menor ancho de banda y que posteriormente sea
recuperable, es necesario filtrar paso bajo de 0 Hz a Nf0/2 Hz, es decir suponiendo un filtro
f
ideal su espectro ser H b ( f )
y su respuesta impulsional hb (t ) Nf 0 Sinc( Nf 0t ) .
Nf 0
Hb(f)= (f/Nf0)

-Nf0/2

YS(f)
X(f) Sinc(f)

Nf0/2

(Hz)

Figura 5.7. Espectro de la seal de salida del multiplexor, filtrada paso bajo ideal.
175

Como se ha usado el tipo de modulador PAM con pulso de cresta plana, la seal que se
entrega a la lnea tiene un espectro X(f) Sinc(f), y por tanto distorsionada en el tiempo. La
conveniencia de usar o no usar pulso con cresta plana vendr impuesto por el contexto en el
que se utilice esta modulacin; ms por las caractersticas del emisor o del receptor y de los
repetidores que por el propio medio en el que se produce la transmisin. Ntese que lo que
importa es el ancho de banda a transmitir sin que resulte importante la forma del pulso (es
ms, no se transmite ningn pulso por el medio).
A modo de ejemplo, si se hubieran multiplexado dos canales como en la figura 5.4, la seal
que realmente se transmite por la lnea sera la seal x(t) que se muestra en trazo discontinuo
en la mencionada figura 5.4.

Consideraciones del MDT en el receptor

En el receptor la seal multiplexada x(t) debe ser de nuevo muestreada a la frecuencia con
que se efectu el MDT en el transmisor y separar estas muestras en los canales originales.
Esta operacin no ofrece ninguna dificultad, salvo la sincronizacin y la asignacin de la
verdadera etiqueta a cada canal. Un diagrama de bloques se muestra en la figura siguiente, en
la que hb(t) es la respuesta impulsional de un filtro paso bajo con el mismo ancho de banda de
la seal MDT transmitida, cuyo objeto es eliminar el ruido, fuera de la banda requerida, que
se ha podido aadir durante la transmisin. La seal de salida x(t) de este filtro es la seal
x(t) transmitida mas el ruido aadido en la banda. Los filtros paso bajo con respuesta
impulsional ha(t) tienen un ancho de banda N veces mas pequeo que el ancho de banda de la
seal x(t).
Para volver a muestrear la seal x(t) es indispensable disponer de sincronizacin con el
emisor, con objeto de conocer a que canal corresponde cada muestra. Si se dispusiera de un
oscilador a la misma frecuencia, pero no existiera sincronizacin con el emisor, no habra
ninguna dificultad en volver a muestrear y obtener la seal x(t), pero no las muestras de cada
canal y tampoco habra forma de saber cual es cual. Es decir se necesita una referencia para
poder etiquetar correctamente los canales. Esta referencia debe estar incluida de alguna forma
en la seal que se recibe. Hay varias formas de hacerlo, cada una con sus correspondientes
ventajas e inconvenientes.

176

Linea de
transmisin

i(t )

Filtro paso bajo


hb(t)

( t nT )

x(t)
Filtro paso bajo

ha(t)

x0(t)

hr(t)=(t-)

ha(t)

x1(t)

hr(t)=(t-2)

ha(t)

x2(t)

ha(t)

xN-1(t)

i(t)

Oscilador sincronizado con el


emisor

Retardos

.
.
.
.
hr(t)=(t-(N-1))

Figura 5.8. Diagrama de bloques del receptor del MDT de seales PAM.

Aadiendo sincronizacin en el emisor

Una forma es enviar las muestras de un canal amplificadas; por ejemplo 10 veces mayor
por el canal 1. As en la seal recibida aparecern siempre unos picos que correspondern a
los instantes de ocurrencia del canal uno.
Otra forma mejor, aunque ms elaborada y por tanto requiere mas circuiteria, consiste en
habilitar otro canal, llamado de control, con el mismo ancho de banda que el resto de canales
para introducir cdigos, seales de supervisin etc, o simplemente para introducir un pulso de
mayor amplitud como en el caso anterior.
Una tercera va consiste en aadir un tono al final del espectro de la seal x(t) que se
transmite. Es decir modular en amplitud una portadora con un pulso que comience en el
mismo instante en que se toma la muestra de, por ejemplo, el canal uno. Esto representa una
combinacin del multiplexado por divisin en el tiempo, para la informacin y el
multiplexado por divisin en frecuencia (MDF) para el control.

177

La figura siguiente muestra el diagrama de bloques de un MDT al que se le ha aadido la


funcin de control mediante este ltimo mtodo.

i ( t ) ( t nT0 )

x0(t)

y0(t)

i(t)

h(t)=(t/)
x1(t)
y1(t)

h1(t)=(t-)

h(t)=(t/)
x2(t)
y2(t)

h2(t)=(t-2)

.
.
.
.

h(t)=(t/)

xN-1(t)

hN(t)=(t-(N-1))

yN-1(t)

h(t)=(t/)

Linea de
transmisin

yM(t)
hb(t)

yS(t)
PM(t)
h(t)=(2t/T0)

Yp(f)

hc(t)=2f0sinc(2f0t) cos(2fct)
cos(2fct)

Figura 5.9. Diagrama de bloques del emisor MDT de seales PAM con control por MDF.
Es sencillo comprobar que la seal pM(t), que se muestra en la figura 5.10, es un tren de
pulsos a frecuencia f0 Hz ( frecuencia de muestreo de un solo canal) que modula a una
portadora de frecuencia mucho mas alta fc y solo ligeramente mas alta que Nf0/2. Los instantes
de inicio de estos pulsos coinciden con los instantes de muestreo del canal uno. Si se elige el
mayor ancho de pulso en el tiempo (T0/2) entonces se obtendr el espectro mas estrecho
posible.
pM(t)
T0/2
A
-A

.
0

.
t

T0

.
(s)

2T0

Figura 5.10. Tren de pulsos de control modulados en AM

178

Como ya es sabido el espectro de esta seal modulada es el espectro de la seal


moduladora, desplazado a la frecuencia de la portadora fc. El espectro del tren de pulsos es
P( f )

1
n
Sinc ( f nf 0 ) con el primer armnico en f0 y el segundo a 3f0, y el espectro

2
2

del tren de pulsos modulado es PM ( f )

1
n
Sinc ( f nf 0 f c ) .

2
2

Es suficiente conservar el primer armnico para determinar la seal, cometido que realiza
f fc
f fc
el filtro paso banda cuyo espectro es H c ( f )

y entonces fc= Nf0/2+f0
2 f0
2 f0

Hz.
X(f) Sinc(f)

YS(f)

-Nf0/2

Nf0/2

fc

(Hz)

B
Figura 5.11. Espectro de la seal a transmitir
La seal que se transmite tiene ahora el espectro mostrado en la figura 5.11, y su ancho de
banda es Nf0/2+ 2 f0 Hz. Ntese que la eficiencia de este mtodo de control, en cuanto a
economa de espectro, aumenta cuando lo hace el nmero de canales.

Uso de la sincronizacin en el receptor

Una vez se ha establecido como est codificada y donde se encuentra la seal de


sincronismo, las acciones a realizar en el receptor estn determinadas.
La primera tarea en el receptor consistir en separar por filtrado la banda de la seal que
lleva la informacin y la banda de la seal que contiene el sincronismo. En el receptor,
prcticamente no hay modificaciones, salvo en la etapa generadora de impulsos. Es preciso
recoger la seal de sincronismo con un filtro paso banda centrado a la frecuencia fc y con un
ancho de banda 6 fc. Como es una seal modulada en amplitud es adecuado detectar la
envolvente, que no es mas que el tren de pulsos de sincronismo con cierta distorsin, ya que
se han usado solo unos pocos armnicos para transmitirlo. As con una sencilla etapa que
179

regenere estos pulsos y cree impulsos en el instante de comienzo de cada uno de ellos se
dispondr de la seal i(t) en la que cada impulso indica que la muestra en curso pertenece al
canal uno.

Lnea de
transmisin

Filtro paso bajo

x(t)

hb(t)

Filtro paso bajo

Filtro paso banda


hban(t)
Detec. Envolvente

ha(t)

x0(t)

Retardos

Conformador de
pulsos

hr(t)=(t-)

ha(t)

x1(t)

Generador de
impulsos

hr(t)=(t-2)

ha(t)

x2(t)

ha(t)

xN-1(t)

i(t )

.
.
.
.
hr(t)=(t-(N-1))

( t nT )
0

Figura 5.12. Diagrama de bloques del receptor MDT de seales PAM con control por
MDF.
La figura anterior muestra el proceso que se ha comentado.

Filtrado real

Cuando el oscilador usado para realizar el muestreo en PAM presenta derivas, es decir que no
es estable con la frecuencia i (t )

(t nT

n ) haciendo la suposicin que n es una

variable aleatoria con media cero, funcin de densidad de probabilidad p n ( n ) y con


transformada de Fourier P n ( f ) , la expresin de salida de la seal PAM, con pulso de cresta

180


t
plana, se escribe ys (t ) x(t ) t nT0 n y constituye un proceso aleatorio. En
n

el supuesto que esta distribucin no cambie con el tiempo, y poder as omitir el subndice n
P n ( f ) P ( f ) ,

entonces

se

puede

demostrar

que

el

espectro

estimado

es


Y s ( f ) X ( f ) P ( f ) f 0 f nf 0 Sinc( f ) . Para modulacin PAM con pulso
n

de

cresta

no

plana

el

espectro

estimado

es

Y s ( f ) X ( f ) P ( f ) Sinc( f ) f 0 ( f nf 0 ) . En ambos casos el espectro de la seal a


n

modular X(f) se encuentra convolucionado con un tren de impulsos cuyas reas tienen que ver
con la funcin de distribucin, transformada por Fourier, de la variable aleatoria n.

t
La estimacin en el tiempo de las seales PAM es y s (t ) x(t ) p t nT0 con
n


t
cresta plana y y s (t ) x(t ) p (t nT0 ) con cresta no plana.

n
En general, el significado en el tiempo, tal como se desprende de las expresiones
anteriores, es que la seal x(t) se muestrea con una funcin peridica que tiene una forma de
pulso igual a la forma de la funcin de distribucin. Por tanto el efecto neto producido al
muestrear con osciladores no estables en frecuencia es una variacin en la forma del pulso de
muestra.
Pero esto es, como se ha dicho, el efecto neto o valor medio, producido por la acumulacin
de infinitas realizaciones del proceso aleatorio.
Los filtros paso bajo utilizados al final de las etapas de muestro se han considerado ideales,
esto es, su espectro de amplitud es un pulso centrado en el origen cuyo ancho es el doble del
ancho de banda de la seal a tratar. Su respuesta impulsional es una funcin Sinc, cuya
amplitud es el valor del impulso, tiene duracin infinita y vale cero en los instantes que son
mltiplos del periodo de muestreo T0. As cuando a este filtro entra una secuencia de
muestras, cuya separacin entre cada una de ellas es exactamente T0 (se ha muestreado la
seal con un oscilador estable) se obtiene a la salida una secuencia, que se solapa, de
funciones Sinc centradas en cada mltiplo de T0. Adems, como las colas de cada funcin
Sinc valen cero en todos los instantes de muestreo, excepto en el suyo propio, en cada uno de

181

ellos la amplitud de la funcin resultante de salida es el correspondiente valor de la muestra.


Se dice que existe interferencia cero entre pulsos.
Alcanzar la condicin de interferencia cero entre pulsos requiere disponer de filtros paso
bajo ideales y una sincronizacin precisa entre emisor y receptor, y adems el medio por el
que se transmite sea un sistema paso bajo ideal.
En la prctica los errores de sincronizacin entre emisor y receptor son difciles de eliminar
y un filtro ideal es fsicamente irrealizable. El resultado es que cada pulso recibido se ver
afectado por la presencia de los pulsos adyacentes, efecto que se conoce como interferencia
entre smbolos (IES).
Se trata, pues, de disear un filtro orientado a reducir la IES. Es necesario un filtro cuya
respuesta impulsional conserve los ceros del Sinc, a la vez que se suavizan los flancos de su
espectro a fin de hacerlo realizable.
La funcin siguiente representa expresin del espectro del filtro paso bajo llamado coseno
elevado.
T0

T
H( f ) 0
2

0 f (1 ) B

1 sen 2 B ( f B)

(1 ) B f (1 ) B
f (1 ) B

En esta expresin, B representa el ancho de banda de la seal a tratar, T0=1/2B es el


periodo de muestreo, y es un parmetro variable entre 0 y 1 que suaviza los flancos del
espectro de amplitud del filtro. La grfica de este espectro se presenta seguidamente.
H(f)

=0

T0

=0.5
=1

-2B

-B

2B

Figura 5.13. Espectros del filtro coseno elevado en funcin del parmetro .

182

La respuesta impulsional de este filtro es h(t ) (

sen(2 Bt cos(2 Bt )
)(
).
2
2 Bt
1 4 Bt

h(t)
=0
=0.5
=1

-3T0/2

T0

2T0 3T0 4T0 5T0

Figura 5.14. Respuesta impulsional del filtro coseno elevado en funcin del parmetro .
Ntese que la respuesta impulsional de este filtro conserva la posicin de los ceros como el
filtro paso bajo ideal; de hecho para =0 es el filtro paso bajo ideal. Tambin conviene notar
que para =1 tiene ceros aadidos a la mitad del periodo de muestreo, y la amplitud de los
lbulos laterales se ha reducido drsticamente, lo que implica poca interferencia entre
smbolos.
La expresin del espectro para =1 es bastante compacta y se escribe:

T0

H( f ) 2
0

f
1 cos 2 B

f 2B
f 2B

Tampoco es difcil de implementar fsicamente.

Ejercicio
Demostrar que la respuesta impulsional del filtro coseno elevado puede hallarse como la
transformada inversa de Fourier de la convolucin de las funciones dadas en las grficas
siguientes:
P(f)
T0
Z(f)

-B

-B

183

2 f f
f
Siendo Z ( f ) cos

y P ( f ) T0

4 B 2 B
2B
Solucin.

2 f f
f
La convolucin de las dos funciones es H ( f ) cos

T0
. Su
4 B 2 B
2B

transformada inversa de Fourier ser z(t)p(t). Primeramente se hallar z(t).


f

2 f
1
z (t ) TF 1 cos
TF
. As z(t) se desarrollar como sigue.
4 B

2 B

1
1
z (t ) (t
) (t
) 2 B Sinc(2 Bt )
2
4 B
4 B

1
1

z (t ) B Sinc 2 B(t
) Sinc 2 B(t
)
4 B
4 B

1
1

sen 2 B (t 4 B ) sen 2 B (t 4 B )

z (t ) B
1
1
2 B (t
)
2 B (t
)

4 B
4 B

1 sen 2 Bt 2 sen 2 Bt 2

z (t )
2
4 Bt 1
4 Bt 1

z (t )

z (t )

cos 2 Bt
1 4 Bt

h(t ) T0 2 BSinc(2 Bt )

h(t )

1 cos 2 Bt cos 2 Bt

2 4 Bt 1
4 Bt 1

As

Finalmente:

cos 2 Bt
1 4 Bt

sen(2 Bt ) cos 2 Bt

2
2 Bt
1 4 Bt

184

En resumen, si bien se pueden realizar filtros que disponen los ceros de la respuesta
impulsional como en un filtro ideal, no es posible evitar la interferencia entre pulsos porque
los osciladores que muestrean las seales no son totalmente estables. Sin embargo, con filtros
adecuados y buenos osciladores esta interferencia puede reducirse considerablemente. Por
otra parte se ha visto que el efecto producido por la falta de estabilidad de los osciladores en
la deteccin de pulsos es modificar la forma stos, hacindolos semejantes a la funcin de
distribucin de las desviaciones temporales que se producen entre el instante de muestreo
correcto y el real. Esta informacin sera muy til en caso de estar interesados en la deteccin
de los pulsos (y no de la seal muestreada), por que podra disearse un filtro con una
respuesta impulsional con la misma forma del pulso, rotada especularmente, que se pretende
detectar. Esto hace la deteccin muy eficiente y es precisamente el concepto de filtro
adaptado.

5.4 MODULACIN POR PULSOS CODIFICADOS

Los tres tipos de modulacin anteriores, son representaciones analgicas del mensaje. La
modulacin por pulsos codificados (PCM) es una modulacin digital en la que el mensaje se
representa por medio de un grupo codificado de pulsos con amplitudes discretas.
En la modulacin analgica, el parmetro modulado vara de forma continua y puede
tomar cualquier valor de los correspondientes al intervalo del mensaje.
Cuando la onda modulada se altera con ruido, no existe en el receptor forma de distinguir
el valor transmitido exacto. Sin embargo, si se permiten solo valores discretos para el
parmetro que modula y si la separacin entre estos valores es grande en comparacin con las
perturbaciones de ruido, es posible en el receptor decidir con precisin los valores que fueron
enviados, eliminndose los efectos del ruido aleatorio, lo cual constituye el objetivo de la
modulacin PCM. No se debe olvidar la facilidad que reportan los repetidores regenerativos
en la transmisin a largas distancias.

5.4.1 CUANTIFICACIN Y CODIFICACIN


Para representar el mensaje analgico x(t) en forma digital, primero se filtra paso bajo
adecuadamente antes de muestrear, con el objetivo de limitar la banda y as evitar el
185

solapamiento de las colas del espectro peridico producido por el muestreo. Vase diagrama
de bloques de un modulador de PCM.

x(t)

Filtro
paso bajo

Muestreador

xs(t)

xsq(t)

Cuantificador

Codificador

PCM(t)

Figura 5.3. Diagrama de bloques de un generador de PCM


Los valores muestra, se redondean o cuantifican al valor discreto predeterminado ms
prximo o nivel cuntico. La seal resultante es discreta en tiempo debido al muestreo y
discreta en amplitud debido a la cuantificacin. Por ltimo, un codificador convierte las
muestras cuantizadas en palabras de cdigo digital, con una palabra de cdigo por cada
muestra. Se ha efectuado una conversin analgico/digital. En la figura siguiente puede verse
el proceso grficamente.
5/8

xqs
xs

x(t)
-1/8
-3/8
-5/8

xsq
numero de
codigo
codigo
binario

-5/8

-1/8

5/8

-3/8

001

011

110

010

onda PCM

Figura 5.4. Proceso de codificacin


Es conocido que:
Q 2n con Q = nmero de niveles cunticos, n = nmero de bits
Como se requieren varios dgitos por cada muestra de mensaje, el ancho de banda de
modulacin por pulsos codificados (BTBB) ser mucho mayor que el ancho de banda del
mensaje (W). Una estimacin de este ancho de banda se obtiene como sigue. Las muestras
186

cuantificadas ocurren a una frecuencia fs2W muestras por segundo por lo que debe haber
r=nfs dgitos (bits) por segundo y si los pulsos son de anchura () la mitad del periodo y
tomando el ancho de banda del tren de pulsos 1

se verifica que el ancho de banda mnimo

es r Hz, esto es :
BTBB r nf s n 2W
Como paso final de la generacin de la modulacin de PCM, la seal de banda base puede
modular una portadora de radiofrecuencia para propsitos de transmisin (ASK, PSK, FSK..),
en este caso el ancho de banda resultara an mayor.

5.4.2 RUIDO DE CUANTIFICACIN


En el receptor, a partir de la forma de onda de la modulacin por pulsos codificados
(PCM(t)) contaminada con ruido aleatorio (n(t)) es decir PCM(t)+n(t), se regeneran las
palabras de cdigo digital mas errores producidos por el ruido. De estas palabras de cdigo el
decodificador determina los valores muestra cuantificados xsqr(t) (con errores de ruido) y se
filtra paso bajo para obtener la seal analgica de salida xsr(t). Si el ruido a la entrada del
receptor no es grande, la probabilidad de error es pequea y se puede despreciar el efecto del
ruido aleatorio en este caso llamaremos xsr(t)=xs(t).
En la figura siguiente se muestra un esquema de bloques de un receptor de PCM, siendo la
xs(t), el mensaje estimado (sin ruido aleatorio, es decir con error igual a cero).
PCM(t)+n(t)
regenerador

decodificador

xsq(t)+error

filtro

xsr(t)

paso bajo

Receptor

Figura 5.5. Diagrama de bloques de un receptor de PCM


A pesar de la condicin anterior, la seal considerada libre de ruido (xs(t)) que sale del
filtro, no es idntica al mensaje x(t) puesto que toda la reconstruccin se ha basado en las
muestras de x(t) cuantificadas, por tanto es imposible la reconstruccin perfecta del mensaje
en los sistemas de codificacin de pulsos, an cuando el ruido aleatorio sea despreciable.
El efecto de cuantificacin es una limitacin bsica de los sistemas codificados, as como
el ruido aleatorio es una limitacin de los sistemas analgicos.
187

La seal cuantificada puede ponerse como:

x sq ( t ) x ( t ) q ( t )
Dado que es intercambiable el orden de muestreo y cuantificacin, los valores muestra
cuantificados conforme se decodifican en el receptor se pueden escribir de forma
x sq (kTs ) x( KTs ) q ( KTs ) y al filtrar paso bajo queda:
xs (t ) x(t ) q (kTs )sinc( f s t k )
k

En consecuencia, el error de cuantificacin tiene una forma idntica a los errores de ruido
aleatorio en la modulacin analgica de pulsos.
Se designa a 2q como el ruido de cuantificacin. Se evala con facilidad cuando los niveles
cunticos tienen espaciado uniforme en este caso q (t )

1
, siendo Q el nmero de niveles
Q

cunticos y la relacin seal a ruido (S/N) a la salida del decodificador es:


S
2 2
3Q x
N
D

siendo x 2 el valor cuadrtico medio del mensaje o dicho de otro modo, la potencia media de
la seal a transmitir.

5.4.3 TRANSMISIN Y RECEPCIN DE LA SEAL PCM


La seal PCM salida del codificador es una secuencia de pulsos mas o menos estrechos, de
la misma amplitud, que segn el tipo de cdigo que se use, pueden ser positivos y negativos
(si se utiliza cdigo bifase) o con la misma polaridad, pero en cualquier caso estn
perfectamente delimitados. Del mismo modo que en PAM, si se pretende conservar la forma
del pulso, el ancho de banda de la secuencia resultante es infinito o en cualquier caso muy
grande, y esto resulta muy poco operativo, o dicho de otra manera, demasiado costoso para
transportar informacin. As, parece adecuado filtrar paso bajo esta seal formada por pulsos
de tal modo que reduciendo su ancho de banda en el emisor, permita recuperar completamente
la informacin en el receptor. Ante todo es preciso destacar, una vez ms, la importancia que
tiene la sincronizacin de los osciladores de muestreo en el transmisor y en el receptor.
La figura siguiente muestra una secuencia en PCM de una cierta informacin, cuyo
formato de salida se ha elegido bifase y con retorno a cero.

188

PCM(t)
xPCM(t)

T0/8

T0

Figura 5.18. Secuencia en PCM en formato bifase y con retorno a cero.


Esta grfica indica tambin que la codificacin se ha realizado a 256 niveles, es decir 8 bits
por muestra y que el perodo de muestreo de la seal codificada ha sido T0 segundos.
Una de las caractersticas de la modulacin por pulsos codificados es que la seal de
sincronismo o de reloj en el receptor puede extraerse, con un poco de habilidad, de la propia
seal PCM. La pregunta ha plantear sera Cul debe ser el mnimo ancho de banda del filtro
paso bajo a insertar en la ultima etapa del emisor para que pudiendo detectar la presencia o la
ausencia de los pulsos en el receptor, permita adems obtener la seal de sincronismo?.
En la grfica anterior se puede apreciar una seal suavizada xPCM(t), muy semejante a una
funcin sinusoidal que, al ser rectificada en el receptor, siempre presentara un ciclo por bit.
Es decir 8 veces la frecuencia de muestreo( 8f0 con f0 =2B) de la seal moduladora o de
informacin.
Por tanto, con un filtro paso bajo en el emisor de frecuencia 16 veces el ancho de banda de
la seal moduladora (16B Hz) se estara en condiciones de muestrear en las posiciones
correctas la seal recibida. La figura siguiente muestra la misma seal PCM dada en la figura
anterior, pero con formato monopolar y sin retorno a cero, que se puede considerar menos
elaborado en cuanto a facilidades para recuperar el sincronismo.
PCM(t)
xPCM(t)

T0

t
T0/8

Figura 5.19. Secuencia en PCM en formato monopolar y sin retorno a cero.


189

La frecuencia ms elevada de una onda senoidal se producira cuando la secuencia fuera de


unos y ceros alternados. En este caso presentara un ciclo cada dos bits y la frecuencia
mxima seria 4f0, representando 8 veces el ancho de banda de la seal moduladora (8B Hz).
Por otra parte, la circuitera para recuperar la seal de sincronismo en el receptor debera ser
ms elaborada.
Conocer los instantes en que se debe muestrear la seal recibida no es suficiente. Para
decodificar la seal PCM tambin se necesita saber donde empieza cada muestra, por tanto
esta informacin debe ser enviada de alguna forma desde el emisor. Podra usarse el mismo
procedimiento, en emisin y en recepcin, que el empleado en PAM, multiplexando por
divisin en frecuencia una portadora fuera de la banda de la seal PCM con una secuencia de
pulsos a la frecuencia de muestreo de la seal de informacin (2B Hz), sin embargo ahora es
el momento de introducir otro procedimiento, pero es preferible, primero, hacer algunos
comentarios sobre MDT de seales PCM.

5.4.4 MULTIPLEXADO POR DIVISIN EN EL TIEMPO DE SEALES PCM


Supngase, que con tcnica PCM, se desea multiplexar en el tiempo 30 canales telefnicos
de 4 kHz de ancho de banda. Adanse dos canales ms para disponer, en uno de ellos por
ejemplo, diversa informacin sobre sealizacin y gestin de los canales, adems de permitir
su supervisin a largo de su transmisin, y en el otro informacin de sincronismo.
La frecuencia de muestreo de cada canal deber ser 8 kHz y cada muestra se codifica con 8
bits. El multiplexado se organiza tal como se refleja en diagrama siguiente.
canal de sincronismo

Todos los canales menos c0 y c15 son de voz o datos

canal de supervisin

C0 C1 C2

.....

C15

............

C31 C0 C1

C2

.....

C15

Trama 0
......
muestra de 8 bits por cada canal

............
Trama 1

C31

...
Trama 0

8 x 488 ns
125 s

Figura 5.20. Estructura del multiplexado en el tiempo de seales PCM


Calcular el numero de bits por segundo que se transmite es sencillo. Cada muestra se
codifica en 8 bits, de modo que 8000 muestras cada segundo representan 64000 bits por
segundo cada canal. Para transmitir 32 canales se precisa una velocidad de transmisin de
190

2048000 bits por segundo. Es decir transcurrido un segundo se deben haber transmitido 8000
muestras de cada canal.
Ahora hagamos algunos comentarios sobre el concepto de filtro adaptado

Filtro adaptado

Cuando se desea detectar una forma de onda particular que est inmersa en una seal,
resulta intuitivo ir comparando esta forma de onda con la seal que se est recibiendo. La idea
de comparacin de seales est inmersa en el concepto de convolucin y tambin en el de
correlacin. La figura siguiente es una representacin grfica del concepto de filtro adaptado.
En esta figura m(t) es la forma de onda a detectar, h(t ) m(tr t ) es la respuesta impulsional
del filtro. Ntese que ha sido creada girando especularmente m(t), tomando su eje vertical
como eje de giro, y despus retardando la funcin girada un tiempo igual a su duracin, para
que su comienzo lo haga en t=0 segundos.
m(t)

h(t)

tr

y(t)

tr

h(-t)

ta
ys()

ta+tr
tr

Figura 5.21. Representacin grfica del concepto de filtro adaptado

La expresin de la seal filtrada es ys ( )

y(t )h( t )dt

que lgicamente es la

convolucin de la seal de entrada con la respuesta impulsional del filtro. Como es sabido, la
convolucin consiste en multiplicar ambas funciones en cada instante de tiempo y despus
integrar el resultado de este producto. Ntese que el parmetro da movimiento a la funcin
h(t) haciendo que esta recorra toda la funcin y(t). Ntese tambin que la expresin de la
191

convolucin obliga a girar especularmente h(t) (puede girarse cualquiera de las dos funciones,
el resultado es el mismo) antes de recorrer la funcin y(t). Por esta razn la funcin h(t) se ha
creado girando antes la funcin de inters m(t). De este modo cuando la operacin de
convolucin la gire otra vez, quedar tal como se encontraba originalmente. Por esta razn es
obvio el hecho de que se use, para crear la respuesta impulsional del filtro, la misma forma de
onda que la que se quiere detectar. Cuando ambas coincidan en el tiempo, al multiplicarse no
habr valores negativos y la integral, o lo que es lo mismo la suma de todas las muestras de
este producto, siempre ser mayor en este instante que en otro en que no haya coincidencia.
No se debe olvidar la operacin de correlacin que, conceptualmente, representa la
comparacin

de

ys ( ) Rmy ( ) m( ) y ( )

dos

seales.

Su

expresin

es

m(t ) y (t )dt , en donde y (t) es la seal conjugada de


*

y(t).
Las operaciones de convolucin y de correlacin son muy similares, salvo que la primera
se realiza en tiempo real; de hecho es la forma de operar de un sistema lineal, as se podra
construir un filtro con bobinas y condensadores que tuviera la respuesta impulsional
requerida. En cambio la correlacin no es una operacin en tiempo real; quiere decir que las
seales que se operan deben estar almacenadas previamente en algn lugar para poder
multiplicarse punto a punto. Pero esto no impide que pueda construirse un correlador
simulando su expresin. El diagrama de bloques siguiente representa un correlador de las
seales m(t) e y(t).
ys()
y(t)

Integrador

m(t+)

Figura 5.22. Diagrama de bloques de un correlador


Puede interpretarse como sigue: Colocando la seal patrn m(t) sobre la seal y(t) en la
posicin segundos, multiplicando entonces las dos seales y despus integrando todo este
producto, el valor resultante ys() ser mas o menos grande segn el parecido de ambas
seales en el instante . Por tanto teniendo un punto de partida en donde alinear las dos

192

seales a multiplicar, la correlacin tambin sera til para detectar formas de ondas
determinadas dentro de una seal.

5.4.5 DETECCIN DE PALABRAS DE CDIGO CON FILTRO ADAPTADO


La aplicacin de un filtro adaptado a una secuencia para detectar un cdigo dentro de esta
secuencia parece un procedimiento prometedor.
Supngase que en el canal cero de cada trama de una seal PCM se ha colocado una
palabra, por ejemplo de 8 bits, que no puede darse en ningn otro canal, entonces si en la
secuencia de bits de una seal PCM se encontrase esta palabra, se sabra que la palabra de 8
bits siguiente corresponde al canal uno y los 8 siguientes al canal dos, etc.

Ejemplo

Una seal PCM transmite un codigo de alineamiento de trama en el canal cero. El


sincronismo de bit se ha extrado de la seal PCM recibida, por tanto se conoce la posicin de
cada bit pero no donde comienza cada trama y por tanto donde comienza cada canal.
PCM(t)

10010001
. . . . .

t
C31

C0

C1

C2

C3

Conociendo el formato medio de la seal recibida y el sincronismo de bit, que tiene una
frecuencia de 2048000 Hz, dibujar un diagrama de bloques que muestre como decodificar la
seal PCM correctamente.

Solucin

La seal patrn m(t) tendra la forma que se ha estimado recibir el receptor, bien mediante
calculo o bien mediante mediciones.

193

m(t)

10010001

h(t)

10001001

As la respuesta impulsional del filtro adaptado ser h(t ) m(tm t ) con tm= 3.906 s la
duracin de 8 bits de la seal PCM
El diagrama de bloques siguiente representa un receptor de PCM

PCM(t)

Filtro paso bajo


de 2048000 Hz

Filtros paso bajo


de 4 kHz

Filtro adaptado
h(t)

hb(t)

ha(t)
Conformador de
pulsos
Generador de
impulsos de trama
Multiplicar la
frec. x 32

D/A

Reg. Desplazamiento

Generador de
impulsos de bit

x0(t)

Retardos

Lach

Conformador de
pulsos

Conformador de pulsos

Rectificador

hr(t)=(t-)

ha(t)

hr(t)=(t-2)

ha(t)

x1(t)

.
.
.
.
hr(t)=(t-31)

x2(t)

x31(t)
ha(t)

= 3.906 s equivalente a 1/256000 Hz que


es el intervalo entre muestras

La salida del filtro adaptado ser un valor elevado cuando a la entrada aparezca la seal de
cdigo. Esto suceder cada 125 s o bien 8000 veces por segundo, que es la frecuencia de
trama. Esta frecuencia multiplicada por 32 representa los 256 kHz de frecuencia de canal.
194

El desplazamiento en el registro se produce cada 1/2048000 segundos y cada 8 veces este


tiempo (que est sincronizado con el comienzo de trama) se activa el lach que da paso a la
conversin D/A del byte. A partir de aqu representa una seal PAM.
La figura siguiente muestra este procedimiento de convolucin implementado digitalmente
con un registro de desplazamiento. Obsrvese que por el hecho de usar un registro de
desplazamiento, la palabra de cdigo a detectar se coloca invertida en los multiplicadores.

PCM(t)

Filtro paso bajo


de 2048000 Hz
hb(t)

Conformador
de pulsos

Rectificador

Reg. de desplazamiento de 8 bits

-1

-1

-1
-1

-1

Conformador de
pulsos
Generador de
impulsos de bit

Deteccin de la palabra de
cdigo 1 0 0 1 0 0 0 1 cuando
la suma tome el valor 8

Un detalle a tener en consideracin. Podra suceder que durante la deteccin, el final de un


byte y el principio de otro presentasen el mismo cdigo que se desea detectar, por tanto se
cometera un error. En realidad el emisor, mediante un procedimiento de aleatorizacin de la
secuencia de bits sin contar el canal cero, correspondiente a cada trama, cuida que nunca se
presente esta secuencia de cdigo. Por tanto en el emisor existe una etapa de codificacin de
la trama que debe ser restaurada en el receptor.

5.4.6 UMBRALES DE ERROR


Si se considera que el ruido aleatorio no puede despreciarse, entonces tenemos ruido de
decodificacin y ruido de cuantificacin.
Dependiendo de la relacin potencia de la seal a potencia de ruido a la entrada del
receptor (S/N)R ,obtendremos mas o menos errores al decodificar ,es decir la probabilidad de
195

error Pe depende de la relacin seal a ruido mencionada. Esta probabilidad de error cuando
es 10-4 puede ponerse como:
1
3 S
Pe 21 Q

2
1 N R
siendo =2 si se escoge el sistema binario (n=Q) y la relacin seal a ruido en el
decodificador como:
3Q 2 x 2
S


2
N D 1 4(Q 1) Pe

5.4.7 COMPORTAMIENTO CUALITATIVO DE LA SEAL PCM


Una forma de obtener una indicacin cualitativa bastante aceptable del comportamiento de
un sistema PCM consistira en visualizar la secuencia de bits de la seal en un osciloscopio.
La escala de tiempo se ajusta para que se dispare a la velocidad de los bits con una duracin
de barrido de algunos bits.

a)

b)

c)

Figura 5.23. Aspecto de la seal PCM en un osciloscopio sincronizado a la frecuencia de bit.


Caso ideal a). Con limitaciones de ancho de banda e interferencia entre smbolos b).
Diagrama del ojo c).
Idealmente, una seal PCM con codificacin bipolar y retorno a cero se visualizara como
en la figura a). Estara formada por una superposicin de smbolos, perfectamente
delimitados, positivos y negativos. La figura b) muestra la visualizacin de una seal PCM
con ruido, limitaciones de ancho de banda e interferencia entre smbolos. Se ha comentado en
anteriores apartados como afecta a la forma del smbolo el filtrado no ideal y la interferencia
entre smbolos.
196

Como esta superposicin de smbolos proporciona esta forma tan peculiar, parecida a un
ojo, se ha dado en llamarla, diagrama del ojo. Proporciona informacin subjetiva acerca de la
degradacin de la seal PCM cuanto ms cerrado est el ojo. El ruido contribuye a cerrarlo.
Tambin da idea del mejor instante de muestreo de los smbolos que es aquel en el que el ojo
esta mas abierto.

5.5 CDIGOS DE DETECCIN Y CORRECCIN DE ERROR

Como la lnea a seguir en este documento es hacer hincapi en las modulaciones digitales
de la seal, no parece desacertado hacer algunas indicaciones de cmo detectar y corregir los
bits errneos que puedan haberse producido, en la seal codificada, durante una transmisin.
Para aumentar la fiabilidad de la comunicacin en presencia de ruido, una posibilidad (que
se comentar ms extensamente en el captulo de modulaciones digitales) es aumentar la
relacin seal/ruido, es decir aumentar la potencia del transmisor. Otra posibilidad, a la que
nos ceiremos en este estudio, es agregar bits a la transmisin, aumentando el ancho de
banda, para detectar y corregir los errores en la transmisin.
Dentro de la posibilidad de agregar bits, deben contemplarse dos modalidades: Cdigos
algebraicos o de bloque y cdigos convolucionales.

Cdigos algebraicos

Se pueden formar agregando dgitos binarios extra al final de cada palabra de cdigo. Estos
bits extra se llaman bits de paridad o de verificacin.
En general una palabra de codigo de k bits (M= b1b2b3....bk) ms r bits de paridad (p1p2...pr)
estara formada por n= k+r bits (b1b2b3....bkp1p2...pr ) . Los bits de paridad deben permitir
cumplir el sistema de ecuaciones lineales.
0 c11b1 c12b2 ....... c1k bk p1
0 c21b1 c22b2 ....... c2 k bk p2
.
.
0 cr1b1 cr 2b2 ....... crk bk pr
Ms manejable sera escribir 0 H T con:
197

c11 c12 .......c1k 1 0 ..... 0


c c .......c 0 1 ..... 0
2k
21 22

H=
.

cr1 cr 2 .......crk 0 0 ..... 1

b1
b
2
.

bk
T=
p1

p2
.

pr

T representa la palabra transmitida y H representa la clave para generar T con ayuda de la


palabra original M. La idea consiste en determinar si en recepcin ha habido o no error.
Naturalmente la clave debe ser conocida en recepcin.
En recepcin se recibir una palabra, llammosla R. El producto S H R recibe el
nombre de sndrome. Si S=0 entonces R=T y no ha habido error. Los k primeros dgitos es la
palabra M originalmente transmitida. Si S H R 0 se ha cometido al menos un error. En
este caso veamos que se puede hacer manipulando las matrices.
La palabra recibida R puede descomponerse en la palabra T correcta ms otra de error E,

R T E . Ambas son de la misma dimensin, es decir n filas por 1 columna. Si no hubiera


habido error E sera cero y si hubiera habido error, determinando E podra conocerse T.
La matriz S tambin se puede escribir,
S H R H T H E H E

Finalmente,

E H 1 S y T R E
S tiene dimensin r filas por 1 columna, por tanto no puede determinar la secuencia de
error sin ambigedad.
Los cdigos de Hamming son cdigos de verificacin de paridad, capaces de detectar y
corregir un error, en los que las columnas de la matriz H estn formadas por las diferentes
secuencias de longitud r distintas de cero. Por tanto habr 2r-1 columna. As es necesario
elegir r para que r+k = n = 2r-1 y el cdigo sea eficaz.
Si hay un solo error, la matriz E tendr un solo elemento distinto de cero y aparecer una
fila no nula de S. El nmero de esta fila indica el bit errneo de la matriz R.
Veamos un ejemplo.
198

Ejemplo

Un codigo Hamming (7,4) ( 4 bits de mensaje y 3 de paridad) tiene la matriz de


1 1 1 0 1 0 0
verificacin de paridad H 1 1 0 1 0 1 0
1 0 1 1 0 0 1

1) La palabra de cdigo T a transmitir si el mensaje es 0 0 1 1


2) Si la palabra recibida R es 1 0 1 1 1 1 0 comprobar que se detecta un error y corregirlo.

Solucin
0
0

1 1 1 0 1 0 0 1 0 0 1+0 +p1 0

1) H T 1 1 0 1 0 1 0 1 0 0 0+1 +p2 0 que implica p1=1, p2=1, p3=0
1 0 1 1 0 0 1 p1 0 0 1+1 +p3 0

p2
p
3

As el mensaje codificado ser 0 0 1 1 1 1 0.


e1
1
e
0
2

1 1 1 0 1 0 0 1 1
1 1 1 0 1 0 0 e3 1

2) S H R 1 1 0 1 0 1 0 1 1 y S H E 1 1 0 1 0 1 0 e4 1
1 0 1 1 0 0 1 1 1
1 0 1 1 0 0 1 e5 1


e6
1
e
0

7

Tal como se ha comentado anteriormente existe ambigedad en la solucin del sistema. La


solucin pasa por determinar los valores de E que cumplan la ecuacin con el cambio de bit
ms proximo al de la seal recibida. Para hacerlo se define el peso de Hammning W(ci) de
una palabra de cdigo c como el nmero de unos que tiene. La distancia de Hamming D(ci, cj)
entre dos palabras de cdigo ci y cj se define como el nmero de posiciones en las que difieren
las dos palabras. Un ecuacin para determinar la distancia D=W(ci cj ), donde denota
suma mdulo dos (suma binaria sin acarreo). En cualquier caso el sndrome de la seal
detectada, en este caso 1 1 1 seala la primera posicin en la matriz de verificacin y por tanto
el primer bit recibido como erroneo. As con la matriz de error E= 1 0 0 0 0 0 0 se obtiene la
correccin.
199

1 1 0
0 0 0

1 0 1

T R E 1 0 1 y el mensaje son los cuatro primeros bits, 0 0 1 1
1 0 1

1 0 1
0 0 0

No se puede corregir los errores si stos se producen en ms de un bit

Ejercicio propuesto

Todas las palabras posibles de un cdigo son 0011110, 0101101, 0111000, 1011001,
1101010. Se recibe la palabra 1011011. Cul puede ser la palabra correcta transmitida si se
considera la distancia mnima de Hamming?

Cdigos convolucionales

Estos cdigos, conocidos tambin como secuenciales, difieren de los cdigos bloque en
que los dgitos de comprobacin se intercalan de forma continua en la corriente codificada de
bits en vez de agruparse en palabras. Para mostrar el principio de funcionamiento parece
adecuado utilizar un ejemplo.
bn
bn

bn cn
bn-1 bn-2 bn-3 bn-4
cn=bn bn-2 bn-4

Figura 5.24. Codificador convolucional de cuatro bits


La figura anterior representa un codificador convolucional formado por un registro de
desplazamiento de cuatro bits. Se suman en binario sin acarreo el bit actual ms el segundo y
el cuarto bit anterior.
La salida esta formada por dos bits, el actual y el bit suma que se han tomado alternados
del codificador. Esto implica que la velocidad de transmisin debe ser, al menos, el doble de
la generacin de bits de mensaje.

200

La informacin que se transmite est codificada con una combinacin de la propia


informacin.
En recepcin el decodificador deber extraer el bit de informacin brn y el bit de
codificacin crn recibidos.
crn

sn=en en-2 en-4 ecn

brn crn
brn

bn-1 bn-2 bn-3 bn-4

Figura 5.25. Decodificador convolucional de cuatro bits


El sndrome sn= brn crn = bn en bn-2 en-2 bn-4 en-4 [(bn bn-2 bn-4) ecn]
ser igual sn= en en-2 en-4 ecn ya que al sumar en binario sin acarreo los bits de brn y crn
los unos se cancelan y solo quedaran bits uno o bien cero debidos al error. El sndrome ser
uno o cero dependiendo si hay error o no y puede ser que se encuentre en el bit de
informacin recibido brn, en bit de codificacin recibido crn o bien en ambos. En este ltimo
caso no se detectara error.
Sin intencin de ir mas lejos, ntese que si en un instante dado bn es errneo, el sndrome
sn, sn+2 y sn+4 sern uno. Si no hubiera mas errores estos bits estaran alternados con ceros
hasta el instante seis. Si se produjeran dos errores seguidos todos estos bits del sndrome
serian uno.
Se puede verificar que puede corregir cuatro bits sucesivos siempre que los ocho bits
siguientes estn libres de error.
Para establecer reglas de decisin orientadas a la correccin de bits es preciso considerar
todas las posibilidades un en un anlisis estadstico.

201

PROBLEMAS PROPUESTOS

5.1.-Se modulan mediante PCM (modulacin por pulsos codificados) 32 canales (de voz o
datos) con un ancho de banda de 4 kHz. y se multiplexan en el tiempo ,para ser transmitidos
por un nico medio de transmisin. El cuantificador del modulador utiliza 256 niveles.para
codificar cada canal. Cual es en su opinin el ancho de banda mnimo que requerira el medio
de transmisin. Razone y apoye su respuesta numricamente.

5.2.- Se desea disear un sistema de comunicaciones PAM para transmitir un canal de voz
con ancho de banda 3.3 kHz, dos canales de datos cada uno con ancho de banda 800 Hz y
cuatro canales de datos de 400 Hz de ancho de banda cada canal. Con objeto de permitir un
filtrado paso bajo adecuado en el receptor, todos los canales se sobre muestrean un 20%. Se
utilizar un filtro del tipo coseno elevado con =0.5 para el filtro colocado despus del
multiplexor. Se pide:
a) Organizar un sistema de transmisin mltiplex por divisin en el tiempo, especificando un
diagrama de tiempos para el multiplexor.
b) Dibujar un diagrama de bloques que del sistema, tanto en transmisin como en
recepcin.

5.3.- Determinar el ancho de banda mnimo necesario para transmitir una secuencia binaria
a una tasa de reloj de 64 kbps si se usa a) seal RZ, b) seal RZ bifase, c) NRZ.
Lo mismo si en lugar de enviar bit a bit se genera un pulso de cuatro niveles posibles a partir
de cada dos bits de la secuencia de entrada.
Y si se combinan tres dgitos de la secuencia de entrada en lugar de 2?

5.4.- Veinticuatro seales de entrada de 4 kHz de ancho de banda se desean multiplexar en el


tiempo. Calcular el ancho de banda mnimo necesario para la transmisin si se utiliza PCM de
8 bits y seal bifase con retorno a cero y tambin si la seal utilizada es NRZ.

202

5.5.- El diagrama de bloques de la figura es un demodulador de pulsos en amplitud (PAM)

para recuperar la seal original x(t). La seal xs (t ) x (t ) (t nT ) x(nT ) (t nT ) son


las muestras de x(t).

xPAM(t)

Recuperador de
sincronismo +
muestreador

xs(t)

Integrador

xd(t)

x(t)

Retardo T

Encontrar la expresin de la respuesta al impulso h(t) y la funcin de transferencia H(f) del


subsistema enmarcado en trazo discontinuo. Que debera ser el ltimo bloque del sistema para
recuperar x(t) original
Lo mismo pero sustituyendo diagrama enmarcada a trazos por el de la figura siguiente
xs(t)

Retardo T

Integrador

Integrador

xd(t)

Retardo 2T

Dibuje aproximadamente como podra ser la seal xd(t). Igualmente explique que debera ser
el ltimo bloque del sistema.

5.6.- Dibujar un diagrama de bloques de un sistema que permita extraer la seal de


sincronismo de una modulacin de pulsos en duracin, bien conformados, tal como se
muestra en la figura.
xPMD(t)
.......
Ts

Despus proponga un diagrama de bloques del demodulador completo.

203

Puede ser de ayuda, a efectos de expresar alguna funcin de seal y/o sincronismo en el
diagrama

completo

apoyar

mejor

su

propuesta,

conocer

la

expresin

t kTs
xPMD (t )
con (k ) 0 1 mx(kTs )
k
(k )

204

CAPITULO 6
6 RUIDO ALEATORIO
6.1 PROCESOS ALEATORIOS

Consideraremos una seal aleatoria como la manifestacin de un proceso (elctrico en


nuestro caso) que se comporta de manera no predecible y que ocurre en el tiempo. Tambin se
llama proceso estocstico, que consiste en una sucesin de variables aleatorias que
evolucionan en funcin de otra variable, generalmente el tiempo. Cada una de las variables
aleatorias del proceso tiene su propia funcin de densidad de probabilidad y, entre ellas,
puede existir correlacin o no. Si esta seal es discreta en el tiempo, constituye lo que se
llama una serie temporal.
Supondremos que el valor observado de la serie en el instante t es una extraccin al azar de
una variable aleatoria definida en dicho instante. En consecuencia una serie de n datos ser
una muestra de un vector de n variables aleatorias ordenadas en el tiempo
(xi(1),xi(2)......,xi(n)). Se denomina proceso estocstico al conjunto de estas variables {xi(t)},
con t=1,2,....,n y la serie observada se considera una realizacin i del proceso. Si la seal es
continua en el tiempo, puede denotarse por xi(t) y tendremos entonces infinitas variables
aleatorias, una para cada valor de t.
Podemos considerar las realizaciones del proceso, si imaginamos disponer de un gran
nmero de generadores iguales de una seal aleatoria, de tal modo, que al ponerlos en
funcionamiento genere el primero, una seal x1(t), el segundo una seal x2(t), etc.., se habrn
obtenido entonces varias realizaciones de un mismo proceso aleatorio. La figura 6.1
representa un proceso aleatorio continuo en el tiempo y discreto en realizaciones.
PROCESO ALEATORIO

x 1(t)

t
x 2 (t)
x (t)
3
Re
al
iz
ac
io
ne
s

t
..

t
.

x m (t)

tiempo

Figura. 6.1. Proceso aleatorio


205

Diremos que conocemos la estructura probabilstica de un proceso aleatorio o estocstico


cuando se conozca la distribucin conjunta de las n variables aleatorias {x(n)}, si el proceso es
discreto en el tiempo, o si es continuo la distribucin conjunta de las variables aleatorias x(t)
(para cada valor de t, por ejemplo para t=ti, existe una variable aleatoria x(ti)). Esta
distribucin determina la de cualquier subconjunto de variables y en particular, las
distribuciones marginales para cada una de ellas.
La determinacin prctica de la distribucin conjunta del proceso requiere observar un
gran nmero de realizaciones. Esta estimacin se simplifica mucho cuando podemos suponer
que la distribucin multivariable conjunta es normal (gaussiana), ya que entonces, quedar
determinada por el vector de medias y la matriz de varianzas y covarianzas entre las variables.
Llamaremos funcin de medias del proceso, llamado tambin funcin de medias en las
realizaciones, a una funcin del tiempo que proporciona las medias de las distribuciones
marginales (las distribuciones para cada variable x(ti) ) x(t) para cada instante:

(t)=E[x(t)]
Un caso particular importante se produce cuando todas las variables tengan la misma
media, es decir que la funcin de medias sea constante. Entonces las realizaciones del proceso
no mostrarn tendencia creciente o decreciente y diremos que el proceso es estable en la
media. Si por el contrario las medias varan con el tiempo, las observaciones en distintos
momentos mostrarn esta tendencia.

dos realizaciones cualquiera

dos realizaciones cualquiera

x(t)

x(t)

Figura. 6.2. Proceso aleatorio a) Estable en la media. b) No estable en la media


La figura 6.2a presenta una realizacin de un proceso aleatorio, que suponiendo un
comportamiento similar en todas las realizaciones, simula un proceso estable en la media,
mientras que la figura 6.2b simula otro con media no estable.
La funcin de varianzas del proceso proporciona las varianzas en cada instante temporal:
206

(t)2 =Var[x(t)]
Diremos que el proceso es estable en la varianza si esta es constante en el tiempo. El
proceso puede ser estable en la media y no en la varianza. La serie de la figura 6.2a no es
estable en la varianza mientras que la serie de la figura 6.2b parece serlo.
La estructura de dependencia lineal entre las variables aleatorias del proceso se representa
por las funciones de covarianza y correlacin. Llamaremos funcin de autocovarianzas del
proceso a la funcin que describe las covarianzas en dos instantes cualesquiera:
Cov(t,t+j)=Cov(x(t),x(t+j))=E[(x(t)-(t))(x(t+j)-(t+j))]
Llamaremos funcin de auto correlacin a la estandarizacin de la funcin de covarianzas:
Rxx(t,t+j)=Cov(t,t+j)/t t+j
En general estas dos funciones dependen de dos parmetros (t,j), siendo t el instante inicial
y j el intervalo entre observaciones.
Es preciso hacer notar que estas estadsticas se realizan sobre el proceso aleatorio en el
sentido de las realizaciones. Dicho en otras palabras, este proceso podra haberse expresado

como x ( , t ) (siendo la variable realizaciones) y efectuando las operaciones de media,


varianzas, etc.. sobre esta variable manteniendo t constante. Es por ello que las estadsticas
dependen de t.
producto p de
x

p1(tj)
p2(tj)

p1(ti)
p2(ti)

x1(t)

Rxx(tj,tj+n)

p3(tj)
p3(ti)

x2(t)
x3(t)
realizaciones
autocorrelacin en el
desplazamineto n

pm(tj)
pm(ti)

xm(t)

n
Rxx(t,t+n)

ti

Rxx(ti,ti+n)

tj

t
t

Figura. 6.2.1. Ejemplo de un proceso aleatorio no estacionario y la autocorrelacin en el


retardo tn.
Se pueden efectuar medidas de la estadstica del proceso para cada realizacin a lo largo
del tiempo, obteniendo por ejemplo el promedio en el tiempo para una realizacin
207

determinada x(t) o escrito de otra manera x( i,t). En general la estadstica efectuada en


las realizaciones es diferente de la estadstica efectuada en el tiempo y ambas no son
relacionables, sin embargo existen procesos particulares muy interesantes en los que si existe
esta relacin.
Rxx(t,t+)

Rxx(t,t+1)
Rxx(t,t+2)
Rxx(t,t+3)
..
..
Rxx(t,t+n-1)
Rxx(t,t+n)

tj

ti

tk

Figura. 6.2.2. Ejemplo de la autocorrelacin de un proceso aleatorio no estacionario

6.2 PROCESOS ALEATORIOS ESTACIONARIOS

Diremos que un proceso estocstico es estacionario en sentido amplio si existen y son


estables la media, la varianza y las covarianzas, es decir ,si para todo t:

t ==Cte.
t2 =2=Cte.
Cov(t,t+k)=Cov(t,t-k)=k

k=0,1,2.....

Para un proceso estacionario la funcin de auto correlacin se calcula mediante:


R(k)=k/0 teniendo en cuenta que 0=2
En pocas palabras, la funcin de auto correlacin depende de la diferencia de tiempos y no
del origen tomado.
Rxx()

estacionario

ti

tj

Figura. 6.2.3. Ejemplo de un proceso aleatorio estacionario


208

Se denomina funcin de auto correlacin o correlograma a la representacin de los


coeficientes de auto correlacin en funcin del retardo.

6.2.1 ERGODICIDAD
Cuando la dependencia entre observaciones tiende a cero al aumentar el retardo diremos
que el proceso es ergdico. La ergodicidad es necesaria para poder estimar las caractersticas
del proceso a partir de una nica realizacin ya que si el proceso no es ergdico todas las
observaciones son muy dependientes entre s, y aunque se aumente el tamao muestral no se
adquiere ms informacin; entonces no es posible obtener estimadores consistentes de los
parmetros. En la prctica, suponer que los procesos estacionarios son ergdicos equivale a
admitir que han sido eliminados, si existen, los trminos sinusoidales deterministas del tipo

xt=Acos(wt+), siendo A,w y constantes fijas.


La estacionariedad amplia o dbil, no garantiza la estabilidad completa del proceso. Por
ejemplo, la distribucin de las variables xt puede estar cambiando en el tiempo. Sin embargo,
si suponemos que estas variables tienen conjuntamente una distribucin normal, como sta
queda determinada por las medias, varianzas y covarianzas, todas las distribuciones
marginales sern idnticas. Diremos entonces que el proceso es estacionario en sentido
estricto.
Concluyendo, si un proceso aleatorio es estacionario y ergdico, teniendo adems las
variables aleatorias una distribucin gaussiana, el proceso es estable y adems nos bastar una
nica realizacin para caracterizar toda la estadstica del proceso. En este caso:
E[x(t)]=x(t)==Cte
Var[x(t)]=Varx(t)=2 =Cte.

Rxx ( ) E x(t ) x (t ) x(t ) x (t )


Ntese

que

estrictamente

esta

x(t ) x (t )dt '

autocorrelacin

debera

escribirse

T /2

1
x(t ) x (t )dt puesto que se trata de seales de potencia media finita.
T T
T /2

xx ( ) lim

Conceptualmente el trato de estas seales es semejante al que se aplica a las seales de

209

energa finita, no obstante a modo de refresco de la simbologa que se usa en el captulo 3,


seguidamente se presentan algunas consideraciones que pueden ser de utilidad.

Considrese la seal aleatoria xT (t ) x(t ) t

de energa finita obtenida a partir del

truncado de la seal aleatoria x(t) de potencia media finita. Tal como se ha visto en el captulo
T

3, su transformada de Fourier es X T ( f )

x(t )e

j 2ft

dt y su funcin de auto correlacin

ser, RxT xT (t ) xT (t ) xT (t ) , por tanto su funcin de densidad espectral de energa es ,


2

S xT xT ( f ) TF xT (t ) xT (t ) X T ( f ) .

Haciendo uso de las expresiones 3.7, 3.10 y 3.11 se puede escribir,


T

1
1
TF xx ( ) TF lim x(t ) x (t )dt TF lim xT (t ) xT (t )dt Gxx ( f )
T

T T

T T 2

Se demostrar ahora que la densidad espectral de potencia de una seal aleatoria x(t) se
puede obtener a partir de esta seal aleatoria truncada haciendo crecer hasta el infinito el
1

intervalo de truncado. Tomando el trmino TF lim xT ( ) xT ( ) y operando se


T T

obtiene,
1
1

TF ( ) TF lim xT ( ) xT ( ) lim TF xT ( ) xT ( )
T T
T T

1
TF
T T
T

lim

x ( )

1 2

T sinc(Tf ) X ( f ) sinc(Tf ) X ( f )
x ( ) Tlim
T

2
2
2
lim T sinc(Tf ) 2 X ( f ) ( f ) X ( f ) X ( f ) Gxx ( f )

Ejercicio.
1 2
2
T sinc(Tf ) X ( f ) sinc(Tf ) X ( f ) lim T sinc(Tf ) 2 X ( f )

T T
T

Demostrar que lim


Solucin.

Si se expresa X ( f ) Real X ( f ) j Imag X ( f ) entonces,


sinc( f ) X ( f ) sinc( f ) X ( f )

sinc(Tf ) Real X ( f ) j Imag X ( f ) sinc(Tf ) Real X ( f ) j Imag X ( f )

210

sinc(Tf ) Real( X ( f ) sinc(Tf ) Imag( X ( f )


2

sinc(Tf ) 2 Real X ( f ) sinc(Tf ) 2 Imag X ( f )


2

sinc(Tf ) 2 Real X ( f ) Imag X ( f ) sinc(Tf ) 2 X ( f )


2

Lo expuesto anteriormente tambin implica que

S xT xT ( f )
T

XT ( f )
T

es la densidad

espectral de potencia de la seal aleatoria truncada. Lo cual quiere decir que calculando la
densidad espectral de potencia de mltiples trozos de seal aleatoria xT() de longitud T
segundos, y realizando el valor medio de todos ellos, se obtendra una estimacin de la
densidad espectral de potencia de la seal aleatoria no truncada.
En general se conocen las densidades espectrales de potencia de las seales ruidosas y a
partir de estas densidades espectrales es posible obtener las funciones de autocorrelacin
como la transformada inversa de Fourier. Esto es,

xx ( )

xx

( f )e j 2 f df Px

Obsrvese que la funcin de auto correlacin en el origen es igual a la potencia media de la


seal aleatoria.

xx (0) G xx ( f )df Px

Tambin puede escribirse en funcin de su estadstica como xx (0) x 2 x2 x 2 y su


densidad espectral de potencia es:
Gxx ( f ) TF xx ( )

El ruido trmico es un proceso aleatorio que constituye un ejemplo de lo expuesto.

6.3 RUIDO

En el proceso de generacin, manipulacin, transmisin etc. de seales en general y


particularmente elctricas, se produce una mezcla de seales ajenas con las de inters, que las
perturban. A estas seales indeseadas las llamaremos ruido, por lo tanto, el ruido es una seal
indeseable que no tiene ninguna relacin con la deseada, adems ser de naturaleza
impredecible, es decir ruido aleatorio.
211

Segn lo dicho, un zumbido a 50 Hz producido, por ejemplo, por la fuente de alimentacin


y que se aade a la seal de salida de un sistema, es ciertamente ruido pero no aleatorio,
puede predecirse y en consecuencia eliminarse, podra catalogarse, quizs, como
interferencia.
En general las fuentes de ruido pueden clasificarse como:
- Ruido producido por el hombre. Se debe a la recepcin de seales indeseables
provenientes de otras fuentes tales como contactos defectuosos, radiacin producida por la
puesta en marcha de mquinas elctricas o alumbrado, etc...
- Ruido creado por perturbaciones de la naturaleza. Procede de relmpagos, tormentas
elctricas, ruido intergalctico, etc..
- Ruido de fluctuacin. Aparece en los sistemas fsicos debido a fluctuaciones espontneas
como el movimiento trmico de los electrones libres en una resistencia, la generacin de
electrones de un ctodo caliente, recombinacin de huecos y electrones en un semiconductor,
etc..
Un sistema fsico, puede ser protegido en mayor o menor medida de ruido producido por el
hombre y del creado por la naturaleza e incluso del ruido de fluctuacin, excepto del ruido
trmico, puesto que es inherente al propio sistema.

6.3.1 RUIDO TRMICO


Este tipo de ruido se debe al movimiento aleatorio de los electrones libres en medios
conductores. Debido a su energa trmica, cada electrn libre dentro de un elemento pasivo
est en movimiento, la trayectoria del movimiento de un electrn es aleatoria debido a sus
colisiones con la estructura del material. El efecto neto de todos los electrones forma una
corriente elctrica que fluye a travs del elemento.
Johnson y Nyquist en el ao 1928, fueron los primeros en estudiar el ruido trmico en
resistencias metlicas, desde entonces y gracias a estudios experimentales y tericos existe un
amplio conocimiento sobre el tema. Cuando una resistencia R se encuentra a una temperatura
T en grados Kelvin, existe un voltaje aleatorio v(t) entre sus terminales en circuito abierto.
Esta seal v(t) constituye un proceso aleatorio y estacionario (consideraremos tambin el
proceso ergdico y por lo tanto bastar una nica realizacin para tener caracterizado el

212

proceso) de distribucin gaussiana y media nula v 0 , con una potencia media dada por su
varianza:

v 2 v2

2( KT ) 2
R voltios al cuadrado
3h

Siendo:
K 1.37 1023 J/grado

constante de Boltzmann

h= 6.62 10-34 J segundo

constante de Planck

La densidad espectral de potencia del ruido trmico viene dada por la expresin siguiente:
G vv ( f )

2 Rh f
h f

kT

V 2 Hz.

En la figura siguiente puede apreciarse que la densidad espectral de potencia de ruido


trmico es prcticamente constante en un margen de frecuencias muy elevado, por tanto
puede considerarse constante de valor 2KTR a todos los efectos electrnicos convencionales.
Gvv(f)
2KTR

f Hz

12

10

Figura 6.3. Densidad espectral de potencia de ruido trmico


La figura siguiente es la representacin muestreada (2300 muestras) de la realizacin de un

Amplitud

proceso de ruido trmico.

Tiempo

Figura 6.4. Realizacin de un proceso de ruido trmico

213

Las figuras a y b siguientes muestran la densidad espectral de potencia del proceso anterior

Amplitud

Amplitud acumulada

y la integral de dicha densidad espectral.

Frecuencia

Frecuencia

a)

b)

Figura 6.5. Densidad espectral de potencia a) y su integral b) del proceso de la figura 6.6.
Si la densidad espectral es plana, su integral debe ser una recta con pendiente, como as
ocurre.

6.3.2 RUIDO BLANCO


Si la densidad espectral de potencia de ruido es constante, significa que este espectro
contiene todas las componentes de frecuencia en la misma proporcin y se designa como
ruido blanco por analoga con la luz blanca de la que se dice contiene todos los colores del
espectro visible.
Adems de las resistencias, existen otras fuentes de ruido gaussiano con densidades
espectrales de potencia planas. Genricamente se expresa la densidad espectral de potencia de
ruido blanco como G xx ( f )

La grfica de la densidad espectral Gxx(f) de un proceso de ruido blanco x(t) es:


Gxx(f)

2
f Hz

Figura 6.6. Densidad espectral de potencia de ruido blanco

214

Naturalmente si la fuente de ruido es una resistencia, entonces =4KTR


Por otra parte, recordando que la transformada de Fourier de la funcin de auto correlacin
es igual a la densidad espectral de potencia, entonces se puede encontrar la funcin de auto
correlacin a partir de la densidad espectral de potencia, mediante la transformada inversa de
Fourier, as:

xx (t )

xx

( f )e

j 2 ft

df

2e

j 2 ft

df

(t )

Que mostrado grficamente es:

(t)
xx

(t)
2
t

(s)

Figura 6.7. Auto correlacin de ruido blanco


Significa que en la realizacin de un proceso de ruido blanco sus variables aleatorias, al no
estar correlacionadas entre si, para cualquier retardo, pueden considerarse son
estadsticamente independientes. Tambin es deducible que dos realizaciones cualesquiera de
un proceso de ruido blanco, adems de ser sus variables independientes, su producto escalar
es nulo y por tanto son ortogonales.
Puede parecer poco intuitivo que la potencia media del ruido blanco sea infinita, pero si
partimos de la idea de que esta seal est compuesta de infinitas frecuencias, cada una de ellas
con una potencia media en vatios de /2 y que la potencia media total es la suma de todas las
potencias medias de sus componentes, entonces es incluso comprensible el concepto de
varianza infinita que se desprende de este razonamiento.
No obstante es necesario recordar que el ruido blanco representa solo un concepto til.
Pinsese que una realidad fsica ms prxima a este concepto es el ruido trmico, en el que su
densidad espectral de potencia decrece a partir de una cierta frecuencia y por tanto su funcin
de autocorrelacin no ser una delta en el origen.

215

6.3.3 COMENTARIOS
Antes de seguir avanzando en este tema, es preciso detenerse y efectuar algunos
comentarios que puntualicen o clarifiquen ciertas posturas o acciones que se toman.
A primera vista parecera intuitivo obtener la densidad espectral de potencia de ruido

Gvvv ( f ) , a partir de la transformada de Fourier de v(t) (V(f)) y calcular esta densidad espectral
como V(f)*V(f), sin embargo no se ha contemplado esta posibilidad en ningn momento a lo
largo de esta exposicin.
La explicacin es la siguiente: v(t) representa un proceso aleatorio con, tericamente,
infinitas realizaciones, siendo cada realizacin una funcin del tiempo, as en todo caso,
podramos considerar a la funcin v(t) como una de las infinitas realizaciones del proceso y
aplicar la Transformada de Fourier a esta realizacin particular. Desde luego, nada nos impide
realizar esta operacin, salvo que no existe (por su propia naturaleza de seal aleatoria) una
expresin de esta funcin, aunque esto no es un obstculo insalvable dado que puedo
aproximar esta Transformada muestreando convenientemente v(t) y aplicando un algoritmo
de T.D.F o T.R.F (transformada discreta de Fourier o transformada rpida de Fourier), de
todas formas, poco se habra avanzado, puesto que otra realizacin cualquiera del proceso
tendra otra Transformada de Fourier, se asemejaran, pero ciertamente seran diferentes ya
que cada variable aleatoria v(ti) toma valores distintos para cada realizacin. Se puede inducir
de lo dicho que aplicando la T. de Fourier a todas y cada una de las realizaciones y
encontrando la media de todas ellas, obtendramos una Transformada de Fourier V(f), que
podra ser considerada como representativa del proceso, sin embargo es obvio lo inoperante
del procedimiento.
Al tomar nicamente una realizacin para extraer la T. de Fourier, estamos considerando
solo uno de los posibles valores que puede tomar cada variable aleatoria dentro del rango de
su funcin de densidad de probabilidad, y esto es muy poco para determinar la T. de Fourier
de todo el proceso.
Centrmonos ahora en el significado de la funcin de auto correlacin para tener una idea
ms intuitiva. Por comodidad seguiremos suponiendo que el proceso es estacionario y
ergdico.
Si se dispone de una funcin del tiempo x(t) que representa una realizacin de un proceso
aleatorio y se desea correlacionarla con ella misma, debemos operar (suponiendo x(t) de
216

energa finita para no complicar la expresin con el lmite para T) tal como indica la conocida
expresin siguiente:

Rxx ( )

x(t ) x

(t )dt

Al representar x(t) una variable aleatoria diferente para cada t, en lneas generales, esta
expresin nos est dando la relacin o dependencia que existe entre dos variables que estn
separadas una distancia y esta relacin no es aleatoria puesto que viene impuesta por la
naturaleza del proceso. Por ejemplo, para el ruido blanco gaussiano las variables aleatorias
son independientes entre si, por tanto, la funcin de auto correlacin ser cero para cualquier
valor de que no sea el valor nulo.
Como el proceso es estacionario, proporciona los mismos resultados "mirar" en el tiempo
que en las realizaciones.
En la figura 6.8, se ha plasmado conjuntamente el proceso aleatorio v(t) y el mismo
proceso desplazado en el tiempo v(t-). En esta figura se intenta mostrar la idea intuitiva de

que la expresin Rvv ( ) v(t )v (t )dt que proporciona la auto correlacin en una

realizacin, proporcionara el mismo resultado si se aplica en la direccin de las realizaciones.


AUTOCORRELACION DE UN PROCESO ALEATORIO

v1(t)

v2 (t)

v3 (t)
Re
al
iz
ac
io
ne
s

..

..
.

t
.

vm(t)

<
>
desplazamiento

tiempo

Figura 6.8. Auto correlacin de un proceso aleatorio

217

Para obtener cada valor, la funcin de auto correlacin obtiene informacin de todas las
variables aleatorias que intervienen en el proceso y no de una sola como cuando se realiza la
Transformada de Fourier sobre una realizacin
Es preciso hacer notar que la auto correlacin es una funcin determinista (no es aleatoria)
y representativa del proceso, por tanto la densidad espectral de potencia, resultado de aplicar
la T. de Fourier a la auto correlacin, tambin ser determinista y representativa del proceso.

6.4 DENSIDAD ESPECTRAL DE POTENCIA DE RUIDO DISPONIBLE

Como se ha enunciado anteriormente, entre los extremos en circuito abierto de una


resistencia de valor R a una temperatura fsica T, aparece una tensin de ruido v(t), cuya
densidad espectral de potencia (de tensin) Gvvv ( f ) es 2KTR vatios/hertzio. Esto se puede
modelar como un generador ideal de tensin de ruido de valor v(t) en serie con una resistencia
no ruidosa de valor R, como en la figura siguiente:
R
v(t)
v
Gvv(f) =2KTR

Figura 6.9.
Si tenemos en cuenta que la densidad espectral Ge(f) a la entrada de un sistema de funcin
de transferencia H(f), se relaciona con la densidad espectral Gs(f) a la salida como
2

Gs ( f ) Ge ( f ) H ( f ) , entonces podemos fcilmente encontrar el equivalente de Norton


del circuito anterior. Esto es:
i(t)
i
Gvv(f)=

2KT
R

Figura 6.10.
Vemos que podemos asimilar toda la problemtica del ruido con los conocimientos previos
de teora de circuitos.

218

Ejemplo

Calcular el valor de la tensin eficaz de ruido, en la banda de 0 Hz a 4000 Hz, generado


por dos resistencias R1= 500 y R2= 1000 que estn en serie.

Solucin

R1

a)

R2

v2(t)
v1(t)

Las densidades espectrales de potencia de ruido de las respectivas resistencias son

Gvv1 ( f ) 2 KTR1 y Gvv2 ( f ) 2 KTR2 . La potencia media de ruido puede ser obtenida mediante
B

la expresin N

vv

( f )df . As,

v12 N1

vv

( f )df 4 KTBR1 4 1.37 1023 300 4000 500 3.288 1014 vatios y

v2 N 2
2

vv

( f )df 4 KTBR2 4 1.37 1023 300 4000 1000 6.576 1014

La potencia suma 9.864 10-14 vatios que equivale a una tensin eficaz de 314.07 10-9
voltios ( vef 9.864 1014 ).

Ejercicio propuesto.

Realizar el ejercicio anterior pero con las resistencias en paralelo.

R1

R2

v1(t)

v2(t)

219

A la cuestin Cual es la potencia mxima de ruido que una resistencia puede disponer?,
responde el conocido teorema de la transferencia de mxima potencia, que enuncia: Un
generador con una impedancia interna Z, transfiere la mxima potencia a una carga ZL,
cuando esta carga es la conjugada de Z. En efecto, si tenemos una impedancia ruidosa
Z=R+jX, la densidad espectral de potencia de tensin es la conocida expresin

Gvvv ( f ) 2 KTR , puesto que en una impedancia Z es la parte real de esta impedancia la que
contribuye al ruido. Por otra parte, el hecho de que exista adaptacin de impedancias hace que
la mitad de la tensin de ruido del generador caiga en la resistencia de carga, lo que equivale a
una funcin de transferencia H(f)de valor 1/2, por lo que la densidad espectral de potencia de
ruido en los extremos de la carga es:

Gvvv ( f ) 2 KTR

4
4

Gc arg a ( f )

En la figura siguiente puede verse el circuito que expresa grficamente el proceso.


R+jX
R-jX

v(t)
v
Gvv(f) =2KTR

>
Gvd (f)= KT
2

Figura 6.11.
Y la densidad espectral de potencia disponible es:

Gvd ( f )

Gc arg a ( f )
R

2 KTR KT
vatios/herzio (W/Hz)

4R
2

La expresin anterior es general para circuitos R-L-C. Ntese que la densidad espectral de
potencia disponible es siempre la misma (

KT
) siendo T la temperatura fsica de la resistencia
2

o de la parte real de la impedancia.

Z (f )
v(t)
G (f) = 2 K T R e a l[Z ( f)]
vv

<
Z (f)

Figura 6.12.
220

As, en general, cualquier dispositivo pasivo visto desde dos terminales, como en la figura
anterior, equivale a una fuente de ruido de densidad espectral de potencia dada por:

Gvvv ( f ) 2 KT Re alZ ( f )
Pudiendo ceder como mximo (a una carga adaptada):

Gvd ( f )

KT
W/Hz
2

6.4.1 TEMPERATURA EFECTIVA DE RUIDO


Existen otras fuentes de ruido blanco que no son trmicas, es decir que la densidad
espectral de potencia no est relacionada con la temperatura fsica. Dicho de otro modo,
pudiera ocurrir, por ejemplo, que en el dipolo de la figura anterior la densidad espectral de
potencia Gvvv ( f ) medida en los bornes de salida, no fuera igual a 2 KT Re alZ ( f ) por
diferentes causas, en este caso podemos definir una temperatura efectiva de ruido Te diferente
de la fsica, de tal manera que se cumpla que la densidad espectral de potencia medida

Gvvv ( f ) pueda igualarse a la expresin 2 KTe Re alZ ( f ) , esto es:


Gvvv ( f ) 2 KTe Re alZ ( f )
De igual manera, este dipolo tiene una densidad espectral de potencia disponible dada por:

Gvd ( f )

KTe
W/Hz
2

Es preciso recalcar que la temperatura efectiva de ruido Te no es la temperatura a la que


estn fsicamente los elementos, sino un valor que permite igualar una ecuacin o una
expresin a una medida.

6.4.2 FILTRADO DEL RUIDO BLANCO


El ruido es un fenmeno que est presente en los sistemas fsicos y se mezcla con seales
tiles formando parte de la entrada de sistemas. Una consecuencia directa de ello es su
filtrado.
La ya conocida relacin de densidad espectral de potencia,
221

G s ( f ) Ge ( f ) H ( f )

la funcin de auto correlacin ,

ss ( )

G ( f ) H( f )
e

e j 2 f df

y la potencia media,

s 2 ss (0)

G ( f ) H( f )

df

proporcionan herramientas de suma utilidad para el anlisis del filtrado del ruido, siendo Gs y
Ge densidades espectrales de potencia de las seales de salida y entrada respectivamente y H
la funcin de transferencia del sistema.
A modo de ejemplo, pero sin prdida de generalidad, se podra considerar el sistema H(f)
como un filtro paso bajo ideal de ancho de banda B Hz, en el que entra ruido blanco de
densidad espectral de potencia Gee ( f )

W/Hz. As la funcin de transferencia del filtro es


2

2
f
f
H( f )
y naturalmente H ( f )
.
2B
2B

La salida del sistema tiene una densidad espectral de potencia Gss(f) con expresin:

f
Gss ( f )

2 2B
Cuya representacin grfica es:
Gss(f)

-B

2
B

Figura 6.13. Densidad espectral de potencia


La transformada inversa de Fourier de la expresin anterior es la funcin de auto
correlacin de la salida del sistema:

ss ( ) B sin c(2 B )
222

Y su representacin grfica es:


( )
R
ss
ss()
B

1
2B

1
2B

Figura 6.14.
La potencia media es B vatios.
Analicemos ahora los resultados comparando las caractersticas del ruido a la entrada con el
de salida.

El proceso aleatorio de entrada al sistema es ergdico, la funcin de densidad de


probabilidad de sus variables es gaussiana (consideramos media cero) con varianza ( potencia
media s 2 ) igual a , puesto que ee ( )
auto correlacin

( ) , adems por el hecho de que su funcin de

es una delta en el origen, las variables aleatorias del proceso son

independientes.

El proceso aleatorio de salida del sistema es tambin ergdico puesto que la dependencia
entre las variables tiende a cero a medida que aumenta el retardo, segn se desprende de la
funcin de auto correlacin ss ( ) . En general, en un sistema lineal, con la estadstica de sus
parmetros que no varan con el tiempo, teniendo a la entrada un proceso ergdico, la salida
es tambin un proceso ergdico. Se puede demostrar que si el proceso de entrada es gaussiano
el proceso de salida es tambin gaussiano, no puede afirmarse lo mismo para procesos no
gaussianos.
As, en nuestro caso, las variables sern gaussianas pero no independientes, puesto que la
funcin de auto correlacin muestra una dependencia con el retardo de aproximadamente
1/2B, significando que variables con una separacin de hasta 1/2B estn relacionadas.
La potencia media de la seal a la salida, B, se ha reducido proporcionalmente al ancho
de banda que tiene esta seal de salida.

223

Al filtrar el ruido blanco, una parte de sus componentes de frecuencia quedan eliminadas,
en el caso que nos ocupa las de alta frecuencia, esto tiene como efecto una reduccin en la
capacidad de la seal filtrada de variar o evolucionar con rapidez. Dicho de otra manera,
cuando una seal, por ejemplo v(t), se filtra paso bajo con un ancho de banda B, se puede
afirmar que, aproximadamente, la pendiente de v(t) nunca ser mayor que 1/B.
Al ser v(t) la realizacin de un proceso aleatorio, y en consecuencia representar un
conjunto de variables aleatorias para cada t, el razonamiento anterior conduce a intuir que
cuando una variable aleatoria toma un valor, las variables aleatorias siguientes, hasta una
determinada distancia, estn sujetas a tomar valores que hagan que la pendiente de la funcin
sea un valor admisible, decimos entonces que estas variables aleatorias son dependientes.

6.4.3 ANCHO DE BANDA EQUIVALENTE DE RUIDO


En la mayora de los sistemas los anchos de banda son lo suficientemente pequeos para
considerar constante la densidad espectral de potencia de ruido (no solo el ruido blanco).
Si tenemos un sistema con funcin de transferencia H(f) (como en la figura siguiente)
g(t)
>

n (t)
>

H (f)

Figura 6.15.
con una seal de ruido g(t) a la entrada y densidad espectral de potencia G g ( f )

, la
2

potencia N de ruido a la salida del sistema es:

N n 2 G g ( f ) H ( f ) df
2

H( f )
2

df

N H ( f ) df
2

En las dos figuras siguientes 6.16a y 6.16b, podemos apreciar el mdulo al cuadrado de la
funcin de transferencia del sistema en la primera y un rectngulo de anchura 2Bn, cuya rea
es la misma que la contenida bajo la curva H ( f )

en la segunda, esta Bn es el ancho de

banda equivalente de ruido.


224

|H (f) | 2

|H (f) | 2
Bn

|H (0 ) | 2

<

>

a)

b)

Figura 6.16. Densidad espectral de potencia del sistema a) Ancho de banda equivalente b)
Segn se ha expuesto anteriormente:

N Bn H ( 0)

As el ancho de banda equivalente de ruido del sistema es:

Bn

H (0)

H( f )

df

El concepto es simple; el ancho de banda equivalente de ruido es una cantidad tal que,
formando con ella la base de un rectngulo de altura H ( f )

a una frecuencia de inters,

(normalmente la que proporciona el mximo de la funcin, en el caso de sistemas paso bajo a


2

frecuencia nula) y calculando su rea, esta es igual al rea bajo la curva de H ( f ) .


Recurdese que el ancho de banda de un sistema se mide en el margen de frecuencias
positivas. Si el sistema resulta ser paso banda centrado a frecuencia f0 ,se pueden realizar
todos los pasos efectuados para el sistema paso bajo, proporcionando estos los mismos
resultados salvo que la frecuencia de inters ser f0.
En la figura siguiente puede verse el rectngulo cuya rea es igual a la integral del mdulo
al cuadrado de la funcin de transferencia del sistema, superpuesto a dicho mdulo al
cuadrado.
|H (f)|2
<

Bn

-f 0

>

|H (f 0 )|2

<

Bn

>

f0

Figura 6.17. Ancho de banda equivalente de un sistema paso banda


225

El ancho de banda equivalente de ruido para un sistema paso banda es:

Bn

H ( f0 )

H( f )

df

Y la potencia de ruido a la salida puede ponerse como:

N Bn H ( f 0 )

6.5 RUIDO GENERADO EN UN CUADRIPOLO

Se dispone, tal como se muestra en la figura siguiente, de un sistema constituido por un


nico cuadripolo cuya funcin de transferencia es H(f), con impedancia de entrada Zin(f) e
impedancia de salida Zout(f), alimentado con un generador de seal con densidad espectral de
potencia Gs(f) e impedancia interna ruidosa de valor Zs(f) y cargado a la salida con una
impedancia de valor ZL(f).
Z (f)
out

Z s(f)
G F (f)
Gs (f)

H (f)

Z (f)
in
>

G (f)
out
Ge n(f)

Z L(f)
>
G L(f)

G in(f)

Figura 6.18. Cuadripolo ruidoso


GF(f) es densidad espectral de potencia de ruido generada por la resistencia de fuente Rs,
Gen(f) es la densidad espectral de potencia de ruido generado internamente por el cuadripolo,
Gin(f) es la densidad espectral de potencia de ruido cedida al cuadripolo por la resistencia
ruidosa de la fuente y su expresin es:
Gin ( f )

G F ( f ) Rin
Z s ( f ) Z in ( f )

Si la resistencia Rs de la impedancia se encuentra a una temperatura fsica T0, entonces


recurdese que G F ( f ) 2 KT0 Rs
226

La densidad espectral de potencia de seal Gsin(f), cedida al cuadripolo es:


G sin ( f )

G s ( f ) Rin
Z s ( f ) Z in ( f )

Si por facilidad de manejo, llamamos ganancia de potencia g(f) al modulo al cuadrado de la


2

funcin de transferencia ( g ( f ) H ( f ) ) entonces :


Gout ( f ) g ( f ) Gin ( f )

Puesto que el mismo sistema genera ruido, la densidad espectral de potencia de ruido a la
salida del cuadripolo es:

Gnout ( f ) Gout ( f ) Gen ( f )


Si se desea la densidad espectral de potencia de ruido en la carga, esta es:

GL ( f )

(Gout ( f ) Gen ( f )) RL

Z out ( f ) Z L ( f )

( g ( f )Gin ( f ) Gen ( f )) RL

Z out ( f ) Z L ( f )

En cuanto a la seal, su densidad espectral de potencia a la salida del cuadripolo es:

Gsout ( f ) g ( f )G sin ( f )
Las potencias de ruido y de seal se expresan mediante las integrales de las respectivas
densidades espectrales de potencia, esto es, para el ruido:

N nout Gnout ( f )df (Gout ( f ) Gen ( f ))df


N nout

g ( f )Gin ( f )df Gen ( f )df

Para la seal:

S sout Gsout ( f )df

g ( f )G

sin

( f )df

Podra ocurrir que al estar el ruido superpuesto a la seal til, enmascarndola, y de ser el
nivel de ruido demasiado alto frente al de seal, esta fuera irrecuperable.
La relacin potencia de seal potencia de ruido (S/N), comnmente llamado relacin seal
a ruido representa el nmero de veces que la potencia de seal est sobre el ruido. Es un
parmetro indicativo de la calidad existente en un punto de un sistema fsico de una seal til
frente a otra u otras indeseadas.
En nuestro caso la relacin seal a ruido a la salida y a la entrada del sistema es,
respectivamente:
227

S sout

N nout

g ( f )G

sin

( f )df

S sin

N nin

g ( f )G

in

( f )df Gen ( f )df

sin

( f )df

in

( f )df

Estas expresiones, son desde luego, poco manejables pero pueden simplificarse bastante
haciendo algunas consideraciones, que en general se cumplen en un sistema real, por ejemplo
que todas las impedancias estn adaptadas. As la densidad espectral de potencia de ruido a la
entrada Gin(f), es la densidad espectral de potencia de ruido disponible, que al ser ruido
blanco se expresa por Gind

KT0
. Se podra haber llegado a la misma conclusin si en la
2

expresin de Gin(f) se considera la adaptacin de impedancias.


De mismo modo Gsin(f) ser ahora la densidad espectral de potencia mxima Gsind(f) que
el generador es capaz de ceder a una carga, llamaremos a esta potencia Ssind.
Por otra parte, podemos considerar g(f) constante e igual a "g" en toda la banda de inters
B, (esto equivale a tener un ancho de banda de ruido equivalente B)
As, la relacin seal/ruido a la salida del cuadripolo puede ponerse como:
B

S sout

N nout

g G sind ( f )df

g Gind ( f )df Gen ( f )df

g S sind
g KT0 B N en

Y a la entrada:

S sin

N nin

sin

( f )df

in

( f )df

S sind
S
sind
KT0 B N nind

6.5.1 TEMPERATURA EFECTIVA, FACTOR Y FIGURA DE RUIDO


El denominador de la expresin anterior, N nout g KT0 B N en g N nind N en , es la
potencia de ruido a la salida del cuadripolo. Muestra que existe una parte de la potencia de
ruido a la salida, Nen, que est generada en el cuadripolo. No tiene nada que ver con la
entrada.
228

Nada nos impide volver a escribir la expresin anterior de la manera siguiente:

N nout gKT0 B gKTe B gKB (T0 Te )


En donde el trmino KTe B representa una potencia de ruido entregada, adicionalmente,
por la resistencia ruidosa de entrada al cuadripolo y que debera estar a la temperatura Te para
que la cantidad gKTe B sea la potencia existente a la salida generada por el cuadripolo.
Es decir, la temperatura efectiva de ruido Te de un sistema, es la temperatura a la que
debera estar una resistencia colocada a la entrada del sistema (con adaptacin de
impedancias), considerndolo ahora no ruidoso, para que a la salida existiese la misma
potencia de ruido que genera el sistema por s mismo.
Es necesario resaltar que la potencia total de ruido a la salida del sistema, considerado no
ruidoso, es debida a una resistencia adaptada a la entrada que est a una temperatura ficticia
T0 Te , esta temperatura se suele llamar temperatura de operacin Top.

La temperatura Te es tambin ficticia, por el contrario la temperatura T0 es la temperatura


fsica a la que se encuentra todo el sistema, por ahora la llamamos tambin temperatura
ambiente.
Se define el factor de ruido F, como la potencia de ruido total a la salida del sistema
dividido por la potencia de ruido a la salida si el sistema fuera no ruidoso, estando la fuente a
temperatura fsica T0. Tambin se define como la relacin seal/ruido a la entrada de un
sistema dividido por la relacin seal/ruido a la salida del sistema si la temperatura de la
fuente es la ambiente (recurdese que para ruidos no blancos la temperatura de la fuente
puede no ser la fsica). As utilizando la primera definicin:
F

gKTe B
T
g KT0 B N en g KT0 B gKTe B
1
1 e

g KT0 B
g KT0 B
g KT0 B
T0

Se puede llegar tambin a este resultado partiendo de la segunda definicin de factor de


ruido esto es:
Sin

N in

F
Sout

N out

229

Se define figura de ruido como F 10log F


Llegados a este punto, parece pertinente efectuar algunas puntualizaciones con respecto a
los conceptos y utilizacin de temperatura efectiva de ruido, factor de ruido y temperaturas
ambiente y fsica. Un ejemplo servir para comentar y ejercitar las ideas expuestas.
Supongamos estar interesados en medir la temperatura efectiva de un cuadripolo, que tiene
un ancho de banda de ruido igual a B, as como conocer el factor de ruido de este cuadripolo.
Se dispone de una resistencia ruidosa adaptada a la entrada del cuadripolo, un termmetro
para medir la temperatura fsica de esta resistencia y un vatmetro adaptado a la salida del
cuadripolo, para medir la potencia de ruido saliente. Se procede de la manera siguiente.
Se efecta una medida de la potencia de ruido a la salida, estando tanto la resistencia
ruidosa a la entrada como el cuadripolo a una temperatura ambiente de, por ejemplo, T0=293
K, es decir, la temperatura fsica de la resistencia generadora de ruido y del cuadripolo es
293K.
La potencia medida es:
N sal1 gK ( T0 Te ) B

A continuacin se calienta nicamente la resistencia ruidosa de la entrada hasta una


temperatura T1 a la que, por ejemplo, la potencia de ruido a la salida sea el doble (por
conveniencia) N sal2 2 N sal1 que la medida anterior. As:
N sal2 gK ( T1 Te ) B

Ntese que si la temperatura del cuadripolo no ha variado, su capacidad para generar ruido
tampoco debe haber variado, es por ello que la temperatura efectiva de ruido Te se mantiene
constante.
De estas dos medidas podemos obtener por igualacin lo siguiente:
T1 Te 2 ( T0 Te )

Con lo que la temperatura efectiva del cuadripolo es:


Te T1 2 T0

Y el factor de ruido cuando la resistencia de entrada est a temperatura T0 es:


F 1

Te
T 2 T0
1 1
T0
T0

Concluimos que la temperatura efectiva Te es un parmetro exclusivo del cuadripolo y el


factor de ruido F es una relacin de potencias en las que tambin est involucrada la
resistencia de entrada. Normalmente estos parmetros estn referidos a una temperatura fsica
230

T0 que es a la que est el cuadripolo y la resistencia de entrada. La norma es tomar esta


temperatura de referencia como 293 K.
Si la potencia de ruido generado por un cuadripolo variase en la misma proporcin que lo
hace la temperatura fsica a la que se encuentra, entonces es fcil comprobar que cuando la
resistencia de entrada y el cuadripolo se encuentran a la misma temperatura el factor de ruido
F no depende de esta temperatura fsica ( F 1

Te
T
T
1 e1 1 e 2 .... ).Tambin vemos
T0
T1
T2

que podemos encontrarnos con elementos en un sistema que pueden estar a distintas
temperaturas fsicas y por tanto ser diferentes de la ambiental y de la de referencia.
Podra ahora dar la sensacin que estos parmetros son poco tiles puesto que solo son
vlidos a la temperatura en que fueron medidos. Esto es cierto, sin embargo resulta sencillo
encontrar sus valores a diferentes temperaturas a partir de otros dados como referencia.
Utilicemos otro ejemplo. Se desea encontrar el factor de ruido F1 del cuadripolo del
ejemplo anterior, sabiendo que la resistencia ruidosa a la entrada est a una temperatura T0 y
el cuadripolo se encuentra a una temperatura fsica Ta diferente de T0.
Parece lgico suponer que la capacidad del cuadripolo de generar ruido haya cambiado por
el hecho de encontrarse a otra temperatura diferente. Supongamos tambin que el ruido que
genera el cuadripolo varia en la misma proporcin que lo hace la temperatura fsica a la que se
encuentra. As, la nueva temperatura efectiva ser:

Te ( F 1) Ta
Y el factor de ruido del cuadripolo con la fuente a T0 es:

F1 1

Te
T
1 ( F 1) a
T0
T0

Ntese que si la fuente tambin estuviese a temperatura Ta, entonces F1=F. Igualmente
dmonos cuenta que, en la expresin del factor de ruido, el denominador del segundo
sumando en el miembro de la derecha corresponde a la temperatura de la fuente.
Veamos ahora otro interesante ejemplo; si el cuadripolo simula una lnea de transmisin
(un atenuador) con atenuacin L (g=1/L), es decir, presenta prdida de potencia en lugar de
ganancia, su ancho de banda es B, y todo el sistema se encuentra a una temperatura fsica T0
y est adaptado. Entonces la potencia total de ruido disponible a la entrada es es KT0B y a la
231

salida es tambin KT0B, puesto que una carga adaptada a la salida del cuadripolo vera al
mismo como una resistencia ruidosa a la temperatura fsica con un ancho de banda B.
El factor de ruido F de este atenuador es:
F

KT0 B
LKT0 B

L
g KT0 B
KT0 B

Que coincide con su atenuacin, por lo tanto tambin puede escribirse como:
L 1

Te
T0

En consecuencia la temperatura efectiva de ruido es: Te ( L 1) T0


Si la fuente generadora de ruido a la entrada est a una temperatura T0 diferente de la del
atenuador que est a Td, entonces el factor de ruido del cuadripolo es: F
la temperatura efectiva de ruido es

KTd B
LT
d y
g KT0 B T0

LTd
T
1 e Te LTd T0
T0
T0

6.5.2 RUIDO DE CUADRIPOLOS EN SERIE


Es tpico en anlisis y diseo de sistemas, encontrase en la necesidad de evaluar el ruido de
varios subsistemas colocados en serie. Nos basaremos en el ejemplo siguiente, que puede
generalizarse a un nmero mayor de cuadripolos, para encontrar la temperatura equivalente de
ruido de un conjunto formado por tres cuadripolos en cascada, con sus correspondientes
ganancias de potencia temperaturas efectivas y factores de ruido. Existe adaptacin de
impedancias, con una resistencia ruidosa R a la entrada del primer cuadripolo a temperatura
ambiente T0 y ancho de banda del sistema B. En la figura siguiente se muestra el diagrama de
bloques.

A
>
T0

F 1 g1
Te 1

B
>

F2

g2
Te2

C
>

F3

g3
Te 3

NTout
>

Figura 6.19.
En el punto A, la potencia de ruido disponible es KT0B.
En el punto B, la potencia de ruido disponible es g1 K ( T0 Te1 ) B
En el punto C, la potencia disponible es g 2 g1 K ( T0 Te1 ) B g 2 KTe2 B

232

La potencia total de ruido disponible NTout a la salida del tercer cuadripolo es:
N Tout g 3 g 2 g1 K ( T0 Te1 ) B g 3 g 2 KTe2 B g 3 KTe3 B

En esta ltima expresin pueden agruparse las temperaturas, as:

T
T
N Tout g 3 g 2 g1 KB T0 Te1 e 2 e 3
g1 g1 g 2

Si este resultado lo asociamos a un nico cuadripolo equivalente como en la figura:

F
R

T0

NTout

Te

Figura 6.20. Cuadripolo equivalente de ruido


Entonces la potencia de ruido a la salida puede escribirse como:

T
T
N Tout g 3 g 2 g1 KT0 B g 3 g 2 g1 K Te1 e 2 e 3
g1 g1 g 2

En donde el primer trmino del miembro de la derecha representa la contribucin de la


potencia a la salida correspondiente a la resistencia de entrada y el segundo trmino
corresponde a la potencia atribuible a los cuadripolos.
As la temperatura efectiva del cuadripolo equivalente es:
Te Te1

Te2
T
e3
g1 g1 g 2

Naturalmente la ganancia del cuadripolo es el producto de las tres ganancias de potencia,


escribindose la potencia de salida N Tout gKT0 B gKTe B de una forma mas compacta.
En general, la temperatura efectiva de una serie de cuadripolos en cascada con sus
correspondientes ganancias de potencia "g" y temperaturas efectivas de ruido se expresa
como:
Te Te1

Te2
T
Te4
e3

g 1 g 1 g 2 g1 g 2 g 3

Es de destacar el hecho de que a la temperatura efectiva de todo el sistema, contribuyen


esencialmente las primeras etapas.
En cuanto al factor F de ruido equivalente, se puede encontrar su expresin haciendo valer
que:
Te ( F 1) T0 ; Te1 ( F1 1) T0 ; Te2 ( F2 1) T0 ; ...............

233

As, efectuando la sustitucin en la expresin de temperatura equivalente de ruido, se


obtiene:
( F 1) T0 ( F1 1) T0

( F2 1) T0 ( F3 1) T0


g1
g1 g 2

Y al dividir por T0 el factor de ruido del cuadripolo equivalente queda:


F F1

( F2 1) ( F3 1)


g1
g1 g 2

Esta ltima expresin, es la conocida formula de Friis.

Ejercicio

Un sistema de transmisin con ancho de banda de 200 MHz, centrado a 1 GHz, tiene la
configuracin siguiente.
Fa=7 dB
75

Gp1
50 dB

Z0=75

Fa=7 dB
Gp3
40 dB

800 m

vef=2 mV

Z0=75

Fa=7 dB
Gp5
40 dB

1000 m

75

Todos los parmetros se pueden considerar constantes en toda la banda de paso. La seal
til a la entrada del sistema viene expresada por un generador de tensin de valor eficaz 0.002
voltios. La ganancia de potencia Gp de los amplificadores es conocida, as como su factor de
ruido Fa. La atenuacin del cable de transmisin de impedancia caracterstica Z0=75
colocada entre amplificadores es de 5 decibelios cada 100 metros (5 dB/100 m). Adems
existe adaptacin de impedancias en todo el sistema y su temperatura fsica T0=300K.
Se desea conocer la relacin potencia de la seal a potencia de ruido ( S / N ) a la salida del
sistema.

Ntese que los valores de factor de ruido, atenuacin y ganancia de potencia se han
proporcionado en decibelios. ( x en dB = 10 log10(x lineal)). Las expresiones no suelen estar
en dB.
Solucin.

El diagrama de bloques siguiente es ms explicativo.


75
2 mV

F1, Gp1,
Te1

F2, Gp2,
Te2

F3, Gp3,
Te3

F4, Gp4,
Te4

F5, Gp5,
Te5

75

234

La temperatura efectiva de ruido equivalente Te y el factor de ruido F de todo el sistema de


transmisin es, Te F 1 T0 Te1

Te 2
Te3
Te 4
Te5

G p1 G p1G p 2 G p1G p 2G p 3 G p1G p 2G p 3G p 4

Para obtener esta temperatura de ruido es preciso calcular las temperaturas efectivas de
ruido de cada elemento del sistema.

Para los amplificadores: Te1 Fa 1 T0 100.7 1 300 1203.5 K


Obviamente, las temperaturas efectivas de ruido Te3 y Te5 tienen el mismo valor.

Para los tramos de cable:


Gp2

1
; L2= 85=40 dB y en unidades lineales L2= 104. As Gp2=10-4 y su factor de
L2

ruido es F2=104.

Gp4

1
; L4= 105=50 dB y en unidades lineales L4= 105. As Gp4=10-5 y su factor de
L4

ruido es F4=105.
Las correspondientes temperaturas efectivas de ruidos son:

Te 2 F2 1 T0 104 1 300 3 106 K


Te 4 F4 1 T0 105 1 300 3 107 K

Ahora ya se esta en disposicin de calcular Te. As,

Te 1203.5

3 106 1203.5
3 107
1203.5

5
2857.3 K
5
5
5
4
4
4
4
4
5
10
10 10
10 10 10 10 10 10 10

Todo el sistema es equivalente a un solo cuadripolo. Esto es:


75
2 mV

Con

F, Gp, Te

G p G p1 G p 2 G p 3 G p 4 G p 5 104

75

F 1

Te
2857.3
1
10.52
300
T0

235

Por otra parte, el factor de ruido de un cuadripolo, conceptualmente, es:

F 1

Te G p KT0 B G p KTe B
N
siendo N la potencia de ruido total a la salida.

T0
G p KT0 B
G p KT0 B

As,

N F G p KT0 B
Sustituyendo: N F G p KT0 B 10.52 104 1.37 1023 300 200 106 86.47 109 vatios
y en decibelios: N dB 10 log10 ( N ) 10 log10 (86.47 109 ) 70.63 dB
En cuanto a la potencia de seal a la salida del sistema:
2

vef
2
6

4 10
10
133.3 106 vatios
S Gp
75
75

y en decibelios -38.75 dB.

As la relacin potencia de la seal a potencia de ruido a la salida del sistema es:

S 133.3 106

1541.5 y en decibelios 31.88 dB


N 86.47 109
Es decir, en recepcin, la potencia de seal es 1541 veces mayor que el ruido.

236

REFERENCIAS BIBLIOGRFICAS

Seales y sistemas
Alan V. Oppenheim, Alan S. Willsky, S. Hamid Nawab
(segunda edicin, 1997)Ed. Prentice Hall.
Tratamiento de la seal utilizando matlab v.4
C. Sidney Burrus, James H. McClellan, Alan V. Oppenheim, Thomas W. Parks, Ronals W.
Schafer, Hans W. Schuessler.
1997 Ed. Prentice Hall.
Tratamiento digital de seales
John G. Proakis, Dimitris G. Manolakis
1997 Ed. Prentice Hall.
Procesamiento de seales analgicas y digitales
Ashok Ambardar
2002 Ed. Tomson.
Introduccin a los sistemas de comunicacion
F.G. Stremler.
1993 Ed. Adison-Wesley Iberoamericana.
Seales y sistemas continuos y discretos
Samir S. Soliman, Mandyan D. Srinath
(segunda edicin, 1999) Ed. Prentice Hall.
Sistemas de comunicacion
A. Bruce Carlson.
1975 Ed. Mc.Graw-Hill.
Sistemas de comunicacion
B.P. Lathi.
1974 Ed. Limusa.
Sistemas digitales y analgicos, transformadas de Fourier, estimacin espectral.
Athanasios Papoulis.
1978 Ed. Marcombo.
Introduccin a las seales y a los sistemas
Douglas K. Lindner.
2002 Ed. Mc.Graw-Hill.
Estadstica Modelos y Mtodos, II Modelos Lineales y Series Temporales.
D. Pea
1989. Ed. Alianza Editorial
Probabilidad y Estadstica.
Louis Maisel
1973. Ed. Fondo Educativo Interamericano S.A.

237

You might also like